Prometric Patent Bar

615 comments

Please post any comments, questions or concerns related to taking the Patent Bar exam at the Prometric testing center below.

Beginning on July 26, 2004, the Patent Bar Exam moved to a computer-based format. The Patent Office has offers yearly written administration of the examination in Washington, DC, normally during the month of July. Written, paper exams will be administered only once per year, the test will be conducted at the Patent Office and the cost for such examination is higher than the cost to sit for the computerized examination.

The computerized Patent Bar Exam (administered by Prometric) is available to take at any point throughout the year. The exam consists of a morning and an afternoon session. Each session lasts 3 hours and is composed of 50 multiple choice questions. There is a 1 hour lunch break between the morning and afternoon session.

Although you will have 100 questions to answer, only 90 of the questions will be scored. You will need to answer 63 questions or more correct in order to pass (70% of the 90 questions).

Outside materials are not allowed. The testing facility provides an online, searchable version of the MPEP. Starting October 19, 2006, the exam is based on the 8th Edition, Revision 4. The facility also supplies paper and a pencil – all papers will be collected before you may leave the examination.

Once you have submitted your admission form and have been accepted to take the Patent Bar Exam, you are given a 90-day window. You must schedule and take your exam with the testing center during this window period. Weekend test dates fill up quickly, so you should schedule a test date as soon as you receive your confirmation if there is a specific day that you want to take the test. Also, you can change your scheduled test date without penalty subject to availability at the testing center and keeping in mind that the new date must fall within the original 90-day window.

The cost of the exam is expected to be as follows; a nonrefundable application fee of $40, a registration examination fee of $200, and a service fee of $150 to the administration facility.

After taking the exam, you should learn of your results immediately. You will then be mailed an official score report that will let you know whether you passed or failed.

{ 611 comments… read them below or add one }

1 GuestNo Gravatar November 14, 2007 at 12:21 am

A note on searching with the MPEP while taking the Prometric exam: my suggestion is to not enlarge the MPEP to the full screen size. This way you can move that window around in order to view the answer choices while you search.

Reply

2 someoneNo Gravatar November 25, 2007 at 7:59 am

what version of adobe acrobat is used?

Reply

3 patentbarNo Gravatar November 27, 2007 at 2:22 am

While not exactly the same, Adobe Reader 5.0 most closely mimics the search functions, etc. on the exam.

Reply

4 RonanNo Gravatar October 31, 2011 at 8:34 pm

Is Adobe 5.0 still the best match for the exam software? (I’ve checked the Adobe website, and the 5.0 download doesn’t seem to be available).

Reply

5 MirandaNo Gravatar January 17, 2012 at 1:11 pm

bump

6 TJNo Gravatar November 29, 2012 at 5:02 pm

You should still be able to get Acrobat Reader 5.0 at http://www.oldapps.com/adobe_reader.php

7 GuestNo Gravatar December 4, 2007 at 11:06 pm

Of course, know the 2002 and 2003 exams inside out. You will also want to have a good understanding of the MPEP and the content in each section. It seems as though some of the new questions on the Prometric Patent Bar Exam are extremely specific (in other words, you will have to look thme up). While you can search the whole MPEP, searching by section will cause fewer headaches.

Reply

8 patentbarNo Gravatar December 6, 2007 at 3:45 am

I agree – Recently, there is a huge shift in emphasis to more detailed questions and questions related to the Patent Cooperation Treaty, Appeals, etc. The online MPEP you get during the exam is about 3000 pages long so there is no way to memorize it. You just need to know the major sections and where to look things up. A strong understanding of title 35 of the USC is critical, and so is a knowledge of the list of “rules” which is quite manageable. The MPEP itself is a beast and is impossible to memorize.

Reply

9 MattNo Gravatar March 2, 2008 at 3:33 am

Can anyone explain the search capabilities of the MPEP document. Can you use the “search” functionality (which returns a list of all results), or are you limited to the single search field that just progresses through the document and stops at each match? Taking the exam in two weeks, and wanted to know if I needed to change my testing strategy. Thanks.

Reply

10 patentbarNo Gravatar March 3, 2008 at 8:05 pm

Matt – I think your results are returned one at a time and you have to scroll through all of the highlighted text. The functionality most closely resembles Adobe Reader v 5.0.

Reply

11 Near Future testerNo Gravatar March 5, 2008 at 10:12 pm

To my undersatnding, there are 10 (ten) questions (beta questions) among the 100 test questions and these 10 questions are NOT graded. If the tester answered correctly these 10 questions, he/she will not get the credit ( not counted) because they are not graded. (no credit) In oder to pass, does the tester need to answer additional 63 questions correctly. It means that the tester needs 73 correct answers( 10 plus 63) in order to pass????

Reply

12 TrenetNo Gravatar March 12, 2008 at 11:34 pm

Don’t worry about the functionality of the electronic MPEP provided on the exam. A well-trained chimp could operate it successfully. You are even given a tutorial before beginning the exam to familiarize you with the functions of the exam and the MPEP.

Reply

13 pbartakerNo Gravatar March 13, 2008 at 8:48 pm

does anyone know about availability of an MPEP boolean search query during the exam?

Reply

14 neilNo Gravatar July 18, 2008 at 6:55 am

All,

quick tip that can save hours of frustration. once the exam begins, on you scratch paper, write out all 27 chapters. takes about 2 minutes to do but warms you up / gets you focused on the exam. that way when you read a question, you can mentally pick up keywords from the chapter listing you just made.

Best of luck
Neil

Reply

15 spe1979No Gravatar August 3, 2008 at 2:57 am

It will be a good idea to make grids for the questions from 1-50 before answering the questions.

Reply

16 patentbarNo Gravatar August 7, 2008 at 3:40 pm

The posting quoting the CFR on user passes jogged something that I meant to post. During the first portion of the exam, I tended to search the index and the MPEP text by key word. During the second portion of the exam, it was more efficient to search the CFR appendix (or USC as appropriate for the question) by quoted rule/statute number to determine the best answer. Also noted that in some sections, keywords that were in the section would not appear during a “find” in that section. I distinctly remember not finding a phrase using the find feature and later running across the word while scanning through a section on a subsequent question. Went back and changed the answer on the previous question due to the “new”information, that was almost directly quoted in a answer choice. Find feature also seemed “finicky” about how the search phrase was entered into the find box and subtle typing changes yielded “hits” where none appeared before. Probably a good idea to practice with the Reader 5.0 to gain better search term success.

Reply

17 eunzukimNo Gravatar September 4, 2008 at 9:17 am

Is it true that only 90 Qs are graded? This seems inconsistent with the statement at the beginning of each test (that you have to get 70 right out of 100) and the Model Answers posted by USPTO which says that “credit given for all answers” in cases of “beta” Qs. I’ve seen so many inconsistent postings re. this issue I’d appreciate it if someone can clear this up once and for all. TIA!

Reply

18 RDS (same as in Intelproplaw.comNo Gravatar September 30, 2008 at 2:12 pm

I posted the same on Intelproplaw.com
——————————————————————————–
Hi folks,

I can proudly report that I passed the exam yesterday here in Lowell at Prometric.
All went well with the equipment and software but we had a power fail for 1 min in the afternoon, at Q 49 or so, my nerve system was about to leave my body…

Thanks to all for their contributions and comments on this forum site which helped me in my studying a lot.

Here is my feedback from yesterday’s test:

— A truckload of repeats but many of them were altered or modified. I recommend to “learn by heart” all the posted repeat questions because you will be able right away to answer the question and it gives you an additional 3 minutes of time for other, more tricky questions to look up the MPEP with the finder.
Do them over and over and over and also check the explanations provided in the answers.

— Watch out for the modifications of the repeat questions, some of them are tricky and difficult to recognize the slight change(s) in the wording. It seems to me as if the USPTO looks up these postings and modifies them all, sooner or later.

I have had also the Q’s (sometimes other names but same facts) on:
Mirror, perpendicular mounting,
PCT, missing Abstract
PCT, deleting benefit claim
PCT, the Costa Rica Question
PCT, the Swedish guys question
PCT, the German guy question
PCT, Canadian guys files in the US
Lip Closs, similar to Einstein,Weissman, but watch out for the wording , but it still ex parte
Spanish Phone
PCT , 20 yrs term from what date (watch out, two separate questions, separate facts, one first filed in Germany, one in the US)
PCT , is the English translation needed or not
Obviousness Q’s which I did not recognize from previous repeat questions.
Terminal Disclaimers , who can sign
40% one assignee, 40% second assignee, 20% remained with inventors, who can sign what
4 Inventors filed a patent, one inventor refuses to cooperate, what’s next
New use of an “old” chemical
Titanium Baseball (I checked the 4th of July answer)
VELCRO

In addition to several PCT Q’s, I have had a truckload of Q’s from 1200 and 1400, you really need to know Appeal, Reissue, and Reexamination. As mentioned above, knowing the repeats, you will have extra time to look up the answers. The PDF finder – assuming you have entered a key word, avoid long phrases – will get you directly to the MPEP section because most of the wording in the question is a CC of the wording in MPEP.

Regards and good luck, send an email to rdsconslt at AOL, in case you need to know more – as long as my memory is still fresh…

RDS

Reply

19 Aug25/2008No Gravatar November 12, 2008 at 9:35 pm

I took the exam for the first time on August 25th, 2008. I came close (65%) but did not pass. Part of the reason was because I had been practicing with a newer version of Adobe Reader, and on the day it was a struggle to find anything. Also, the resolution of the Mpep was so bad I had to zoom in more than 100% to read anything, and so I had to scroll from left to right. I STRONGLY agree with what others have said, practice searching on the version of Adobe that prometric offers (Adobe reader 5.0). They really should update it, its ridiculous…

Reply

20 MattNo Gravatar January 12, 2009 at 11:50 am

Regarding the Prometric facilities, what is the test environment like? Is it crowded? Noisy? Distracting? I’m wondering whether putting together the request to be in a separate room would be worth the time. Any thoughts on the testing atmosphere would be greatly appreciated.

Reply

21 AnnaNo Gravatar February 12, 2009 at 1:17 am

Hi Everyone,

I just took the pat bar and passed a couple of days ago. I want to thank people on this board and thank the blog owner! I found the blog a couple of weeks ago. Both the outline and the question analysis sections are is very helpful!

It looks like my question pool is quite similar to RDS’s (see above)

I have had also the Q’s (sometimes other names but same facts) on:
Mirror, perpendicular mounting,
PCT, missing Abstract
PCT, deleting benefit claim
PCT, the Costa Rica Question
PCT, the Swedish guys question
PCT, the German guy question
PCT amending abstract
Spanish Phone
PCT , is the English translation needed or not
Obviousness – the reasonable expectation standard
Terminal Disclaimers , who can sign
40% one assignee, 40% second assignee, 20% remained with inventors, who can sign what
4 Inventors filed a patent, one inventor refuses to cooperate, what’s next
New use of an “old” chemical
Titanium Baseball
VELCRO
priority of design app.
102(e) dates for app. before and after 11/29/2000, originated in US and outside. (I think 706.02(f)(1)? has a pretty good analysis on this; I found it during the exam and it was helpful)
Japan 45 days
maintenance fee, check returned

There were a lot of PCT Q’s, and a fair a number of Appeal, Reissue, and Reexamination.

There were also a few RCE related questions, but I cannot recall the details.

My advice is also to study the old exams. I took most of the exams between 2000-2003.

Knowing something about the possibility of these “new” questions was helpful, but I have to say that I don’t remember any of the “answers” during the exam. The “knowledge” was re-assuring, but if you study the old exams well, you will know where to look for the answers.

The exam facility is really quite. You can’t even take your watch or chewing gum in there.

It is definitely doable if you put in the time – I spent about 100 hours (mostly just taking the old exams and then checking/studying the concepts I missed; I have been working in the field so I sort of have a head start).

Best of luck to everyone who is studying to take the exam!

Anna

Reply

22 KazalNo Gravatar September 14, 2010 at 7:31 pm

Are you a patent examiner?

Reply

23 DanNo Gravatar March 6, 2009 at 5:11 pm

Hi All,

Just wanted to give you my update based on the exam which I just took… and not passed.

In my opinion the reason I did not pass the exam was due to not mastering the 1200, 1400, 1800, and 700/2100 chapters. Even thou I put about 5.5 months into studying, as a beginer was very hard to find the right direction…

It tood me about 3 months for a general review of the MPEP – through a self-based paid online course (only $495 but I do not recommend to anyone); followed by a more defined review of the material for another 2 months; followed by a month of exams. Which I don’t think was enough…

By the end of the studying, I took a week of from work considering that would be enough to master the material, and as of a day before the exam, I knew I don’t have what it takes to pass the exam.

So, here are my 2 cents in what it takes to pass the exam:
– the exam included 85% of old questions including variations
– new questions were very easy to find, or distand variations of the old questions…

To be able to pass, as everyone else stated in this blog, below are required:
– read information about all the chapters to understand the general view
– read and master the hot topics the people add to the blog
– make sure you master: 1200, 1400, 1800, and 700/2100
– take all the previous exams, and retake them till you pass all of them with 100%
– then you are ready for the exam…

Thanks,
Dan

Reply

24 Art WilliamsNo Gravatar June 12, 2009 at 9:13 am

Hi Dan,

I don’t know how to reconcile two seemingly inconsistent bits of advice. On the one hand, you and others did not find the $495 program (OmniPrep?) helpful. On the other hand, everyone strongly recommends past exams. Doesn’t the $495 program offer 90% of the current exam. If past exams are helpful, the current exam would seem to be even more so.

Thanks, Art Williams

Reply

25 peter b.No Gravatar September 16, 2009 at 11:46 pm

The new exam is not tested on omni prep

26 Matt B.No Gravatar March 11, 2009 at 11:13 am

I took and passed the test about a week ago, and will post various comments under those sections to which I believe they likely relate.

As to the Prometric facility and testing procedure, I believe there is a few things I would have liked to have known ahead of time:

– My test was scheduled for 9am, and I showed up at about 8:30, they put me in the big room and started the test, after the tutorial was over, the test instantly started at maybe 8:45. It did not start at exactly 9am, as I had assumed. As a result, other people arriving for (presumably) 9am tests were shuffled in by the employees during the first 20 minutes of my test, which was very disruptive. If given the choice again, I would have killed time until the last moment possible so that I would be one of the last in the room.

– You literally can’t bring anything in with you. I had to even remove my wristwatch, which I thought was a bit ridiculous. I’m pretty sure I’m not capable of using a analog watch to cheat. I was also not permitted to bring any Kleenex, which I assumed was a courtesy to other test-takers, rather than sniffling from a cold the entire time.

– You also cannot bring any pens/pencils into the room, and instead you are limited to the 2 pencils they give you. You are not permitted more than 2. If one gets dull, you must swap it for a new one. I specifically asked to have more than 2 and they refused.

– You are given 4 bound pages of scratch paper. If you use it all, you must trade in the entire booklet and get a new one. I made a 102 chart as well as a grid of all my answers, and obviously did not want to give that up. I ended up using one booklet in the morning and one in the afternoon. It was more surprising than anything, and wish I had practiced with that scenario. Also, I was told I was not permitted to tear any sheets out — though I’m not sure what the result of doing so really would have been.

Reply

27 sinkingNo Gravatar March 11, 2009 at 6:15 pm

I have a general question about Prometric PC based test. It might be stupid, but can anybody tell me whether the title of each charpter is shown on the test? For example, the chapter is shown as “700 Examination ofApplications” or just “700”? Thank you very much.

Reply

28 Matt B.No Gravatar March 13, 2009 at 11:24 am

I was worried about the same thing when I took the test. You CAN see the titles of the chapters in the test software. However, you can only open 1 at a time. Additionally, it’s a little tricky to see both the MPEP and the questions at the same time. You can do so by shrinking the MPEP window with your mouse and then moving it to the side/top/bottom.

Reply

29 AlisaNo Gravatar April 13, 2009 at 10:07 pm

Matt B’s info on the Prometric test process was super helpful!

Some additional notes on test center procedure and what it’s like to take the exam:
– I was told you cannot take on/off your jacket/sweater once you get inside the test room, but apparently you can if you step outside the room and leave your jacket outside the room (but that would waste time).
– My testing room was actually surprisingly somewhat warm – I didn’t mind the warmth, but you might want to ask what the temp of the room is at check in and figure out what you want to wear inside.
– You get one 15 minute optional tutorial when you start the AM session that starts the clock for your entire test- You can finish the tutorial by clicking out early, but you can also use the time to make up a numbered answer sheet on your scratch paper booklet. Note: you don’t get a tutorial before the PM session and you keep the same scratch paper, so you’d have to make room for all 100 questions.
– The optional 1 hour lunch break is strictly timed by your test computer! The 1 hour timer starts immediately and automatically after you finish the AM section! So, you have to make sure you get back to your computer on time!
– You can use the test center earmuff things or they can give you disposable earplugs (which I used). Since people will be walking in and out of the test room (they have different tests scheduled to start at different times), you might want to use earplugs! btw I could “feel” the people walking behind me during my exam, since the floor was somehow not that sturdy, but at least I had earplugs!
– They have small lockers with key locks that you can use to keep some of your belongings in, which was great, because you can’t bring anything in, not even tissue.

Hope this helps someone!

Reply

30 shivaniNo Gravatar March 11, 2012 at 12:12 pm

Hi, a quick question: I am not quite sure why you are keeping track of all your answers on the scrap papers. Is it because you fear the computer might stop working in the middle of it or else??
thanks

Reply

31 shannonwarrenNo Gravatar April 25, 2009 at 3:58 pm

These are my notes RE the exam software and timing experience. Thanks for the great website. Sorry if my notes are redundant of your note above…

The exam is administered on a PC and the testing software has two notable features: the first is a searchable MPEP and the second is the timer. This section of my write-up will cover both of these features and how to use them to your advantage.

• MPEP: The MPEP is given to you in its entirety on the examination, but a few handicaps make this fact less useful than it could be.
• First, the chapters and appendices are all broken down into separate PDF files. Thus, to search for a term you must be able to navigate to the proper PDF file quickly. You should generally know the MPEP chapter names and general contents therein. If you get tripped up on the exam, you can check the table of contents, but skipping this step is worth a few minutes over the whole exam.
• Don’t get too dependant on the MPEP. If you are looking up every question you will more than likely run out of time and fail. Be sure to work practice exams under real time constraints. Some people prefer to leave all look-up questions for the end of the exam, others want to look them up as they go. You need to know your best approach before your test day.
• When you do use the MPEP on the exam, be smart about it. Know what materials are in the big chapters (700 & 2100) and look there first if the question falls under these chapters. My fallback resources in the MPEP are “Appendix R — Patent Rules” and the Index. Between these four resources I was able to knockout 6/10 of the questions I needed to lookup. Chapter 1800 (PCT) was a surprisingly important chapter also.
• Probably three times during my exam the answer was “(e) All of the above” , where the components (a)-(d) were exactly from a list of alternatives in the MPEP. It was as though the PTO copied and pasted the MPEP directly into the exam. A smart MPEP search made these questions a snap to answer.
• What is a smart MPEP search? Generally, it is knowing the tricks of a good PDF search. My approach is to search for the unique words or combinations of words found in the call of the question. For example, search the entire term “correspondence delivered by hand” rather than “correspondence” or “hand”. Using quotes on a multiple words combination confuses the most current Adobe Acrobat Reader on Windows, but I cannot recall their effect on the ancient version of Reader used during the exam. A little trial and error will help you know how to find answers quickly on exam day.

• The Exam Timer: The time allotments are broken down as follows: (1) 15 minutes** to read the instructions, (2) 3 hours for the morning portion of the exam, (3) 1 hour lunch break, (4) 3 hours for the afternoon portion of the exam, (5) exam center survey, and (6) exam results.
• ** the “15 minutes to read the instructions” may have been 10 minutes.
• During the introduction time I followed the advice of previous test takers to make an answer sheet (example included); I also wrote out all of the mnemonics and tables I had invented to use as a reference during the examination period. Use this time wisely because you won’t get another introduction period after lunch.
• The 1 hour lunch break does not appear to have any extensions, so don’t be late coming back into the exam room! I sat down at 59 minutes 45 seconds and had to get right back to work.
• During the lunch break I read the answers and explanations of old exam questions. This may sound like a panicked thing to do, but it really did help. Several answers that I read in old exams during lunch came up in the afternoon session, so it paid off. You may elect to clear your mind and relax.
• After the afternoon session had ended my heart was pumping pretty good. I expected to see my score report as the final second of my afternoon session elapsed. I was disappointed to find an exam center survey between me and the results. This took about ten minutes to complete and I hated ever second! Yes, everyone was courteous… just give me my score!
• Eventually, you will be given your “Preliminary Test Results”. If you passed, it will say so, if you fail, it will indicate your actual score. The results are “preliminary” because there is a possibility that some question will be thrown out and thereby your pass/fail designation will change as well.
• Those with a preliminary pass should take heart. Upon obtaining a preliminary pass, a colleague of mine called John White (the primary teacher of PLI’s Patent Bar Review), who reported that he has never heard of someone getting a preliminary pass which was later changed to a fail. White was not asked whether any preliminary “fail” has been flipped.

Reply

32 VinayakNo Gravatar June 11, 2009 at 2:35 pm

Thanks to all those who took the effort for the postings above. I am planning to take the test in next few weeks. Someone mentinoed about preparing an “answer sheet” during the 1st 15 min intro part, does anyone have a sample?

Reply

33 SammyNo Gravatar June 11, 2009 at 3:27 pm

Vinayak,

Simple make two tables 1-50 and 51-100. Number 1-50 vertically and across the top label a,b,c,d,e.

A B C D E
1.
2. x x x

While going thru questions, make an x on the answer that u are sure is wrong. it will help in process of elimination

I am also taking it in the next few weeks. May I ask your study strategy? What have u done to prepare so far?

Sam

34 CWNo Gravatar June 11, 2009 at 3:29 pm

one can be found at http://freepatentbar.com/products.html#

its under cheat sheet.

35 VinayakNo Gravatar June 11, 2009 at 7:05 pm

My study strategy is based on PLI material & advise. Currently I am doing the previous exams & practice exams. I am also:
1 – Reading MPEP 700 & planning to read 2100, 1800, time permissting read 1200 & 1400
1- Signed on to websites like mypatentbar & IP law server for more info from folks who have passed the exams. I am Any other suggestions?

36 Art WilliamsNo Gravatar May 22, 2009 at 11:48 am

The copy/version of the MPEP (E8R7) presently available on the PTO web site includes a “Table of Contents” file that enumerates the names and numbers of the MPEP chapters. Clicking on a chapter in the “bookmarks” pane of the TOC file transfers to that chapter. The bookmarks pane for each chapter enumerates that chapter’s section headings. Clicking on a section in the bookmarks pane transfers to that section.

Is any of this available in the Prometric environment?

Thanks, Art Williams

Reply

37 RoyNo Gravatar May 31, 2009 at 3:35 pm

Hello Everyone,

First, thank you! To all of you who contributed to this – you are true Americans. This site is full of accurate details and pertinent, friendly advise- all of it helped me pass last week.

I was especially grateful for Advise on the exam center conditions – they do not allow watch, cannot remove jacket, and cannot keep first booklet if you request second. Also, I had to step outside the building to eat outside – no food on premises allowed! Need extra time for that.

Questions were like those by RDS and Anna above. I shall go to the relevant sections to post what I remember. It struck me that variations were more common in my exam. Especially, if it has been recommended to choose an answer on this panel (eg., ‘forward to IB), in the PCT Costa Rica case it was the correct choice but in the Swedish (?) case it was disguised within an incorrect choice – In my opinion, the correct choice was the choice that included the phrase ‘(USA) not competent site’.

Reply

38 ARNo Gravatar June 16, 2009 at 6:22 pm

Just wrote the exam and got a preliminary pass. Does the “preliminary pass” I passed the bar or the USPTO now goes and take off marks for answers to their dummy Qs.

Reply

39 VinayakNo Gravatar June 17, 2009 at 1:30 am

I am preparing using the PLI material & planning to take it in next few weeks. Need some info om testing at prometric:
1. Does it have a timer for each question & a total timer?
2. Does it have ability to bookmark & tag unanswered questions?
3. What version of acrobat is used?
4. Has anyone tested at the prometric in San Jose, CA?

Reply

40 AshwiniNo Gravatar June 17, 2009 at 11:12 pm

Hi Vinayak,

I am starting to prepare for the BAr Exam. How did u get the PLI materials? Any tips on how to prepare for studying? I am considering buying a review course.. however am not able to decide which one to buy because of the high costs. can u give some advice on this? thanks for your time..

Reply

41 VinayakNo Gravatar June 18, 2009 at 3:03 am

Ashwini,

I bought the PLI homestudy course. You may find it on sale at Ebay, craig’s list or check out this site:
http://www.intelproplaw.com/ip_forum/index.php/board,16.0.html

Regarding tips, this site has very good tips. I would suggest you go thro’ PLI material once, answer all the quiz/test/homework & prime questions before looking at the previous exams. Also, dont forget to look at the repeat questions as mentioned above by few people.

Last bu not least, get used to the electronic version of MPEP as quickly as you can since u use it on actual test.

42 VinayakNo Gravatar June 17, 2009 at 6:34 pm

A question from 2003 exam:

John filed a non-prov. pat. appl. in PTO claiming 2 distinct inventions, a combination & a subcombination. At the time of filing he recorded an assignment with all right of inventions to ABC corp. In the 1st OA the examiner required restriction & John elected combination. A year later during the pendency of appl. John filed a divisional claiming the subcombination. At the time of filing divisional John assigned all rights & interests of claimed inventions in divisional to XYZ corp. & the later party recorded the assigment within 3 months of assignment. Which of the following is false?

(A & B are wrong, for sure.)
C) ABC corp has no ownership rights in subcombo
D) XYZ corp has no ownership rights in combo
E) XYZ corp has no ownership rights in subcombo

Answer as per patware is (C). I think XYZ corp should have rights in subcombo.

Reply

43 MikeNo Gravatar July 28, 2009 at 1:00 am

ABC’s ownership was assigned and recorded properly before XYZ had ownership rights assigned. And in a divisional, the ownership rights recorded with the parent application carry over.

If it would have been a CIP I think you might be right though.

Reply

44 TaraNo Gravatar October 16, 2009 at 12:33 am

I am confused. I think (D) and (E) should be correct as ABCs ownership to the material includes the combination and subcombination, according to MPEP 306. Can someone clarify?

45 MacieNo Gravatar June 23, 2009 at 5:49 pm

Thanks all who had posted the helpful information. Are patent bar test for recent years are available some where? I am about to take a test in July, coming soon and a little nervous still.

Some of you posted the test question related to:
PCT, the Costa Rica Question
Spanish Phone
VELCRO
Japan 45 days

I do not recall I have saw questions in my old test suite, so wondering where to fine them?

Reply

46 scruffNo Gravatar July 13, 2009 at 9:52 pm

Vinayak-

The assignment to ABC corp. was made before election of the combination. It seems that ABC has ownership of the combination and the subcombination in the divisional application: MPEP 306: “In the case of a division application, a prior assignment recorded against the original application is applied to the division application because the assignment recorded against the original application gives the assignee rights to the subject matter common to both applications.” It seems as though the invention is no longer John’s to assign to XYZ, but rather ABC’s.

Reply

47 pttNo Gravatar July 16, 2009 at 9:07 pm

I appreciate the website! It helped me pass the exam on the 2nd attempt yesterday. My first exam i April was a real ‘hardball exam’ — very few repeat questions, 60% of the test on the detailed workings of appeal, reissue, reexam, PCT, and 102(e). Having had the PRG video course during the prior year, this didn’t appear to me to be the ‘typical’ exam as the PRG suggested. I did learn from it, however. Went on to study all old exam questions 2000-2003 while cross-referenceing the answers with the MPEP, and every question on this web site people had seen (also cross-referencing with the MPEP). The 2nd exam was a more typical experience as suggested by others on this site (more repeat questions-Tribell, Mario Lepieux, Bloc, Moondust, et cetera). If you get the ‘hardball’ exam, just try your best. If you pass great, if you don’t, then don’t get discouraged…take the test again, ensuring you study throughly and you will likely get a test more typical of the experience of those on this website.

Reply

48 DJNo Gravatar July 17, 2009 at 11:03 am

Ptt,

Can you advise some new/unusual questions you remember? I’d truly appreciate it. Thank you

Reply

49 pttNo Gravatar July 18, 2009 at 7:30 pm

DJ,
Here is what I remember. Best of luck!
1. Fact Scenario asks relevence of 37 CFR 1.105. Search 1.105 in MPEP, and select the answer that is FALSE (…all of the following EXCEPT… type of question). Recommendation: Have an understanding of 37 CFR 1.105 before you go into the exam, so you don’t burn time reading it during the exam to make sense out of it.
NOTE: Overall, my 2nd exam in July had about a dozen questions similar to 1) in that the fact scenario was 1-2 sentences with question dealing with a section of the MPEP actually provided by the USPTO test writers in the fact scenario of the question. I selected an initial answer, then looked it up to be sure I didn’t pick an incorrect ‘trap’ answer. I actually had to change my answer to a more correct answer on about 5 of these questions. The LESSON: If the USPTO provides the MPEP section, take the time to look it up. If you don’t, there is an even odds chance your going to PAY for it in a selection of an incorrect answer option.
2. 102(e) regarding PCT. The exam in April and my July Exam this week BOTH had two questions on PCT situations concerning 102(e). Review all the 7-8 scenarios in the MPEP at 706.02(f) BEFORE you take the Exam so when you take the exam you can dial right in to match the fact scenario with the correct situation. Other people I know who have also recently taken and passed the test have indicated they had 1-2 PCT 102(e) questions on their exams. You don’t want to miss these, because you know they are NOT Beta questions.
3. A question on what is considered a formal drawing. The fact scenario was a short, single sentence (..all of the following except) and each answer option was a short, single sentence. I was surprised that it stumped me for a bit…just the way the answer options were stated. Ans. I chose that color photos are not considered formal drawings. Incorrect (Trap) answer: formal drawings require submission with the application.
4. 102(b) question (…all of the following except): Ans. Examiner cannot make a CUMULATIVE rejection. I read this in a paragraph in MPEP 700, during my study leading upto the exam, so this is what I chose after review of the question. This is the type of question you were not likely to find exact wording of the answer options quickly in the MPEP, so you just had to know this ahead of time. Other answers were a) Examiner can make one 102(b) reference rejection, b) Examiner can use two 102(b) references to make the rejection.
5. Question regarding identification of the situation where the Examiner does not need to provide an explanation as to why he/she allowed the claimed subject matter. This question was in Roman Numeral format with I, II, III, IV scnearios. Ans. Examiner made specific rejections and the Applicant made amendments to the claims to overcome each specific rejection such that it was clearly evidient why the claimed subject matter was allowed. Incorrect (trap) answers: The other three roman numerals were sitations that the Examiner had to make expressily clear why the allance was made because it was not exactly clear from the prosecution why the allowance was made due to minor differences between the claims and the prior art.
6. Piecemeal Examination (which is TRUE?) — go to MPEP 700. Type in ‘piecemail’ and hit find. Ensure the scenarios given in the MPEP are listed that way in the answer option. Ans. (E) all the above. (A)-(D) in MPEP 707.07(g) where the answer options in the question. See Q48 on this website for further discussion.
7. PCT Question: Int’l application sent to the US RO and US RO does the Int’l Search. After the Int’l search is complete the time period is now between Stage I and Stage II and where should the correspondance be sent? Ans. Applicant can make changes to the claims and sent it to the IB. Incorrect: (A) Send changes to claims to US RO, (B) send changes of claims and spec to US RO, (C) send changes of claims and drawings to the IB….you get the idea.
8. Accelerated Examination regarding a new type of dirt used to excelerate growth for plants…what is required for submission for an invention that desires Accelerated Examination EXCEPT…Roman numerals I, II, III, IV. Ans. Select MPEP 700. Type in ‘accelerated examination’ and hit find. Check what the MPEP says versus what is found in the answers and pick out the ones that don’t apply.
9. Interview Question: When is it appropriate? This was another question where I thought I selected a reasonably correct answer. When I looked up in the MPEP I was surprised i had the wrong answer, and had to change it. Ans. An internet interview can be conducted if previously arranged by written authorization with the Examiner by the Applicant ahead of time. The email serves as a summary of the interview (put a paper copy in the file). MPEP 713.01(I). LESSON: Take the time to look it up to ensure you really sure you really selected the most correct answer.
10. See Q22 PCT, Germany. I have included further comments on this question regarding John the practitioner who is in Germany regarding the correct way to get a declaration and a reply to the USPTO on time given that it is the waning hours of the last day of the 6 month statutory period.
11. Appeal. 10.03.38a.
12. Claim counting; 4.00.13p. Ans. 8
13. 102(d) question, but not the same format as former exam question 4.00.29p. I believe this varient required the test taker to select the TRUE answer option.
14. Clearly know the differences between 37 CFR 1.99 submission and protest (no remarks, 2 mo. after publication; protest requires furnishing remarks on pertinent sections BEFORE publication). Both exams had at 2-3 questions on these topics.
15. A question regarding showing proof to the USPTO when an application was mailed using the U.S. Express mail and the USPTO never recieves it (Which answer is true? kind of question). Look up in MPEP 500. Make sure you know what the USPTO excepts as proof. Some things they will accept a copy of the article, some others they will not and the call of the question is see if the test taker knows what is acceptable proof.
16. When to properly use 1.131 declaration. Varient of old exam questions on this topic.
17. Know that unrecorded assignee can sign small entity status declaration (MPEP 400).
18. Appeal. Know what happens when Board reaffirms rejection on some claims, reverses the decision on other claims and issues a new ground of rejection. MPEP 1213.02.
19. Restriction question. Answer options provided scenarios of combination/subcombination and the species thereof on an invention and the test taker had to pick out the answer where the Examiner’s restriction requirement was proper. MPEP 800.
20. Mark Twine Broadening Reissue Question; 4.03.14p.
OVERALL:
1. I only recall two claims question on EACH exam (April and July). The ‘Smith Laminate’ question showed up on each exam (or one its varients — (04.00.17a) and a different question dealing with 35 USC 112, Parg. 6…regarding structural equivalants…quantity of claims questions much lower than the old exams.
2. 3-4 questions regarding dead inventors on the April Exam….0 questions on dead inventors on July Exam.
3. April Exam (‘hardball exam’) had many questions where I had to scroll the to the next section of the computer screen. July Exam every question fit only on the one screen (many short 1-2 sentence fact scenarios, short single sentence answer option…many “according to the MPEP except”…).
4. April Exam — No easy ‘gimme’ questions such as the Markush Claim question 4.00.45a. The April Exam required you to WORK for, and EARN the correct answer on every question. July Exam – had 2-3 gimme’ questions that included Markush Claim question 4.00.45a.
5. Take the time to get comfortable with what’s in the MPEP. In my case it’s taken me 1.5 years since I started reading it to begin to really feel like I know my way around instead of being afraid of even attempting to venture in there…there’s 200 years of law in there that is a bit overwhelming to the newcomer. The MPEP is not learned when treated as a ‘weekend warrior project’. Additionally, I have been very fortunate to also be working in the field over the last 1.5 years that has helped my understanding. With the on-line MPEP and the PROMETRICS testing format gives the USPTO the opportunity to justify asking tougher, more detailed questions as I experienced on the April exam. Knowing the overall contents of the MPEP well and how to quickly search with what you know meant more correct questions for me in both the exams I took. By the July exam, I knew the MPEP even better because I cross-referenced everything I studied since April with the online version of the MPEP which gives you every day practice, which translates into less search time per question on exam day — good thing…upper right hand clock keeps ticking down. Knowing the MPEP and how to effectively search in it was a major reason I passed the July Exam and fell a little short on the April Exam.
6. Read MPEP 1200 Appeals several times BEFORE the taking the test. Seems I had 6-7 questions on each exam regarding appeals.
7. One type of question I would advise just to ‘mark’ until you reach the end of the exam session and then go back and try to answer it if you didn’t know the answer on the first pass. This type of question is often with ‘according to the MPEP’ fact scenaio, but has answer options wherein the essence of each answer option is in a different MPEP chapter. I had at least 3 questions on the April exam and the USPTO does not put any 37 CFR XX/35 USC XX reference marker in each answer option to help you. If you don’t know where it might be to check the accuracy of the particular answer option, it’s tough sailing and burns alot of time off the clock to single out the most correct answer. Thankfully, I did not have any of these type of questions on my July exam.
8. Go over the 60 questions in this website that others have seen since the 2003 paper exams and find every answer surrounding the blog discussion in the MPEP as a help to yourself in better knowning the MPEP as discussed in 7. above and better prepared to answer varients of any given question should a similar question come your way on your particular exam.

Reply

50 patentbaracudaNo Gravatar August 10, 2009 at 12:01 pm

I am so happy to report that I passed the exam last week. This was my third attempt. I am a long time lurker of this site which is extremely valuable for passing the exam. I missed my first test Feb-2009) by just 1 Qs, the second test (May-2009) I didn’t review in detail all the newly reported Qs and got caught be surprise by Qs such as who can sign what in which situation etc (scored 65%). In the last week before my exam I studied all the newly reported Qs looking up all Qs in the MPEP to optimize my skill searching the MPEP. I also went to the USPTO to look up the Qs of my previously 2 missed exams. This all helped me tremendously. I just want to stress to how important and valuable this site is and would like to thank every single contributor to this site – this is a great community – keep going so that future test takers will also benefit from this site. Now I have to find a job after working for a very long time in the corporate world.

The exam itself contained many Qs that have been reported on this site, as well as a few old repeated questions such as Tribell, Moondust pencil, Bloc (cure for cancer – alleviate pain), Potter (102e rejection, what cannot be done to overcome), DRAM, Multiplicity/Piecemeal, PCT – Costa Rican guys, Swedish guys and German guy (A = US/RO is incompetent send appl to IB and keep F/D), RCE during Appeal, nonpublication request at US F/D => filed appl in Jp, Laminate;
a lot of 102(a), (b – including 2 not so obvious Qs re on sales bar), 102 (d) and 102 (g) and 103 Qs.

The new Qs below is a list so of Qs as far as I can recall:
•Rejection w/ 3 mo SSP, applicant relies w/i 2 mo, advisory action by exam before the 3 mo SSP – what is the date to calculate any extension of time => date of end of 3-mo SSP
•Final rejection w/ 3 mo SSP, applicant relies w/i 2 mo, advisory action by examiner after the 3 mo SSP – what is the date to calculate any extension of time => date on which the Office mails the advisory action
•Non-prov appl w/ executed oath – who can sign the appl, except? A = atty signed for single inventor who could notbe located after diligent search (wrong answer)
•Filing provisional appl – one page of drawings is missing – How can this be corrected? => file a new complete provisional appl (MPEP 601.01(d)

•Correction of inventorship – which of the following statement are correct?
(i)Using CFR 1.48(a) in a non-provisonal aplpl where error arose w/o deceptive intension on the part of the person named as an inventor or through error was not named as an inventor, the inventorship of the nonprov appl may be amended to name only the actual inventors
(ii)Using CFR 1.48(d) in a prov appl to add an inventor were omitted in the prov appl
(iii) In a reissue where assignee has 100% ownership, but one of the inventors disagrees
A = all 3 statements are correct

•All claims 1-10 rejected, claim1 is only independent and claim 2-10 are dependent. On appeal the Board affirms the rejection of c1-7, but reverses the rej on claim 8-10
A = examiner converts claim 8-10 into independent claims and issues appl, or set a 1-mo time limit in which appellant may rewrite dependent claim in independent form

•When is it proper to use a 1.377 petition to correct maintenance payment errors? None of the A provided were correct (no patent #, no appl #; file w/I 3 mo (must be 2 mo); petition signed by atty, applicant or service payment company (must be part of interest); cannot recall the other Answer)

•Correcting claim for foreign priority for an appl that was filed on or after 11/29/00 and that has been granted to claim priority to German appl – correction can be done using a reissue appl – what of the following statements are correct?
1.Identify prior appl by appl #, or be intl appl # and intl filing date, indicate relationship
2.File reissue oath identifying foreign appl
3.Submit a certified copy of priority docs in the reissue appl to perfect prio
4.File petition for an unintentionally delayed priority claim in addition to filing reissue appl
5.submit ADS
Statements 1-4 are correct, ADS is wrong – is not required

•examiner request submission of information – Which of the following statements are correct, except: A (wrong) = information that is only material to the patentability – MPEP 704.12a

•protest by third party alleging that invention was on sale > one yr prior to F/D. Protest was filed after publication of appl, but before N. Allowance- Should examiner enter protest => Protest should not be entered b/c it was not filed before publication

•Appl is filed using Express Mail (Q from 2000-Apr-PM-Q14) – USPTO didin’t receive appl- which of the following statement are correct, except? A (wrong) = file copy of entire appl using Cert of Mailing

•How can benefit claim be deleted after N. of Allow. ? A= before paying issue fee, file RCE w/ amendment to delete benefit claim

•Applicant dies before filing application – Who can sign the oath, except? A(wrong) = a joint inventor who is a minor cannot sign the oath until he is a “legal” person

•Ex parte Reexam – What prior art can be submitted, except? A (wrong) = prior art citation can include a statement as to the claims violating USC 122#1 (MPEP 2205)

•What docs can an assignee sign w/o having established ownership ? => small entity statement

•40% ownership assignee A, 40% ownership assignee B, 2 joint inventors – Who can sign the reissue application – all 4 parties = 2 assignees and 2 jv’s

•after paying issue fee, applicant wants to submit a PA reference cited by foreign counsel, but doesn’t want tp pay for RCE – What can applicant do to submit PA reference – A= applicant can send PA reference after issue fee payment, PA will be put into file wrapper, but won’t appear on issued patent (MPEP 1302.13)

•new Appeal Q – New ground for rejection by Board affirming examiner’s rejection. Applicant submitted amendment that overcame the new rejections and the affirmed rejection and made the claims allowable. I am not quite sure about the Answer, but I think the examiner can allow the application during limited prosecution before examiner w/o returning it to the Board

MPEP 1214.01 – …..”Prosecution before the examiner of the 37 CFR *>41.50(b)41.50(b)< rejection. If
the application becomes allowed, the application should not be returned to the Board.

•Q regarding correction of inventorship in an issued patent that has a 100% assignee (corporation) with inventors A & B via reissue or Cert. of Correction; company wants to add inventors C & D, and inventor B disagrees and only agrees if patent is divided so that inventor B gets his one patent, but inventor A agrees to add inv B&C. What can be done to correct inventorship?
This is a little bizarre Q – there was not really an A ‘Not possible” as reported in this site, and the other options were filing an divisional reissue is not quite correct, but Cert. of Correction cannot be used, b/c inventor B disagrees. Perhaps this is one of the beta Q.

Thank you again and good luck to everybody taking the exam.

Reply

51 patentgeekchickNo Gravatar September 22, 2011 at 9:09 pm

8. Accelerated Examination regarding a new type of dirt used to excelerate growth for plants…what is required for submission for an invention that desires Accelerated Examination EXCEPT…Roman numerals I, II, III, IV. Ans. Select MPEP 700. Type in ‘accelerated examination’ and hit find. Check what the MPEP says versus what is found in the answers and pick out the ones that don’t apply.

V – is ADS. I don’t thing ADS MUST be submitted. That’s why andwer is I, II, III, IV

Reply

52 DanNo Gravatar August 10, 2009 at 4:37 pm

Hi Guys,

Just wanted to say I took the exam today and passed. This site was very helpful with providing questions on the exam. To those about to take the test – review the questions posted on here. I had so many repeats. It made the exam a breeze. I also used the PLI course and took the practice exams religiously. I studied very hard and learned how to use the MPEP. I had a lot of questions on appeals, some PCT, 102(e) and all the stuff everyone else talks about. Study hard, study the questions and you will do fine.

Reply

53 ADNo Gravatar August 10, 2009 at 4:57 pm

Hi Dan,

Congrats on passing! I am in the 1st week of my studies for the bar.. and am stressed already. I am using the PLI materials for the prep. Can you please tell me, how should I incorporate “learning to search the MPEP” at this stage. I listened to the lectures on the CD, read the notes and try to solve the Prime Questions. Please suggest how can I study smartly and effectively. Thanks for your time!

Reply

54 DanNo Gravatar August 10, 2009 at 11:32 pm

AD,

Searching the MPEP was a concept I didn’t FULLY embrace until a week or two before the exam. At this point I had a solid knowledge of the basic’s covered in the PLI course and my brute force methods of learning the material, but being able to search really made a difference in my ability to consistently answer questions correctly. It takes the pressure off of remembering every little detail. I find that even if I *know* the answer I like to make sure with a quick search.

What I did to get better at searching was to look at questions individually (checking the answer after each question) and try to identify what section of the MPEP the question referred to and then look up. After you try to look it up, check out the answer and explanation. See where they cite the answer and confirm. At first I wasn’t that good but I could see the answer was close to where I was looking “there is a method to the madness…”

What you will find is that a lot of the answers are verbatim out of the MPEP. It won’t be long before you get reasonably good at looking stuff up.

Oh READING (or at least skimming) chapter 2100 helped me a lot. John White recommended this and it helps because a lot of questions are from 2100 and if you have some idea how it’s all broken down it makes searching that much easier. Get a grasp on 700 too. No need to read it with a microscope and take notes – just get a feel for how it’s laid out.

So you take all of the practice tests (especially the more recent ones) until you know the answer to the question before you even finish reading it. Then you look over the questions posted here (“exam questions and concepts” and repeats) and try to familiarize yourself with that material. Then when it comes time to take the test you will see like 30 repeats, some questions you can easily answer, a bunch of questions that you saw off this site, and then you are left with questions that you can search/use your knowledge on. A couple questions might be way out of left field but they will be so few and far between that it’s alright.

I found that I had more then enough time – I finished the first section in 2 hours and then went over everything for an hour. The second section took me 2.5 hours and then I got excited so I finished the exam early.

Yeah I had lots of appeal and some PCT questions, but this stuff wasn’t too bad because I could search. Most of the stuff is right out of the MPEP, it isn’t rocket science. I decided to take this exam before my first year of law school. As an undergrad I was never a great student… I put the time in for this test, at least the 150 hours. By the time I took the test it was all so familiar I had no problem passing.

Hope this helps.

Reply

55 Art WilliamsNo Gravatar August 13, 2009 at 4:09 pm

This is very helpful. I would like to know more.

How, for example, would you apply your search strategy to the titanium-baseball question?

My searching produces a few relevant passages, but I don’t read them as consistent or decisive.

My understanding is that the question focuses on three dates in the following order:
1) When the brochure went on restricted display
2) When a competitive application was filed minus one year.
3) When the brochure went on open display

As others have said, the relevant portions of the MPEP appear to be 2128 and 2133 which offer the following material

1) A reference will constitute a “printed publication” as long as a presumption is raised that the portion of the public concerned with the art would know of the invention even if accessibility is restricted to only this part of the public. But accessibility to applicant’s thesis was restricted to only three members of a graduate committee. There can be no presumption that those concerned with the art would have known of the invention in this case.).

2) … the expertise of the target audience, the existence (or lack thereof) of reasonable expectations that the material displayed would not be copied, and the simplicity or ease with which the material displayed could have been copied.

3) “Once an inventor has decided to lift the veil of secrecy from his [or her] work, he [or she] must choose between the protection of a federal patent, or the dedication of his [or her] idea to the public at large.”

4) … the inventor puts the invention on display or sells it, there is a “public use” within the meaning of 35 U.S.C. 102(b)

Can you or your search technique help me to answer the question of whether or not the date of the restricted display constitutes a 102 (b) bar?

Thanks very much, Art Williams

Reply

56 ADNo Gravatar August 11, 2009 at 8:53 am

Dan,

Thanks a million for writing a detailed response. I truly appreciate your gesture. It helps a lot. It tells me im on the right track and not totally lost as i was feeling :).

Thanks again! All the best for Law School!

Reply

57 GuestNo Gravatar August 22, 2009 at 8:17 pm

Hey, would appreciate an answer to this question for those who’ve taken the test: I understand that a searchable version of the MPEP is provided on the test on a chapter by chapter basis. My question is, how is it laid out? Do they give you chapter names like Ex Parte Reexaminations? Just numbers like MPEP 0200, or some combination thereof? Thanks in advance for your help 😉

Reply

58 GuestNo Gravatar August 22, 2009 at 8:28 pm

Are there any actual True/False questions on the exam as I’ve seen on Examware 2007? Or do all questions have 5 answer choices, A-E?

Reply

59 peter b.No Gravatar September 16, 2009 at 11:56 pm

all questions are 1-E and most are a lot more detailed than the examware questions.. You will need to know the questions on this website better than the ones on the examware…

Reply

60 ChrisNo Gravatar August 24, 2009 at 11:40 am

This site is very helpful !!!

Here is a question on PDF search:

Q: When you start a PDF search during the test, does it always start from the beginning of the chapter, or from the current page you are looking at?

For example: my search term is “102(a)”, and I am at page 2100-71, when I start a search of “102(a)”, does it start from page 2100-71 or 2100-1?

If it always starts from the current page you are looking at, it’s much more efficient b/c you can narrow down to certain section b4 you search. Otherwise it might be a headache.

Thanks a lot

Chris

Reply

61 Ch333No Gravatar August 24, 2009 at 2:11 pm

I had a friend just take the test last month. He said that the search starts from the cursor on the page/current page. (i.e. not from the beginning.)

Hope this helps.

Reply

62 ChrisNo Gravatar August 24, 2009 at 2:26 pm

Thank you Ch333!
If that’s the case, life is easier :)

63 ChrisNo Gravatar August 24, 2009 at 2:34 pm

Can you use Ctrl-C/Ctrl-V to copy key words from the question and paste them to MPEP PDF search window?

64 Ch333No Gravatar August 24, 2009 at 4:13 pm

idk. The Patware software from PLI that I use doesn’t let me copy or highlight, so if the real test does, that’s another plus.

Reply

65 Opuskv626No Gravatar September 5, 2009 at 5:12 pm

I passed the exam on sept 2, 09. 1) How long does it take to receive the official results? 2) After sending in your registration ( the form that mailed with your official results) how long before you are officially registered as a patent atty? 3) overall, how much time does it take between passing the exam (provisional results) to being able to represent patent clients?

Reply

66 JohanNo Gravatar November 14, 2009 at 6:57 pm

Did anyone ever respond to this inquiry? I just got home from the exam (preliminary pass) and am curious as to the same points Opuskv626 has raised. Any insight is much appreciated!

Reply

67 BuddyNo Gravatar September 20, 2009 at 12:30 pm

Congratulations to those who passed.
But I didn’t pass so I have to take again. I have to wait 30 says before I can re-take it and that still falls in my 90-day window.
I am still within the 30day period from the first test and I tried to register with my old USPTO ID for the second test, the system is not allowing me, saying my ID is not recognized. Looks like I can’t even register within the 30day period, will the system recognize mold ID after 30 days?
Anybody has such experience? Thanks

Reply

68 studierNo Gravatar November 18, 2010 at 4:54 pm

Buddy, what was the deal with this? Were you able to register before the 30 day period was up? Did you have to wait until the USPTO mailed you your acceptance letter?

Reply

69 wNo Gravatar February 22, 2011 at 3:39 am

You have to re-apply, you can’t just try to schedule a new date w/ prometric even though it’s within your 90 day window. When you pay to register to take the patent bar w/ the USPTO, it is only for ONE exam. I had to fill out a new form (although you don’t have to send in your transcripts and stuff again, they save it for 1 yr) and wait for a response from the USPTO to get a new ID number to schedule w/ prometric.

70 RMNo Gravatar September 22, 2009 at 2:18 pm

Could someone please advise me as to which Prometric exam center between Fremont and Santa Clara/San Jose is most preferrable? I am planning to take the test sometime in October, pending availability of dates.

Thanks a bunch.

Reply

71 ajclegalNo Gravatar September 29, 2009 at 10:19 am

I just passed yesterday on my first attempt. I did not spend one red cent on study materials and do not believe it is necessary to do so to pass. Thanks to the mass amount of repeats, this test is pretty easy if you put in the time. Here is my .02 cents on test strategy:

1. Download this outline (it is really good) and read it over a couple of times.

2. Do ALL the old exams from 2000-2003 at LEAST twice. Not only are you learning the important concepts, but you are memorizing answers, which is key. You are also learning how to use the MPEP search function, which is also key. When the real exam came, I probably looked up answers to at least 30 questions that I wasn’t sure of. You need to be scoring at least 85-90 on each test before you are ready for the real exam. (I made an outline based upon questions that I was repeatedly getting wrong).

3. Learn all the new exam topics posted here.

4. The week before the test, read MPEP chapters 700, 1200, 1400, 1800 and 2100.

That is literally all you need to do to pass this test. As for my particular exam, I had at least 30 repeats from the 2000-2003 exams. The following commonly tested newer topics also appeared:

Tradename – VELCRO
Moondust
Tribell
Japan – 45 days
Wireless Telephone
Amending the PCT Abstract – ISA rewrites
Electric Toothbrush
Death of Inventor after he’s assigned part of invention to his atty
PCT 102(e) file date – Sweden/Costa Rica
Assignee not of record signing small entity claim
Hairgel (the answer was ex parte reexam)
MDC fee calculation
Door knob – means plus function
Titamium Baseball (Prior Art – July 4)
Mirror reflective qualities
Joe Blockhead
Piecemeal Exam
Return of Maintenance fee
Parking meter
misspelled name (Jon Jones)

I had several Q’s on new grounds of rejection by appeal board
I also had a question on telephone calls to FIU

I hope this helps!

Reply

72 ajclegalNo Gravatar October 13, 2009 at 3:57 pm

I passed 9/28 on my first attempt. I did not spend one red cent on study materials and do not believe it is necessary to do so to pass. Thanks to the mass amount of repeats, this test is pretty easy if you put in the time. Here is my .02 cents on test strategy:

1. Download this outline (it is really good) and read it over a couple of times.

2. Do ALL the old exams from 2000-2003 at LEAST twice. Not only are you learning the important concepts, but you are memorizing answers, which is key. You are also learning how to use the MPEP search function, which is also key. When the real exam came, I probably looked up answers to at least 30 questions that I wasn’t sure of. You need to be scoring at least 85-90 on each test before you are ready for the real exam. (I made an outline based upon questions that I was repeatedly getting wrong).

3. Learn all the new exam topics posted here.

4. The week before the test, read MPEP chapters 700, 1200, 1400, 1800 and 2100.

That is literally all you need to do to pass this test. As for my particular exam, I had at least 30 repeats from the 2000-2003 exams. The following commonly tested newer topics also appeared:

Tradename – VELCRO
Moondust
Tribell
Japan – 45 days
Wireless Telephone
Amending the PCT Abstract – ISA rewrites
Electric Toothbrush
Death of Inventor after he’s assigned part of invention to his atty
PCT 102(e) file date – Sweden/Costa Rica
Assignee not of record signing small entity claim
Hairgel (the answer was ex parte reexam)
MDC fee calculation
Door knob – means plus function
Titamium Baseball (Prior Art – July 4)
Mirror reflective qualities
Joe Blockhead
Piecemeal Exam
Return of Maintenance fee
Parking meter
misspelled name (Jon Jones)

I had several Q’s on new grounds of rejection by appeal board
I also had a question on telephone calls to FIU

Reply

73 MichelleNo Gravatar October 14, 2009 at 8:07 pm

Hi ajclegal,

Can you describe a little more about “Joe Blockhead” Question you had? Is it related to what to do after issuing the reexam certificate OR removing a priority claim after allowance to extend the term? Thanks. -Michelle

Reply

74 sinkorswimNo Gravatar October 14, 2009 at 7:48 pm

Is there any way to get a hold of the newer tests, i.e. the 05-09 tests?

Reply

75 EmilyNo Gravatar October 20, 2009 at 12:33 pm

Roughly how long does it take the PTO to admit you to take the test once you send in your materials? I want to make sure I am timing this right. Also, I was admitted to take the test 2-3 years ago but never signed up to take it – I am assuming I will have to start the process over. Is this correct? Thanks!

Reply

76 Art WilliamsNo Gravatar October 20, 2009 at 12:37 pm

Hi Emily,

I called the PTO/OED about a month after receiving the self-addressed post card I had inserted with my application. I was told that six weeks is typical.

Good luck, Art Williams

Reply

77 EmilyNo Gravatar October 20, 2009 at 12:53 pm

Thanks for the quick reply! I am having a hard time figuring out how long the studying will take me. (I’m working full time and not really sure how much time I will have in a given week to devote to studying depending on how busy I am at work). I don’t want to have to rush through at the end, but I also don’t want to wait too long after I am ready to take the exam in case things start leaking from my brain…

Reply

78 Emily M.No Gravatar October 25, 2009 at 11:19 pm

Can someone please clarify for me what is currently being tested about claim drafting? I know at one point you actually had to draft claims, but you don’t anymore. I’m using an older (I think 2002 or 2003) version of the PLI materials, and I noticed that the next 3 CDs are devoted to claim drafting, and I’m not sure these are necessary. When did the Patent Office switch the way these are tested, and what’s the best way to prepare now? Thanks!

Reply

79 chcorNo Gravatar November 20, 2009 at 1:15 am

looked over this site a bunch before my exam took the PLI course and then jsut did exams over and over as well as the exam concepts part of this website, it was a huge help and i passed on my first time.

morning was much easier than afternoon. And morning wasnt that easy.

15-20 on re-exam
10 appeal
10 reissue

repeats i got:
Mario lepuix
moondust
Shoe polish/hairgel
revival of rexam
japan 45 days
modified (adhesive layer/light sensitive layer)
Tribell
door handle
piecemeal examination

questions i got
103c prior art disqual
jake and rake make a product rake fies for patent with him and jake as inventors.
jake thinks its his own what does he do? i put file own app interference not sure if its correct

reissue copending with reexam
reissue time extensions 3 ques
claim counting question answer was 8
terminal disclaimer who signs
appeal when claim rej ask for reconsid and rejected on new basis
how to overcome 102b 102a 102e rejections (3 separate questions)
you claim range of 8% – 11% prior art says it is 9% overcome by putting range in spec and claiming 10%
time extensions for reexam
102e dates for pct apps
PCT app where its in Japanese language applicant is us resident japanese citizen -> i put forward to Japan receiving office US not proper if its not in english
priority claim not in provisional what to do -> file new app with priority claimed and let old one go abandoned
filing IDS before issue fee paid after notice of allowance
prior art drawings have no scale on them -> irrelevant still prior art
obviousness questions overcoming and definitions
incomplete drawings in PCT
102 art when you can combine them
question about examiner reporting fraud -> only when he has indepdent knowledge of a judicial decision claiming there was fraud on the patent office
what can assignee not of record sign
2 or 3 questions on express mail and certificate of mailing.
protest after publication of app

Also to answer emily M’s question there is no claim drafting whatsoever on the test

Reply

80 PTOverstudyingNo Gravatar November 23, 2009 at 1:30 pm

Just passed yesterday. A big thank you to mypatentbar.com for a great resource for review. And, thanks to others that have posted helpful suggestions.

1. Most important thing to do: take the 2001-2003 tests until you score 90-100% every time. Once this happens, a general sense about what the PTO wants you to know and where to go to find it becomes a part of your tool box. And, that helps when all those new questions start popping up on the Prometric exam.

If you miss a question on the old tests when reviewing, go to that section in the MPEP that is indicated on the answer sheet. Spend time on that section and get the concept. Then, rinse and repeat and repeat and repeat… This really helps with getting the MPEP familiarity that will allow you to look up sections of the MPEP during the test and get the right answer.

I think this is what some of the professional review programs are trying to get across–but, it seems to come across as skim and read it independently. Why not just get the familiarity with the MPEP from what the PTO wants you to know?

2. Look at the repeat and exam concepts sections here. Do, memorize all old PCT questions: I had 10-15 questions. For me, this was the maker-breaker section. Germany, Sweden, Costa Rica, Mexican Nationals–like it has been said before on this site, this is the psychological part of the test because the answers are consistent across similar fact patterns. Just hang your hat on the right answer and don’t waiver. If you read anything, read MPEP 1800 and the outline on this site.

3. Do a review course if you are not generally familiar with patent law concepts–but, don’t waste any additional time there because this test is specific to what the PTO wants. It is not a general knowledge test. It is better to quickly get a general review and then start taking the old tests. Seriously, the old tests are it…

4. Relax the night before the exam and put the confounded stuff away. Try to schedule a weekend exam so you can start the test at 9:00 A.M. instead of 8:00 A.M. Then, you can get up and get your coffee and let the brain rev-up before heading into the test.

5. Recalled repeats:
PCT: Germany, Sweden, Costa Rica, Mexican Nationals, PCT&102(e)
Mario Lepieux
Mirror
Lancer
Moondust
Tribell
Bloc
103 Graham
Potter
Broom
Fan, Lamp, Clock
Death of Inventor
Death of Joint Inventor–affect on POA
Multiplicity

Cheers and good luck…

Reply

81 njtNo Gravatar December 1, 2009 at 2:24 am

I took the patent bar yesterday and passed (well its preliminary I guess)! Does anyone know of the likelihood that a preliminary pass won’t in fact turn out to be a pass?

Anyway, the first session was brutal, and not at all what I expected! I only had 2 repeat questions (I had understood that there would be a lot more), and I didn’t even have a high number on the MPEP chapters I had focused on. I had no PCT, 2 Appeal, and only a handful of 2100 and 700. I had a large number of 800 (at least 6), 600 (probably 8), 300, 400, and correction of inventorship. The mix seemed really weird, and at the the break I felt disillussioned because I had been averaging high 80s and 90s on the practice tests but the first session seemed nothing like it.

Anyway, after lunch was pretty easy — thank goodness, because I am sure I wouldn’t have passed if it had been a repeat of the morning. In the afternoon I had a mostly 2100, Appeal, and PCT. I had a ton of repeats including:
Multiplicity
Death of inventor
Tribell
Potter — its a variation though, so be careful
Lipgloss
PCT: Mexican Corp/Nationals and the Japanese language filing
PCT: publication of only US designation
Tip & Point (moondust)
Titanium baseball (publication version)
Japan 45 days
Return of Maintenance Fee
Piecemeal
Jon Jones (misspelling)
Small Entity error (1.28(c))

My advice (like everyone elses) is to practice and familiarize yourself with the 2002/2003 exams and all the new questions reported on this site. Although, my morning session hardly had any repeats, my familiarity with the MPEP meant I could look up at least 10-12 questions. In the afternoon, the high number of repeats really helped my confidence.

Good luck!

Reply

82 Lucifer JohnsonvilleNo Gravatar January 6, 2010 at 11:43 am

1. 3 inventors A, B, and C on a pending application. Applicant discovers the ADDRESS is incorrect for Inventor B. How do you make an address change on the application? (one inventor may be unavailable, and these answers are just surmises)

A) Assignee of entire interest sends in an Application Data Sheet correcting
B) 3 inventors declare oath with correct addresses
C) Send in notification of Inventor B’s address
D) Assignee declares oath with all the correct Inventive Entity info
E) Assignee signs a petition under some number naming all inventors A, B, C declaring all inventors and identifies Inventive Entity

What do you all think?

Reply

83 chempgentNo Gravatar February 4, 2010 at 9:40 am

Anyone figured out what the answer might be for this?

Reply

84 mdavis68No Gravatar February 4, 2010 at 9:43 am

*** I answered A).

603 Supplemental Oath or Declaration

37 CFR 1.67 Supplemental oath or declaration.

(a) The Office may require, or inventors and applicants may submit, a supplemental oath or declaration meeting the requirements of § 1.63 or § 1.162 to correct any deficiencies or inaccuracies present in the earlier filed oath or declaration.

(1) Deficiencies or inaccuracies relating to all the inventors or applicants (§§ 1.42, 1.43, or § 1.47) may be corrected with a supplemental oath or declaration signed by all the inventors or applicants.

(2) Deficiencies or inaccuracies relating to fewer than all of the inventor(s) or applicant(s) (§§ 1.42, 1.43 or § 1.47) may be corrected with a supplemental oath or declaration identifying the entire inventive entity but signed only by the inventor(s) or applicant(s) to whom the error or deficiency relates.

(3) Deficiencies or inaccuracies due to the failure to meet the requirements of § 1.63(c) (e.g., to correct the omission of a mailing address of an inventor) in an oath or declaration may be corrected with an application data sheet in accordance with § 1.76.

(4) Submission of a supplemental oath or declaration or an application data sheet (§ 1.76), as opposed to who must sign the supplemental oath or declaration or an application data sheet, is governed by § 1.33(a)(2) and paragraph (b) of this section.

Reply

85 CaseyNo Gravatar January 11, 2010 at 6:06 pm

I think I had this question on my “practice” test a month ago…

Reply

86 MichelleNo Gravatar January 20, 2010 at 9:26 pm

Can anyone provide a link to the version of the MPEP that is used for the Patent Bar? I would really appreciate it. Thanks!!!

Reply

87 Emily M.No Gravatar January 20, 2010 at 9:39 pm

Here ya go: Edition 8, revision 4! http://www.uspto.gov/web/offices/pac/mpep/old/mpep_E8R4.htm
Don’t get thrown off that most of the chapters say “revision 3” on the bottom – it’s because those chapters didn’t change in the 4th revision.

Reply

88 Patent Bar PasserNo Gravatar February 4, 2010 at 6:13 am

I passed the Patent Bar today (2-3-10) on my first try. Here is my thorough recap:

Note, when I say “Answer” below, that does not necessarily mean that is the correct answer. That is either the answer I chose on the exam OR that is a concept you have to know in order to eliminate a wrong answer choice.

“New” Questions

Titanium Baseball
Answer: Prior art as of date article indexed in library catalogue

Mirror
Answer: mirror discloses parallel and perpendicular and PHOSITA would not be able to understand which it is

Amending Abstract
Answer: if the Abstract is provided by the ISA, practitioner has 1 month to submit comments on said Abstract

Deleting benefit Claim
Answer: pay issue fee and file RCE

Costa Rica
Answer: U.S. Receiving office is not competent; application sent to IB; application is afforded international filing date as of the date it was filed with the U.S. receiving office

Sweden
Answer: U.S. Receiving office is not competent; application sent to IB; application is afforded international filing date as of the date it was filed with the U.S. receiving office

Velcro
Answer: maintain rejection under 112(2)

Information Requested From PTO (via telephone)
Answer: Ask the caller lots of questions and conclude the caller cannot establish the proper association to receive status of publication date

Trade Secret (all of the following except….type of question)
Answer: Can’t remember but had to deal with petition to expunge and public’s access to expunge or non-expunged information in RCE)

Maintenance Fee Refused Entry (MPEP 2500…(asked in the form “all of the following except”)
Answer: Can’t remember, but know that if there is no “mandatory identifier” (no patent number, no patent application number), a 1.377 petition cannot be used

Investigating Deceptive Intent:
Answer: If examiner has independent knowledge of a judicial decision of fraud, inequitable conduct or if applicant makes an admission, investigation of deceptive intent is made

Who can sign a Terminal Disclaimer (40% A, 40% B, 20% by two inventors)
Answer: Need signatures from all above

Piecemal
Answer: I, II, III, IV (all of the above)

What can an assignee not of record sign?
Answer: Assertion of small entity status

What document does not need a signature and still be accepted?
Answer: Assertion of small entity status

Appeal and examiner’s rejection or examiner’s objection to claims
Answer: If examiner objected to a claim that would otherwise be allowable except for a rejection of an independent claim, the objected claim dies
Answer: If examiner rejected a dependent claim, Board reverses examiner, then applicant allowed to rewrite dependent claim to independent form or Examiner should allow applicant 1 month to rewrite dependent claim to independent form

Third Party Submission:
Answer: Limited to patents and publications; know that it should not contain any explanation or underlining or notations when submitted

Combination / Subcombination
Answer: Very tough question, using process of elimination, I was able to pick the correct answer…after about 15 minutes! It’s tough because the wording used in the choices do not match up with the MPEP, but are pretty bad paraphrases.

What is required to receive a filing date for a provision application?
Answer: only the written description and any drawing if necessary; know that oath and fee can be submitted later

John Jones spelled incorrectly as Jon Jones…what is cheapest way to correct inventorship after the patent has issued?
Answer: File ADS; the wrong answer said to telephone examiner and have examiner change the declaration—this has to be wrong because once a declaration is filed, it cannot be amended in any way, especially by a third-party (i.e. the examiner) who isn’t the declarant!
A couple of question on Restriction Practice and on CIP.

The question where there is inventor A and B, who assign interest to Corporation. A patent is issued. Corporation later determines that C and D should be named as inventors. B, who was fired from Corporation, disagrees and thinks only A and B are inventors.

A question where the facts state that the two inventions were not commonly owned. Question was about how should examiner proceed? From that sole fact, I eliminated the three answer choices that had the words “double patenting.” I think double patenting rejections are only applicable if the two inventions are commonly owned.

The question about realizing that inventor’s address (this is not the correspondence address) is missing from the declaration. What to do? The correct answer had to do with B needs to file a supplemental declaration that names the ENTIRE INVENTIVE ENTITY, but only signed by B. The wrong answer choice said to name only B and signed only by B. This was another tough question because it required me to look up something in the MPEP, which cross-referenced a Rule. When I looked up said Rule, it cross-referenced another rule! It was very time-consuming.

A question requires you to know that for divisional and continuations, a prior recorded assignment recorded against the original application is applied to the divisional and continuation. But for substitute and CIP, a prior assignment of the original application is NOT applied to the substitute or CIP application.

Repeats

April 2000 – AM
Tommie and Jo(4.00.23a/4.00.24a
Bond 60%C or 60%D (4.00.27a)
Obviousness (4.00.32a)
Prior Art (4.00.42a)
Reexamination (4.00.47a)

April 2000 – PM
Federal Court Decision of Validity is Not Binding on PTO in Reexam (4.00.19p)
102 (f) (4.00.44p)

April 2002 – PM
Hair Gel (4.02.37p)

October 2002 – AM
Facsimile Transmissions (10.02.49a)

April 2003 – AM
1
2
8
25
33 (Choose answer talking about on/off switch location)
37
41

April 2003 – PM
4
19
26
28 (Claim Counting where answer is 147)
32
44

October 2003 – AM
7
8
11
16
19
23
24
32
36
38 (variation…know the concept)
42
44
50 (Claim Counting where answer is 147)

October 2003 – PM
4
6
10
14 (Variation..know this concept)
18 (Big time variation of Jon Jones. I chose answer of File ADS.)
24
28
29
30
38
40 (Variation…know the concept)

The morning session was difficult. After the first go through of the morning session, I had marked 9 question to review. These 9 questions I really wasn’t sure…couldn’t eliminate many of the choices. As I was reviewing the 9th question, I ran out of time. The afternoon session was much better. I marked 8 questions for review. I was able to look up the correct answers to 6, leaving me 2 that I wasn’t 100% confident on. At this point, I went ahead and ended the exam, because I was pretty sure I got around 90% of the afternoon questions correct.

For all the questions I had to look up, I could pretty much find everything I was looking for. I think there were about 10-12 questions that I couldn’t figure out where/how to search. These questions weren’t simple look ups, but rather a specific fact pattern, that required application of the law to the facts to get the correct answer.

One thing to note ,as another person had commented is how to search the Prometric MPEP. When you click on the pull-down menu to access the chapters of the MPEP, you need to click on a button called “Open.” That opens the particular chapter you’ve selected. Then you may proceed with the search by clicking on “Find.” If you want to see a second hit, you click “Find Again.”

The bad habitat I got into was that I was say looking in Chapter 2100, which I have open. I then wanted to search in Chapter 700. Therefore, I click on the pull-down menu and select Chapter 700. I forget to click “Open.” Then I perform my search, and I don’t realize that I’m still in freakin’ Chapter 2100 searching! I caught on to this bad habit after about 4 times of inadvertently searching the wrong chapter. Be forewarned.

There were a lot of repeats in both the morning session and afternoon session as seen above. The test today has definitely changed from the 2001-2003 exams. In the 2001-2003 exams, the answer choices were nearly word for word from the MPEP or have the word “not” inserted. Therefore, in the 2000-2003 exams, you could search pretty long phrases, sometimes half of the entire sentence, so long as you knew what chapter to look in. My experience is that you can no longer do this today. Now, not only must be in the correct chapter, but you should only chose 2- word search phrase to search. Once you find the hit(s) in the electronic MPEP, you have to read the entire sentence (and before and after it) to see if that concept is what is conveyed in the answer choices. In other words, today, the answer choices today are often paraphrases from the MPEP, which makes searching more difficult. How will you know what buzz words to search for?? By reviewing the prior exams, you’ll know what the buzz words are.

My recommendation is to do all the 2001-2003 exams, three times each. Yes, this takes forever to do, but it’s worth it. The important part is to review all the explanations provided by the USPTO because they are very good. I find that all the “wrong” answer choices from the past exams, show up again as wrong answer choices on today’s exam, but on non-repeat questions. So if you know why an answer is wrong back then, you’ll be able to quickly eliminate it today. The good thing is that those “wrong” answer choices sometimes will show up VERBATIM on today’s exam. On the flip side, those “wrong” answer choices from 2001-2003 also turned into “right” answer choices today. For example, a “wrong” answer from the Oct. 2003 exam was wrong because it had the word “not” inserted in the wording from the MPEP. Today, that wrong choice is now “correct” because the word “not was eliminated. Therefore, the moral is to know why ALL past answer choices were either right or wrong.

My methodology to pass was as follows and more or less, in this sequence:

1) I have the PLI course. Go over PLI DVD course to the letter. Take notes in the margin of the PLI notebook. If John White says to shut off the DVD player and do a mini exam now, then do it. I did all the Prime Questions and Homework and Quiz excercises as scheduled. (At this point, I have not yet touched the PLI Patware CD—this comes in step 7. Also note, I did not listen to any of the PLI Audio CDs.)
2) Take officially released practice exams starting with October 2003 and ending with April 2001. This is 6 practice tests. (Note, I did not do the April 2000 or October 2000 tests, except for the known repeats reported on this website). The most time-consuming part of the practice exams is reviewing the answers. In reviewing the answers, I looked at virtually all the Rules and MPEP sections cited in both the right and wrong answers. While I did this, I took notes. All I did was hand-write verbatim from the PTO explanations of the concepts I was not understanding or had trouble remembering. (in the end, I ended up with a 25 page hand-written notes. They weren’t in any organized format…just random PTO rules and law)
3) Review the outline from this website.
4) Review the Repeat question section on this website. http://mypatentbar.com/repeat-questions/
5) Review the Exam Questions and Concepts question on this website. http://mypatentbar.com/current-questions/ Again, on a separate piece of paper, I kept notes of fact patterns and probable answer choices for these “new” reported questions.
6) Repeat step 2 from above. Take more notes.
7) Do all exams in PLI Patware.
8) Repeat step 2 from above, again! Take more notes.
9) Review sections from PLI notebook that I had tabbed from step 1 above.
10) Using Patware, create a customized exam of only Appeal questions. Take notes from Patware explanations.
11) Using Patware, create a customized exam of only PCT questions. Take notes from Patware explanations.
12) In last week prior to exam, I reviewed the Repeat Question Section and the Exam Question and Concepts sections again. For all Repeat Questions, I memorized the fact patterns and the answer choices. I did this until the point to where I could cover up the entire fact pattern and by looking only at the answer choices, I could pick the correct answer. More specifically, I read everybody’s comments who had listed the question number of their personal repeats. I then went back and re-did all the questions that people have reported.

Throughout this methodology, beginning with doing the old exams, I was constantly reviewing my hand-written notes. Focus on the known repeated questions. But honestly, what helped a lot was to know ALL the questions from the 2001 – 20003 practice exams for the concepts of the rules and laws conveyed. These concepts are tested on today’s exam.

I studied almost everyday for 3 months. I put in anywhere from 2-4 hours per day. PLI’s recommendation of 150 hours is not nearly enough. I never took a full 100 question practice exam in one day. I always just did 50 questions in one day followed with immediate review of the 50 questions (took about 2.5-3 hrs hrs to review 50 questions). The next day, I would do the remainder 50 questions followed with immediate review. I wasn’t worried about stamina. During the real exam, you’ll be pumped up that you won’t feel sleepy and time will fly by
However, be sure to pace yourself. During the exam, when the clock says 1:30 remaining, you should be on question 26 or higher.

In retrospect, I feel that I had over-prepared for this exam. There were so many concepts that I went over and over again that did not show up on the real exam. For example, I didn’t need to know how the 2-month rule works or that you have a max of 7 months to file an Appeal Brief. I was really concerned with whether a 1.136(a) or 1.136(b) petition would be available in certain situations, but that was not tested either. I didn’t have a single question on plants or biotech rules. I didn’t have any “weird” dates (e.g. things filed prior to 11-29-1999, 11-29-2000, prior to 6-8-95, prior to 5-29-2000). In other words, all of the questions I had applied to the latest rules. Nothing on PTE or PTA.

Thanks to all who have contributed. I could not have passed without this website. Good luck to all the future test takers!

Reply

89 DYNo Gravatar February 6, 2010 at 1:47 am

Congratulations for passing the exam.

Thank you for sharing these helpful information.

Combination / Subcombination
Answer: Very tough question, using process of elimination, I was able to pick the correct answer…

Do you remember what is the correct answer?

Reply

90 Patent Bar PasserNo Gravatar February 14, 2010 at 8:14 pm

I do not remember the correct answer. I do remember though that typically for most questions, the paraphrases from the mpep could all be found in consecutive sentences from the SAME paragraph. In this question, I think I had to look in three-four different paragraps to eliminate the wrong answer choices. I can’t remember if these three-four paragraps were in consecutive order. I do recall that the paraphrasing in the answer choices required very careful comparing the wrong choice with the text in the MPEP. My advice on this one is to be able to quickly do a word choice for “subcombination” to find the relevant section

91 DYNo Gravatar February 18, 2010 at 2:59 am

Thank you for the advice.

Reply

92 chempgentNo Gravatar February 18, 2010 at 5:04 pm

A question from April’01

16. Stan, through a registered practitioner, files an application for a patent. During the
prosecution of Stan’s patent, in an amendment, the practitioner admitted in his discussion as to
“all the claims” of Stan’s application, that “the most pertinent available prior art known to the
Applicants and their representatives is the Acme Patent, cited by the examiner.” Within one year
after the patent issues, Stan comes to you and wants to file a reissue to broaden his claims, based
on the fact that the Acme patent is not prior art. He has ample evidence to show that he
conceived and reduced his invention to practice before the filing date of the Acme patent. Which
of the following is true?
(A) Stan should file a reissue application accompanied by a declaration under 37
C.F.R. 1.131 to swear behind the date of the Acme reference. The statement by
the registered practitioner, who formerly represented Stan, that the Acme patent
was prior art constituted error without deceptive intent and may be corrected by
reissue.
(B) The explicit admission by the registered practitioner, who formerly represented
Stan, that the Acme patent constituted prior art is binding on Stan in any later
proceeding involving the patent.
(C) Stan should file a request for reexamination and submit the Acme patent along
with evidence in the form of affidavits or declarations showing that the Acme
patent is not prior art.
(D) Since the Acme patent was cited by the examiner and not by the registered
practitioner, who formerly represented Stan, Stan can not be held accountable for
the error. Moreover, the statement by the registered practitioner was directed to
the pertinence of the prior art and not to the issue of whether the date of the Acme
patent could be sworn behind. Accordingly, the statement has no binding effect.
(E) (A) and (D).

Answer:
I would think (A) to be the answer. But I see (B) to be the answer. Can somebody explain it to me,pl?

Thank you for the help.

Reply

93 JasonNo Gravatar February 24, 2010 at 1:50 am

The question is about what the lawyer or agent stated in order to get the patent. Here, he stated that the admission from Acme was pertinant and thus it is now and always will be prior art and the claims were modified to avoid this prior art. Thus no reissue can be sought as it would be recapturing.

Reply

94 chempgentNo Gravatar February 25, 2010 at 9:52 am

Thank you for the reply… Makes sense.

95 Pbar hopeful 2010No Gravatar February 22, 2010 at 11:49 pm

This has been a wonderful site for my patent bar prep! I have a few questions that I’m curious to hear what other people think, and please feel free to chime in if you’ve passed and remember as well!:

1) Are the 4 exams (2 from ’03 and 2 from ’02) the most important? Is it useful at all to look at anything from ’01, ’00 or even ’99, or would you recommend to spend that time to review the more recent topics of e.g. Appeal, PCT? Has anybody else noticed/observed that not only do these pre-02 exams have lots of fact patterns (as compared to recently where it seems like its “rules”), but they seem really more complicated/detailed?

2) How many repeats can I expect overall? (on average?) – e.g. at least 5? 20? it seems to vary a LOT from person to person

also, for these repeats, how many are exactly worded the same with the exactly worded answer? like, could i immediately recognize “B” or whatever without reading the entire question. i understand there are some variants

3) I took the April 2002 exam and scored a 63 first try. I thought it was much harder than any of the ’03 exams and the Oct. 02 exam (anyone agree or disagree?) Is this any indication of how I might do on the actual exam? I’m asking this b/c I took it under simulated conditions for the first time, trying to replicate the actual exam, and only saw a handful of repeats.

4) Is the general consensus that Adobe 5 is the closest replica of the MPEP for Prometric? Further, can I view the MPEP and question at the same time, or do i have to switch back to each window? Are the chapters labeled by number or by title and number?

5) Lastly, I sent in my application on the 4th, and haven’t heard a peep. The check hasn’t been cashed in yet. Is this normal or should I be worried?

Thanks!!!!!!

Reply

96 Emily M.No Gravatar February 23, 2010 at 12:24 am

Lots of questions…I’ll see if I can help on some of them.

1) I think the four most recent exams are the most important, but I don’t think it would be a waste of time to go through the older exams, at least once. If you look at the “repeat questions” part of this site (which you may have already done), you can see which questions from those exams that people have reported seeing again, so if you’re crunched for time, maybe just focus on those questions. The one downside to focusing too much on older exams is that some of the rules have changed, and that can throw you off a little. Probably the repeated questions wouldn’t be those though.

2) It probably does vary a lot, and you may just want to take a rough average of what people have reported on this site. I probably saw 15 or so. Of these, many (if not all) looked to be word-for-word repeats, with the only difference maybe being the order of the answer choices. Some had slightly tweaked the answer choices to clarify them. Another hugely useful resource is the exam concepts portion of this site as well as http://www.patentbarquestions.com where people discuss new questions they came across. Although people don’t remember the questions and answer choices word for word, you get a sense of what the question is going for, and especially after reading people’s discussion of what the answers should be, you tend to remember them. I saw several of these on my exam.

3) Sorry, I don’t really remember which exams I scored better on than others.

4) I used that, and it was pretty close. I don’t think you can look at both windows at once (although I thought I remembered someone saying you could make both windows smaller and see both, but I couldn’t figure it out on my exam). The most important thing is getting familiar with the “find” function. The chapters are labeled with title and number.

5) I don’t remember what the general turnaround time is, but I don’t think a few weeks is time to panic yet.

Good luck!

Reply

97 Pbar hopeful 2010No Gravatar February 24, 2010 at 10:42 pm

thank you for your comments E, very helpful!

98 KNo Gravatar March 2, 2010 at 4:25 pm

I am unclear about the process of searching for stuff during the exam. What exactly are we searching in … the MPEP, 35 USC, 37 CFR, or all of them? Also, where do you go to download these to mimic the testing conditions most closely? When I go to the USPTO website, I get a list of each chapter/section of the MPEP, USC, or CFR and the titles of each. Are these titles available on the version we look up during the exam? Thanks!!!

Reply

99 EmilyNo Gravatar March 2, 2010 at 4:37 pm

You’re searching the MPEP, which I believe includes 35 USC and 37 CFR as appendices – I never used those. You pull up each chapter separately from a drop-down menu that lists chapter number and title, and that is what you search. People say that Acrobat 5 (an old version) is pretty similar to the reader on the exam. I would download that (I can’t remember the website where I found it, but I just googled), and then use that to search the chapters from the USPTO website. (Make sure you’re using 8th Edition Revision 4 – the current edition on the website is revision 7 or something, but click on the link for “archives” and you can get to revision 4.)

Reply

100 alex tNo Gravatar March 2, 2010 at 8:06 pm

Took the patent bar and passed today on my first try! (3/2/2010).
Thanks to this site.

Here are my recommendations (the recommendations of “patent bar passer above on feb 4 2010 are pretty good):

1) A PLI or PRG course is nice (I had PRG since work paid for it).
2) Do the practice exams over and over and over and over and over. Focus on Oct 2002, April 2003, and Oct. 2003. A lot of the questions on the older exams are out of date (the answer is not quite right due to rule changes. Appeal is one example of that)
3) review the new questions on this site and patentbarquestions.com (I found that reading the “dump” area of that site helped a fair amount
4) Get to 92%+ on every of the 3 practice exams. When you study, don’t just memorize the answer. Actually look up every answer in the MPEP even the incorrect ones. I had several instances today where the question was reworded with old questions but instead of saying which is right *except*, it would say which is right. I had several pick I,II,III,IV of new questions reported in this site, but variants thereof. It is very important that you go look up the rules for the new questions to understand the meaning.

Also, I was able to do a 1/2 practice exam (an AM or PM) in about 40 minutes. You will need this speed so you don’t run out of time.

I had a lot of repeats. The monring was a little harder than afternoon because I had to search so damn much. With about 1 hour left I had 12 or so I had to go back and look things up more, and pretty much got to all of them using all my time. The afternoon were a lot of questions that were easier to lookup and/or repeats or variants of old questions reported that if you prepared, you could figure out.

Several questions were goofy (I even found one with a typo) so Im pretty sure those were beta questions.

My MPEP search technique was to lookup in index and then go to the section (not the most efficient, but it worked for me).

I had a lot of re-exam questions (including ex parte re-exam).
Some PCT including one where part of drawing is missing and the practitioner files something like 10 months after notice to correct what is the 102e date. (you had to find that you have 32 months + 5 monhts max from priority date), then figure out that the US provisional that the PCT claimed priority on was the date.

One on what happens in appeal if the board issues a new ground of rejection and practitioner files an amendment for some of claims (not responding to all), but ALSO files a request for rehearing. I forget the correct answer but that scenario is in the MPEP.

Lots of 112 enablement stuff. probably 5 questions…so know that.

Here are other new questions I got (some of the labels are from patentbarquestions.com so go look there and study the concept in that case) ….

lipstick

japan 45 days

147 claims

a couple on trade secrets (more than one)

maintentence fee with errors on it.

assignee of record can sign a small entity statement

cancer one

an indefinite claim whre claim was rejected as obvious claiming 5-7% of one thng and 7-11% of another , the reference had 5.5% and 8% (I put down the answer that said re-do the claim and put the 2nd item fixed at 9%)

trade secret

investigating deceptive intent

I think I had a couple on protests (mixed in with previous questions related to RCE). one of them had a fraud component (sorry my memory is fading me)…

combination/subcombination-

there were 2 or three questions on provisionals. for example, if you forget to include a drawing what can you do (I forget the answer I put, but I do remember you cant amend a provisional).

a couple on translations and perfecting priority in PCTs

correction of inventors address

correction of inventors in reissue

corection of priority in reissue

spanish 102d question (design patent)

I had a variant of red ink/black ink, the answer was a variant, but asked which of the following is true EXCEPT (pick the write in black ink )

splitting up patent claims question file a reissue and then a divisional reissue

a specific question on what a 1.131 affidavit can do (required knowing some more finer detail, I spent a lot of time looking up the minutia which is required to answer this)

another means plus function variant (how to compare a claim in a reference which uses mpf claims to the current claim which does not.)

restriction requirements -2 or so questions

laurel abott and harvey

pct question on publication when IA is only filed in US, designating only US, but applicant doesnt file national stage (who published the app and when….there was some minute detail to this)

faxing contracting state information in PCT (not allowed is the answer)

[edit] Claim for Priority via Reissue question (ansewr is file reissue plus certified copy of priority document, english translation not required)

Tribell

broadening reissue

RCE during appeal

hairgel question

mirror question but not as presented before. IT simply asked you which of the following woud be a proper 112 rejection (and then it lists various cases of mirror/reflecting surface and the purpendiclar/parallell answer (which is What I put).

federal court decisions was not a question by itself,but listed on a “what is in acccordance with MPEP” questions.

I had one where the practitioner got 20% of assignement of an application by a sole inventor for prosecuting the app, then the inventor dies. what happens to the case? (I forget the answer, but there are scenarios in the MPEP on that).

there was an obscure 102(a) and 102(g) thing, that I guess on

indefinite claim using “high” (I think I put as long as the spec had some implied meaninng of what “high” means its not indefinite).

means plus function determinng equivalence question

again, lots of repeats, just do 2003 exams and oct 2002 exams. Note how the rules have changed (follow some of the dialong on those questions on this site)

Good Luck!

Reply

101 patjdNo Gravatar March 25, 2010 at 3:33 pm

I passed last week, thanks for the great site.

I am selling my almost entirely unused PRG study materials. This includes: 1) All 4 volumes of volumes of “Patent Practice” textbook,
2) Printed practice questions corresponding to each volume of “Patent Practice,”
3) CD-ROM with over 2,000 practice questions (really helpful),
4) Printed version of MPEP (4 large binders),
5) PRG’s “Latest Tips and Tricks for Success” for passing the patent bar on your first try, and
6) Binder with all handouts from class and my notes.

Price is 750 (paid almost 3,000).

Please respond to this thread if interested.

Reply

102 Pamela WebberNo Gravatar December 10, 2010 at 10:39 am

I am Interested in purchasing if materials are stiil available.

Thanks,
P. Webber

Reply

103 SNo Gravatar March 29, 2010 at 6:58 pm

Was there anything wrong with last two posts? I see that the whole format has been changed in the last couple of hours.

Can somebody pl put out a format to use this new website?

Thanks a million.

Reply

104 TexasIPNo Gravatar March 31, 2010 at 12:36 pm

I am looking to sell my PLI materials for pretty cheap… email me at oo7swoosh@gmail.com if you are interested!

Reply

105 SallyNo Gravatar April 5, 2010 at 5:14 pm

Tex,

Did you pass?

Please tell us some of the new questions :-)
Sally

Reply

106 TexasIPNo Gravatar April 7, 2010 at 2:51 pm

Sure np Sally, I actually passed several months ago, and this was my write up:

Just passed last week on my first try! Would have posted earlier, but like one poster said, had to celebrate, and let off some steam. Anyways, I’ve got to give back, so here goes. My preparation consisted of the PLI materials and THIS WEBSITE. I basically studied for the last 4 weeks full time, and before that for about 3 weeks just glancing at stuff. I also had the bullseye outline material, but didn’t look at it too much.

After I finished listening to PLI CD’s, I started using the patware software to take old tests repeatedly, in addition to the PLI exercises. I only really worked on the 02 and 03 tests. The first time I took them cold, I was getting around 60%. The next time it jumped to 80-90%. Basically, once you do the questions once, it’s not hard to memorize them. Just look at the questions constantly and it’s not too hard.

My test was very very similar to “irrational’s” above. Like others have noted on here, the tests lately have been very focused on Appeals (1200) and PCT (1800) questions. Also, don’t get too stressed about 102(e) because the fact patters are all very similar to the 700 flowcharts which help a TON! Look over those closely. I must have had close to 35-40 repeats on the test, so that was nice. Also my morning section felt way more difficult than the afternoon one. I remember feeling iffy after the first part, but don’t get discouraged. This most likely means there are a bunch of beta questions because the second half, I felt like a crushed it. Anyways here are some of the questions I can remember… Enjoy and Goodluck!

– Velcro
– mirror (parallel/perpendicular conflict)
– 1.131, 1.132 affidavits
– germany, costa rica, and sweden nationals filing PCT in USPTO
– no ownership required for small entity status
– door handle
– investigating deceptive intent
– claim counting (147)
– piecemeal
– smith laminate
– dead inventors (several q’s)
– no certificate of mailing from foreign country
– japan 45 days
– lancer toothbrush
– bloc – synthetic z (alleviate pain/cancer cure
– smith DRAM
– tribell
– late IDS when client knew about prior art
– parking meter
– smith over jones appeal
– Larel Abbot Hardy
– Lots of Correction of Inventorship
– hairgel
– Titanium baseball (july 4th)
– 5 steps to cross road
– Supplemental Oath as amendment
– Jon to John (variation of which I think ADS was the answer)

Hope this helps, and I am also trying to sell my used PLI materials for pretty cheap. Let me know at oo7swoosh@gmail.com. Thanks!

Reply

107 SallyNo Gravatar April 7, 2010 at 3:21 pm

Thanks, Tex, and Congratulations!

Reply

108 J. SimhaNo Gravatar April 10, 2010 at 1:18 am

I am little confused over prior art date of declassifed material. There is a related quesion in 2002 October Morning session exam . Is the prior art date for 102(a) rejection different from 102(b) rejection for declassified material?

Reply

109 PeterNo Gravatar April 10, 2010 at 8:48 pm

Here is my read on this.
For declassified material, the publication date is the date of release when the material was made available to the public. If the date of release was greater than 1 year prior to the patent application, then this would be classified as 102(b). If it was less than 1 year prior to patent application then this would be classified as 102(a) under the “described in a printed publication” part of 102(a).

However, there is another aspect of 102(a), which states that: USC102(a) the invention was KNOWN OR USED by others in this country, or patented or described in a printed publication in this or a foreign country, before the invention thereof by the applicant for patent.

For the “known or used” part of 102(a) section 705.05(f) goes on to say:

MPEP 705.05(f): For the purpose of anticipation predicated upon prior knowledge under 35 U.S.C. 102(a) the above noted declassified material may be taken as prima facie evidence of such prior knowledge as of its PRINTING date [i.e. before it’s release to the public which is the publication date] even though such material was classified at that time. When so used the material does not constitute an absolute statutory bar and its printing date may be antedated by an affidavit or declaration under 37 CFR 1.131.

So if the declassified material was either printed or released to the public less than 1 year before the application–it will be cited as 102(a).

Reply

110 thwallsNo Gravatar May 1, 2010 at 11:15 am

That’s exactly how I read it as well. It seems like one of those facts that they love to insert in the answer set to try and trip you up, or at least get you to waste your time searching for it.

Reply

111 BrendaNo Gravatar April 15, 2010 at 11:11 am

Hi all, How much for the exam fee? $400 or $450.00. Thank you

Reply

112 AnonymousNo Gravatar May 2, 2010 at 4:59 pm

So I had a question. Last month I submitted my registration paperwork to take the exam and am scheduled to take it on the 18th. I said that I had not had any traffic violations that were over $100 but just realized that I could have had one in 2002, which “may” have been over $100. The problem is that it is not on my driving record anywhere as if it was expunged by the state where I received the ticket. The application says that any change before registration requires the updating of the USPTO by the submission of another application with the updated material. Should I say anything even if I don’t have any evidence that I received the ticket?

Reply

113 AlexNo Gravatar May 5, 2010 at 7:10 pm

Hi all,
I applied for the test over two weeks ago and I haven’t heard back and my check has not been cashed. Typically how long did the review process take internally at the PTO?

Maybe I’m just getting anxious, I really want to take the test.
Thanks,

Alex

Reply

114 thwallsNo Gravatar May 8, 2010 at 1:08 am

Just wondering, did you happen to send a self-addressed post card to confirm that they received your materials?

Reply

115 SNo Gravatar May 8, 2010 at 9:51 am

Nope. I forgot, but still heard back from them in 2 weeks.

116 newtopatentbarNo Gravatar May 6, 2010 at 1:42 am

Alex,
I got my confirmation letter in two weeks and I used a credit card instead of checks. My credit card was charged within a week. However, I’ve heard the waiting time could be anywhere from a few weeks to over a month. So just sit back and relax.

Reply

117 SNo Gravatar May 6, 2010 at 7:41 am

I also got my confirmation letter in two weeks and I used a CC. A friend of mine got it after 6 weeks. So, it depends.

Reply

118 thwallsNo Gravatar May 11, 2010 at 11:15 am

I have my test next week and I really wish it was sooner. Anybody else feel that way? Where you just want to get it over with.

Reply

119 SNo Gravatar May 11, 2010 at 11:28 am

I think I hear you. A three more weeks for me.

Good luck

Reply

120 RofelNo Gravatar May 29, 2010 at 9:14 am

thwalls, how do you find head hunters for patent agents? do they call you or something? I was a technical analyst for 1 and 1/2 years until I got a lay-off . my previous employer is still cutting on its number of patent lawyers lately..

anyways, i am preparing right now for state bar..(im a foreign grad from PI and hopefully I can get over with my jobless status soon)

thanks and good luck.. thanks again to mr. patentbar whoever he is.. i owe you a part of my being a patent agent right now :)

Reply

121 LimaNo Gravatar June 1, 2010 at 6:01 pm

Passed last week. I used this site and patentbarquestions.com–thanks to all who have contributed to these sites. Before going into the exam, I felt confident (based on the experiences shared by many on this site) that I would get a fair number of repeat and/or recently reported questions, leaving me more time to search in the MPEP for answers to new questions. However, what I found is that concepts were repeated (going back all the way to the 2000 exams) in new forms of questions, but there were not as many actual repeat questions as I might have expected. Based on my experience, I would advise future test-takers to study the 2000-2003 past exams for concepts (noting the changes that have occurred in the MPEP), but not necessarily for actual repeat questions. I only got 2 repeats in the morning and at most 5 in the afternoon. I got virtually none of the major recently reported questions that many others have reported getting (i.e. titanium baseball, mirror, German/Swedish/Costa Rican inventors filing in the USRO, velcro, piecemeal, etc). I may have gotten 5-10 recently reported questions (i.e. investigating deceptive intent, documents and assignee not of record can sign, combinations/subcombinations, maintenance fee/1.377). Bottom line, I think that you cannot safely assume you’ll get many repeats and recently reported questions, but it’s still valuable to study and perhaps memorize the concepts from past questions. Good luck–the test is definitely passable.

Reply

122 xplorNo Gravatar June 4, 2010 at 10:24 am

Took the exam and passed. There are about 30 repeat questions either from old exams or new questions posted on this website. The sequence of the answers maybe different but the words are almost the same. Another 10 repeats are modifications. I do not have anything new to post here, the exam contains a lot of appeal and reissue, and several PCT. As posted here above, questions include Laurel Abbot and Hardy; ABCD/BCDE; Non-signing inventor (divisional reissue); 150 degree/300 degree material; Potter; Inert gas; New ground of rejection by board; Copper substrate; Smith/DRAM; black ink; Mirror; Hair gel; lip gloss; Japanese patent (PCT); piecemeal; several 102, 103; public access to certificate of reexamination; second appeal upon board; examples of on sale; MPEP2183 about element function, interchangeability.
This website is the most helpful for my preparation. I spent 2-3 hours every day on PLI material for 2 months (twice). Any other similar material should be fine to let you familiar with MPEP. Then I spent another one and half months working on old exams and reading mypatentbar.com, as well as reading several MPEP chapters. It would be better if I have 1 or 2 weeks more on this stage, so I did not have to review all the repeated questions the day before the exam, which made me tired. I used the cheat sheet from freepatentbar.com, so I can write down my comments on each answers with simple symbols, which helps be to make priority during review of questions, instead of just mark/unmark in the exam. The “find” function in MPEP in the exam is not good and it helps me with only a few questions that have exact citation from MPEP. During review, I first looked at the title in the beginning of the correct MPEP chapter, then went to the corresponding part of MPEP and browse several pages to find answers.
The most important to the exam: (1) old exams and new questions from this website; (2) read several MPEP chapters, 700, 2100, 1200, 1400, 1800, and maybe 600. It is better to read the study guide on this website together with the chapter.
Thanks again for patentbar’s nice work on building this website.

Reply

123 SNo Gravatar June 4, 2010 at 10:22 pm

passed. Now plan on passing out. more later.

Reply

124 OffToTheRaceNo Gravatar June 8, 2010 at 11:48 pm

Here is what people can expect after passing the USPTO exam.
Exactly 1 week after I passed the exam, I received the official letter from the USPTO with a registration form and declaration for a patent agent (or patent attorney). I returned these notarized forms by express mail on the next business day. Almost exactly 4 weeks after the exam, my name was posted in the “official gazette”, for a period of about 3 weeks. Two business days after the official posting was over, I received my patent agent registration number. The total time from passing the exam to receiving the registration number was about 7 1/2 weeks.
Best of luck to everyone on this board.

Reply

125 BunnyNo Gravatar June 11, 2010 at 3:58 pm

Hi all, I just passed the exam today. I would like to thank whoever put this site together because this is the only study material I used. I did not spend a dime on anything else. I found this website about one month ago and was very glad I did. Thank you all and good luck to the ones about to take exames.

Reply

126 JimNo Gravatar June 13, 2010 at 11:52 am

Good morning all,

I will taking the test on the 18th and I was wondering as I study, does the prometric version of the MPEP include the appendicies such as appendix L and R ?

Thanks,

Reply

127 IrishNo Gravatar June 13, 2010 at 8:51 pm

I don’t remember if the appendices are available or not. The index is. I know that I passed without looking in them though (didn’t use them to study or during the exam).

Reply

128 thwallsNo Gravatar June 14, 2010 at 12:12 pm

Both appendices are available, just like every chapter of the MPEP. I relied on them more than the MPEP for studying but that was just the way I went with it.

Reply

129 Mike GarofanoNo Gravatar June 16, 2010 at 6:15 pm

Got a “Preliminary Pass” on my first attempt today. Thank you to all who support this website. It was instrumental to obtaining this result.

I was surprised as to how many non-repeat questions there were, but my advice is still to have down cold the 2002-2003 exam questions. These repeats will be your “anchors” that save time and boost confidence as you go.

I also observerd that the USPTO is getting more clever in how they construct their answers. In the old exam questions (especially those applicable to 2100), I would often search the appropriate MPEP chapter on the exact phrase in the answers. This would often take me right to the info needed for the answer. I noticed that in today’s exam, the answers were paraphrased a bit so that searching on them yielded no results.

Good luck to all.

Reply

130 PhantomNo Gravatar June 22, 2010 at 8:08 pm

Took and passed the exam today. I drove about 90 miles because the testing center near my home did not have any opening until the second half of July. The testing center is small but quiet. The staff was quite polite but they were firm about not bringing in anything to the testing room. Now onto the exam:

The exam is a lot hard than I thought. I spent about 3 and half months, part time, doing PLI twice and all the questions from 1997 to 2003. The real questions felt much more difficult. I could finish the 2002 or 2003 exam between 2 hours to 2 hour 15 minutes and get high 80 to 90. Today each session took me about 2 hour 30 minutes. There were about 8 PCT questions, two on how to correct a priority claim based on a foreign filing or national filing and two on 102(e) date. A general search of MPEP often yielded nothing because the answers were not straightforward from the MPEP, like before, anymore. Instead, they were paraphrased or added some words. For exam, the MPEP would say “claims” and there answer is word for word except it used “documents.” I felt like every single question I had to use elimination and often was down to two similar choices and must use my intelligence to choose one over the other. There was one question I could not find anything so I guessed purely, which never happened during prep. No easy questions like attorney ethics or multiple dependent claims.

There are a few repeated questions. That was the main reason I spent a lot more time. The ones I could remember are:

04-00-14(a) Practitioner Costello
Variant of 04-00-19(b)
variant of 10-01-11(a)
variant of 10-11-46(p)
variant of 10-02-5(p)
04-03-22(a)
04-03-30(a)
variant of 04-03-44(a)
04-03-50(p) 147 claim counting
10-03-07(a) compound Y
10-03-4(a) copper alloy
10-03-09(a) DRAM
Japan 45 days
Corporation has no duty to disclose.

Thanks everybody for help. I will post more if I could remember other questions.

Reply

131 lets goNo Gravatar June 27, 2010 at 12:22 am

I understand some of you took an exam starting at 8am or 9am. Did it finish after 8 hours since starting the exam i.e. 8am to 4pm, 9am to 5pm? I checkecd the availability at Prometric and the reserved time was 8 hours duration. However, isn’t it 7hours in total -the test time 3 hours for the first part of the exam, 1 hour break and 3 hours for the second part of the exam? I heard there would be questionair about the test site in the end but would it be finished probably about 10 mins?

Reply

132 DerekNo Gravatar July 15, 2010 at 6:47 am

I just took it (and passed) on yesterday. I my scheduled start time was 8AM, but I was able to start at around 7:30, shortly after I arrived. I was done by 3PM. The 7 hours is as you say (3/1/3), but there is 15 minutes of tutorial before the exam, and a few minutes of survey afterwards. So, probably not a bad idea to conservatively allot 8 hours.

Reply

133 YCNo Gravatar July 8, 2010 at 3:07 am

Thanks for all the help I passed on the 2nd try. Repeat ?’s were all those mainly from Oct. 03 and I had several in the morning and afternoon session. IN addition there were some crazy scenarios like submitting a drawing in an IA with US as R/O and requirements for correcting a drawing that was fulling disclosed but sheet missing after 2 mos, GREAT WEBSITE THANKS!

Reply

134 darthNo Gravatar July 21, 2010 at 3:00 pm

Took the patent bar yesterday and passed on first try thanks in large part to this site. I also used the PLI materials and the index cards from the TAPRE course (which you can now purchase separately on their site).

I’ll post more on the repeat newer questions already listed on the site and the ones from old exams in those sections. I could not agree more with those previous posters who recommended having old test/repeat questions down cold. During the test it was extremely valuable to see a question I recognized and knew the answer for and being able to spend <30 seconds making sure the Q&A were the same and answering. I was unlucky and only had 15-20 repeat questions some posts I've read people had ~40 however each one I saw was free points and extra time.

I "took" the Oct 2002, April 2003 and October 2003 3 times and reviewed the listed repeat questions from those and the other listed exams on this site twice. After reviewing a few practice tests I recommend using one or two of the 50 questions sections you have not looked at and recreate an actual timed test scenario. There will be repeats from some of the previous tests you've already reviewed so it will be close to way the actual test will go. I practiced this way with the two sections from the October 2003 and it definitely helped me priorities the time I spent in the MPEP during the actual exam.

Be sure to review the Exam Questions and Concepts section as well. I had 15-20 of these questions on the test. Some don’t have the exact Q&A nailed down so be sure to investigate the relevant MPEP passages. This serves two purposes; you will get an idea of how you would answer this question before hand and if you have to go to the MPEP for the answer you will know exactly where to look.

Reply

135 darthNo Gravatar July 21, 2010 at 3:24 pm

Also, besides the usual 600, 700,1800, 2100 questions (600 was definitely the fewest from the group) I had a ton of 1200 Appeal and 1400 Correction of Patents questions, at least 10 from 1200 and maybe that many from 1400. It also seemed like there were more reexam questions that I was expecting 5+.

Reply

136 MrjazzNo Gravatar July 28, 2010 at 8:59 pm

Can not locate the cheat sheet that everyone is referring to in the Freepatentbar website. Anyone have a copy and can email it to me? mrjazzitup@hotmail.com.

Thanks a million.

Reply

137 KQNo Gravatar August 7, 2010 at 11:55 am

I am taking the exam tomorrow and just found the following link. For anyone who is wondering what the exam software is actually like, there is a tutorial here:
http://www.prometric.com/demos/uspto/starthere.htm

Reply

138 ATNo Gravatar August 9, 2010 at 12:46 am

Took and passed today. I relied heavily on this site for my preparation. I printed flash cards of 2002-2003 repeats, and reviewed these until they became automatic. Despite reports here of the old repeats becoming rarer, I had a ton (seemingly close to 50%). There were also plenty of the “newer” repeats — like the mysterious mirror question, and several instances involving inventors of various nationalities wrongly trying to use the USPTO as their receiving office. If you are well drilled in these, you should have plenty of time to cull the MPEP for the less familiar, more challenging questions (I used a lot of this time to work through some 102(e) questions). For almost every question there is a line in the MPEP or the CFR exactly on point (even if these aren’t reproduced verbatim in the answer choices so much anymore).

As some here have noted, they appear to be testing the topics of appeal and PCT heavily recently. I also got a LOT of obviousness (103) questions, but this may be a fluke (anyone else?).

Thanks to everyone who contributed to this great site.

Reply

139 KQNo Gravatar August 11, 2010 at 10:07 am

I also had a LOT of obviousness questions on 8/8/10.

Reply

140 AshNo Gravatar August 10, 2010 at 10:32 pm

Hi im selling my patent bar exam materials for $600 (+S&H). Please let me know if anyone is interested. the materials are current. includes lecture audio cds, study materials, workbook, exam software. email me at : ashwini.durve@gmail.com if u need more info.
Thanks!!

Reply

141 AaronNo Gravatar August 10, 2010 at 11:11 pm

I passed the test today. Thanks to everyone for the contributions on the site. After reviewing my own materials, this website really put me over the edge to get prepared for the test. The “new” questions are invaluable and several came up on my test (“mirror”, “sweden and costa rica PCT”, japan pct) exactly as quoted.

I’m selling my patent materials for $200. I have two separate complete (and curent) crash courses including audio lectures and software. Please contact me at apbowling@gmail.com if you are interested. Thanks!

Reply

142 Reggae FeverNo Gravatar August 12, 2010 at 5:39 pm

Hi,
I just got back from the exam and received a “Preliminary Pass”. I hope they’re not pulling my leg :) I studied full time for 2 1/2 months using PLI and spent the last 2 weeks on this website. All the tips on this site was extremely helpful, especially the “new questions” tab that I discovered 2 days ago. So, before I head out for the evening and drink until I pass out, I thought I contribute to this site. Thank you guys!

Questions from the “Exams Questions and Concepts” tab
————————————————————————

Q2) Titanium Baseball
Q3) Mirror (Inherent Function)
Q5) Amending the Abstract – PCT
Q8) Missing Parts – PCT
Q13) International Search Reports – PCT
Q14) Costa Rica and Sweeden – PCT
Q17) Velcro (Trademarks in Claims)
Q26) Maintenance Fee ? How to correct a mistake that was applied to the wrong patent?
Q27) Reissue (variant of Tommie and Jo)
Q29) Death of inventor before application is filed
Q37) Terminal Disclaimer
Q38) Piecemeal
Q41) Documents Requiring Signatures
Q42) Missing parts in PCT Application (30 days)
Q63) TP Submissions
Q67) Obviousness
Questions from Previous Exams :
—————————————-
Tommie and Jo – Variant?
Bond 60%C or 60%D (4.00.27a)
Hair Gel (4.02.37p)
Parking Meter (10.03.28p)
3rd Party Submission – Japanese publication (10.03.10p)
Moondust (10.03.6p)
Five Steps to Cross a Road (10.03.23a)
Door Handle (10.03.24a)
Tribell (10.03.16a)
Bloc; Synthetic Z (10.03.7a)
147 Claim Counting (10.03.50a/4.03.28p)
** I’m sure there were more, just make sure you review ALL of the previous exams..

Comments
————–
-The PM section was much harder than the AM portion. Halfway through the PM section I started to feel discouraged and started doubting myself. I just took a deep breath and fought thru til the end !
-As noted from other posts, my exam was also heavily tested on Appeals & PCT
-Received an Ex Parte / Inter Parte Rexam question that I’ve never seen before
-Had an ownership question that went something like this.. T & J assigned 40% of the rights to XYZ company and 40% to ABC company. Who should sign the request for a Reissue (or was it RE-EXAM?)
-Ran into a lot of questions that I didn’t really know, I had to hussle and use the full time to look up all the the remaining questions in the MPEP

Reply

143 TommyBoyNo Gravatar August 23, 2010 at 7:49 pm

Congrats Reggae!

It sounds like we had similar tests, but looks like you got a few more repeats than I had.
——————————
Comments
————–
-The PM section was much harder than the AM portion. Halfway through the PM section I started to feel discouraged and started doubting myself. I just took a deep breath and fought thru til the end !
-As noted from other posts, my exam was also heavily tested on Appeals & PCT
-Received an Ex Parte / Inter Parte Rexam question that I’ve never seen before
-Had an ownership question that went something like this.. T & J assigned 40% of the rights to XYZ company and 40% to ABC company. Who should sign the request for a Reissue (or was it RE-EXAM?)
-Ran into a lot of questions that I didn’t really know, I had to hussle and use the full time to look up all the the remaining questions in the MPEP
————————————–
My a.m. section was WAAAY harder than my p.m. section. I had the same T&J assignment question. What did you put for the answer. I think I said T&J, and ABC and XYZ all had to sign. Not sure…still aren’t sure.

I also had a question about Ex-Parte Re-exam and asking for a re-exam within 6 years of the expiration of the patent.

Reply

144 StephenNo Gravatar August 12, 2010 at 6:04 pm

Congrats Reggae! I’m taking my exam the 16th, hopefully I have the same success.

Reply

145 TexIPNo Gravatar August 13, 2010 at 4:28 pm

Anyone who knows time span of sending copy of Patent Agent Certificate after passing USPTO Patent Bar? I have my registration number already from USPTO website; however, I have yet to receive the copy of my registration number.

Thanks! Good luck to future examinees!

Reply

146 AtlantaNo Gravatar August 15, 2010 at 3:37 pm

Just passed today. Thought i would give some feedback since i used this site so much.

Studied for 3 weeks. Did the 2002, 2003 practice tests once. Felt very uncertain about passing.

The morning session was ridiculously difficult. Lots of topics i have little experience with. Lots of long winded questions. I ended up making educated guesses for the last 7 or 8 since i was running low on time. Second half was much easier and had a lot of repeats.

The adobe at the testing service (Atlanta) sucks dick. It was slow, hard to navigate and the search function kept searching from the top of the section. Very frustrating.

Looking back, i would have definitely studied more if i had time. I decided kind of last minute to use the patent exam to help build my grad school profile, so did not have much time to get the exam done. You should definitely go over the practice exams multiple times. In addition, the tested concepts were all over the place.

Good luck to you guys!

Reply

147 AaronNo Gravatar September 14, 2010 at 3:56 pm

I also found the morning session to be difficult. During lunch I thought I pretty much have to get every question in the afternoon right in order to pass. To my surprise I passed.

Reply

148 Dave GNo Gravatar August 22, 2010 at 12:57 pm

Last I took the bar, a year ago, I got a 68%. I was encouraged because I knew I hadn’t studied enough. I took it again last Firday (Aug 20.) This time I memorized all the questions on the 2003 and 2002 exams. I learned so much more than I knew for the last exam.

But after an hour on the Aug 20 exam, I knew I was in trouble. There were no repeat questions, and few questions that required analysis. Most required me to look up some minute bureaucratic detail. The second half had some repeat questions, but not many. I got a 64%.

What do I do? Should I memorize the questions on this page? Some of them were on the test, so I suppose that wolud help. What seems most important is the ability to search thru the big MPEP for the answers, using a “one page at a time” PDF browser with a primitive search function. But nobody teaches that skill, and I can’t develop it on my own because I don’t have any more practice tests to work with.

Did I just get a bad exam? Are the pass rates published? Is the USPTO cracking down because there are too many registrations?

Any suggestions would be helpful.

One guy said you have to devote a month of your life to this project. Perhaps I tried a shortcut.

Reply

149 TommyBoyNo Gravatar August 23, 2010 at 7:41 pm

Dave G…I took the test on August 17th and also finished with a 68%. I was bummed at first, but then thought to myself that I have gone from relative newbie to getting a 68% on one of the toughest exams out there. I was pretty encouraged. I was further bummed to read through this website and see about 4 or 5 questions that were ‘repeats’ on my exam that I didn’t know about. I think I was 1 or 2 questions away from passing and the exam questions from this site could have helped push me over the top.

I agree that the exam was very difficult and required a LOT of searching in the MPEP. For me, the morning section was almost impossible. I had only seen about 2 or 3 of the questions before anywhere and the questions that were on the morning section were of a calibre that I had never seen before. On a handful of the questions I wasn’t exactly sure what the question was asking…at all?! I am hoping those were the Beta questions because they were pretty much unfair and entirely irrelevant.

I think that expecting to put in one month’s worth of studying and expecting to pass is pretty foolish, unless you have a strong background in the Patent industry. As for me, being a relative new-comer to this endeavor I think 3-4 months worth of studying is more reasonable.

Reply

150 BillNo Gravatar September 2, 2010 at 8:54 am

Dave, I feel your pain. I ended up with a 67 this time around. I was totally shocked when I saw I didn’t pass. I got a 65 last time and didn’t know half as much as I knew this time.

I had many questions about tiny ridiculous details. Ironically enough I thought I found the exact sections that the test was referring to because it had the same phrase as the question/answer but evidently I was wrong.

Goodluck brother, hopefully 3rd time is the charm for both of us. I’m going to try to take it again as soon as possible.

Reply

151 HCNo Gravatar August 29, 2010 at 1:59 pm

How big is the monitor?

Reply

152 DanNo Gravatar October 17, 2010 at 4:23 pm

Mine was about 17 – 19 inches, flat panel LCD. Optical mouse with roller ball.

CTRL + F doesn’t work, you have to click “Find” in reader.

Reply

153 Patent barNo Gravatar September 6, 2010 at 1:06 am

I have graduated from foreign university and I have only one original copy of the transcript. If I submit the original transcript to apply for patent bar exam, do they return back the transcript ? Any suggeastions ??

Reply

154 CBDADNo Gravatar September 7, 2010 at 10:42 pm

Yes, I submitted my original transcripts, with return envelop + postage, with my application. After reviewing and granting my application, USPTO returned my transcripts.

Reply

155 herewegoNo Gravatar September 14, 2010 at 1:52 pm

Taking the exam in a little less than a month, question for anyone who has taken the exam about the ‘find’ function – apologies if this has been answered elsewhere, I wasn’t able to find it after a quick search.

Can you search for words near each other?

and also can you search multiple words together in quotes or something?

any other search functions we should know?? THANKS

Reply

156 AaronNo Gravatar September 14, 2010 at 3:54 pm

Nope. Only can search exact words/phrases.

Reply

157 AaronNo Gravatar September 14, 2010 at 3:53 pm

Took the exam yesterday and passed. Do you ever find out your actual results? Lot’s of PCT on my exam.

Reply

158 RichNo Gravatar September 19, 2010 at 8:39 pm

Passed the test yesterday on my first try! There were a ton of PCT and appeals questions, and not as may repeats as I thought there would be (maybe only around 20 or so). I also found the morning session to be much more difficult. I was asked a variation of the same PCT question about 4-5 times; it involved what happens when an applicant files an international application in a non-competent receiving office (i.e., an office where they do not have citizenship). The correct answer is the application still gets an international filing date as of the date received in the non-competent office, and the application gets forwarded to the IB for further processing.

As for more general advice, I spent about 150 hours studying over the span of 2 months (I used PATBAR, which I think has a GREAT system, and is about middle-of the-road in terms of cost). I had no prior patent experience at all (although I am a second-year law student, which may have helped minimally). I think anything over 150 hours might be a little bit of overkill, provided you are studying properly.

I spent about 75 hours going over all the MPEP topics once, and then going over practice exams the rest of the time. Focus your studying on 600, 700 and 2100 and pay added attention to appeals, PCT and reissue/reexamination. Memorize 35 USC 102 and know how to apply it backwards and forwards.

I was able to break down the questions into three categories: repeats, questions you can find easily in the MPEP, and questions that are very difficult to look up and almost impossible to answer. If you know the 01-03 exams down cold, that’s 20-30 questions right there that you can put in your back pocket. As for the remaining 70-80 questions, I’d estimate there are only 10-15 that I’d put in the impossible/very difficult category. Remember that all the questions are worth the same, so don’t bog yourself down and spend too much time on the impossible ones! Make your best guess and move on.

Practice exams should be taken in 50 question blocks. You should aim to try to answer all the questions without looking in the MPEP in about 1.5 hours. This will give you 1.5 hours to look up the questions that you were unsure of/weren’t repeats. You should be ready to take the test when you are scoring 80-85% on the practice exams under timed conditions.

Most of all, when you are taking the exam have confidence in yourself! Remember that around 1/3 of the questions are repeats and tell yourself that you are sure those answers are correct. You should then be able to look up most of the remaining answers easily. Don’t sweat the 5-10 questions that seem impossible to find or answer, and confidently knock out the 80% of the test that are either repeats or easily found in the MPEP. You should then have some extra time at the end to scour the MPEP for the more difficult ones. Use all 3 hours you are given for each section!

My only advice for the test center experience is to not drink alot of fluids if you are like me and have a small bladder. You are allowed to use the bathroom but it takes a fair amount of time to leave and check back into the room since Prometric uses a fingerprint scan ID. Having to go will likely cost you 5-10 valuable minutes.

Good luck, and remember, never give up!

Reply

159 EriNo Gravatar September 20, 2010 at 1:48 pm

I’m an alien with US residency. I got biology degree in my country and now I’m working in biological field in US. I’m interested in taking patent bar of USPTO, but wondering whether I’m eligible to take the exam. I have a transcript of degree from a foregin university, but is there any other requirements?
Thanks so much, Eri

Reply

160 ChrisBNo Gravatar October 14, 2010 at 7:18 pm

JUST PASSED….

got home….this is my report…(ll make a longer one later).

The exam was easy going. About 30 minute in..i knew i was going to pass. Why?

Because THIS SITE. No kidding.

got the guarenteed formula for passing.

1. Know 2000-2003 tests in and out. Know the right answers..but also know why the wrong answers are wrong.
2. Memorize everything on this site.
a. Go though “Exam Questions & Concepts”..and create an outline for all the probs you only sorta understand.
b. Then go through this sites Forums and get all the reported problems that commenters leave under “Prometric Patent Bar” (there were like 6 problems from the forums i saw on their today). and also, if you have time go through the forum comments of “Exam Questions & Concepts”…
i. Be creating Rules of Law for all the ones you dont get…like “Rule: you can’t do this before you do so and so”…simple sentences.
Thaaaaats it.

As for general info..I saw about 10-15 repeats..not too many. But i was blown away how many answres I already knew from this site. . .

I got done with each section in about two hours, and had ample time to go back and double check and change ones i missed the first time.

USE Process of Elimination…write down A, B, C, D, E vertically and cross them off or put a question mark by the ones your not sure. .. I didnt do it for all the questions…about 20 per test…..you start getting fuzzy from exhaustion about halfway through each section. …this will keep you on track.

anyways….ill coment more on stuff later…but yeah..everything here is ON THERE.

Reply

161 JamesNo Gravatar November 2, 2010 at 5:14 pm

Recently passed the bar exam. Here’s what I can tell you:

– Inventors are reportedly dying left and right these days. Note to would be inventors: Don’t file for a patent. You’ll probably die.

– Apparently the BPAI has taken to issuing all kinds of new grounds of rejections. As far as I can tell, it must now be their primary function.

– People do not learn from their mistakes! That’s right, the same cast of characters keep waltzing in with a nearly, but not quite, identical problems. Hey, Bloc, didn’t you learn your lesson last time? Are you retarded or something?

– If the term “US/RO” looks funny to you, it won’t after you take the exam.

Reply

162 DamienNo Gravatar November 21, 2010 at 7:59 pm

I will follow up with a post in more detail later but wanted to share initial thoughts about exam. I just took and passed the exam today and want to stress the importance of repeat questions. I estimate my exam had 15-20 word-for-word repeats from old tests. My strategy for studying was the following: made a somewhat detailed outline of the MPEP (emphasis on important sections – 700, 2100, 1800 etc) and hammered a ton of review questions, i only did the past exam questions from this website that had indicated they were repeats. I knew those questions cold, i could answer based on the answer choices. I also went over the section on with the repeat questions not on past exams (lipgloss problem, inherent function mirror problem, spanish phone [all 3 on exam]) and drew up a strategy if i had these questions.

Test is pass-able, study this website, know past exams and be able to identify which section in the MPEP the question is asking without having to search the index over and over.

please feel free to email me with any questions or if you want to make a donation and receive my outline :)

damienjogensen@gmail.com

Reply

163 damienNo Gravatar November 22, 2010 at 1:06 pm

I got a test question involving the filing of a provisional application where soon after the applicant realized they left out figure 3 of the app. The question asks what is the best method to ensure figure 3 is included and the application is complete (something of that nature).

one answer choice was file another provisional app with figure 3 include (my selection). Other choices involved various amendments and whatnot….

Reply

164 fengyuwuzuNo Gravatar February 14, 2011 at 12:41 pm

if I failed in the exam the first time, for how long shall I wait to take the second time?

Thanks

Reply

165 LMNo Gravatar February 14, 2011 at 1:31 pm

60 days – it says on the exam result.

166 patenttipsNo Gravatar March 23, 2011 at 6:43 pm

You only have to wait 30 days, re-apply right away to get your new exam window.

167 yingNo Gravatar March 14, 2011 at 6:23 pm

I failed too. I am concerned about the scope of the exam will change effective from April 2011. Anyone heard about that and will the questions change a lot?

Reply

168 patenttipsNo Gravatar March 23, 2011 at 6:45 pm

Yes questions will change. Contact PTO for list of changes.

Reply

169 patentworldNo Gravatar March 14, 2011 at 9:00 pm

LM- No, you have to wait 30 days actually.

Reply

170 TommyBoyNo Gravatar March 17, 2011 at 5:56 pm

I just a notice that the entire exam will change.

Effective on or about April 12, 2011, the computer-delivered registration examination will be updated to utilize the following reference materials:

1. Manual of Patent Examining Procedure (MPEP), Edition 8, Revision 8 (available at http://www.uspto.gov/web/offices/pac/mpep/mpep.htm);

2. Examination Guidelines Update: Developments in the Obviousness Inquiry After KSR v. Teleflex (available at http://edocket.access.gpo.gov/2010/pdf/2010-21646.pdf);

3. New Interim Patent Subject Matter Eligibility Examination Instructions, August 2009 (available at http://www.uspto.gov/web/offices/pac/dapp/opla/2009-08-25_interim_101_instructions.pdf);

4. Interim Guidance for Determining Subject Matter Eligibility for Process Claims in View of Bilski v. Kappos (available at http://edocket.access.gpo.gov/2010/pdf/2010-18424.pdf);

5. Interim Guidance for Determining Subject Matter Eligibility for Process Claims in View of Bilski v. Kappos, July 2010 (available at http://www.uspto.gov/patents/law/exam/bilski_guidance_27jul2010.pdf); and

6. Supplementary Examination Guidelines for Determining Compliance with 35 U.S.C. § 112 and for Treatment of Related Issues in Patent Applications (available at http://edocket.access.gpo.gov/2011/pdf/2011-2841.pdf)

These reference materials will be available on the computer monitor during the examination.

There will be a blackout period prior to the initial administration date for the updated registration. No examinations will be administered during the blackout period, which is expected to begin on or about April 5, 2011.

Source: http://www.uspto.gov/ip/boards/oed/exam/exam_reg_prac_update.jsp

Reply

171 misspatentNo Gravatar March 27, 2011 at 9:46 pm

Is it just me or do the PLI materials truly suck? Poorly organized and formatted, and so many typos. Anyone happy with their non-PLI system?

Reply

172 StudyingTooMuchNo Gravatar March 29, 2011 at 5:41 pm

I have been using the Longacre book ($20 on Amazon), this website and the old exams posted on the USPTO website and think the combination is pretty extensive. I take the test on Thursday, so I will find out if this method actually works.

Reply

173 StudyingTooMuchNo Gravatar March 29, 2011 at 5:42 pm

* Comprehensive, not extensive. My brain hurts from studying.

Reply

174 SteveNo Gravatar March 31, 2011 at 9:48 pm

Can someone who’s taken the Prometric exam tell me exactly how the time is measured? Specifically, PLI’s Patware has a clock that measures time spent on the current question. Does Prometric have a similar clock for measuring time on the current problem?

Reply

175 patentworldNo Gravatar March 31, 2011 at 11:16 pm

Steve: there is a stopwatch-ish clock in the upper corner that counts down minutes:seconds. When you get done with the instructions and indicate you are ready to start, it starts with 3:00:00, and proceeds until 0. PS: CANNOT take a watch in.

Reply

176 SteveNo Gravatar April 1, 2011 at 5:30 pm

Thanks for the info, but damn, thats disappointing as I’ve relied at least in part on knowing exactly how much time i’ve spent of my current question. I should tell PLI not to mislead people like that.

Reply

177 patentworldNo Gravatar April 1, 2011 at 6:57 pm

well all you really have to do is keep an eye on it, and know that counting downward, you should be at certain points at certain times (ex. on question 24 at 1 hour and 30 minutes, in the least, for example).

Reply

178 HNo Gravatar April 2, 2011 at 10:14 am

On the Thomson Prometric website there is a tutorial of their testing software. The tutorial allows you to click on the words of answer choices to strike through them. But nobody in this forum seems to mention this for the test. Does the actual test let you do this? And, if so, do the strike-outs stay there even when you leave a question and later come back to it?

Thanks.

Reply

179 patentworldNo Gravatar April 2, 2011 at 12:14 pm

H- No it does not allow strikeouts. Basically, you can mark an answer, and then mark it “MARK” so that you can go back and check all of your “marked” ones. If you do not UNMARK it doesn’t matter, it goes with what answer you have provided.

Reply

180 JustPassedNo Gravatar April 4, 2011 at 10:53 pm

Hey everyone,

Just passed the exam and thought that I would share some thoughts on strategy. This worked well for me, and it might work well for you:
1. Create an answer sheet during the tutorial time (as others have suggested).
2. Answer all questions WITHOUT looking any up– think them through, but don’t look them up.
2a. If you are confident you got the answer right: just write in the letter on your sheet and don’t mark it.
2b. If you are pretty sure, but not positive you got the answer right: put one slash (e.g., ” – “) next to the question number and mark it in the software.
2c. If you don’t think you got it right, or if you know it is one choice or another: put two slashes (e.g., ” = “) next to the question number and mark it.
2d. If you have no idea what the answer is, put three slashes next to the question number and mark it.
3. After you have gone through and answered ALL of the questions once through, look at your answer sheet and revisit the questions you marked in the following order: three slashes first, then two slashes, then one slash.
Again, some may disagree, but this method worked for me and has a few advantages: 1) In the event you run out of time later on, at the very least you have already gone through and answered every question with your best guess. 2) By prioritizing your look-ups based on how little confidence you had in your answer (i.e., starting with 3 slash questions and working to 1 slash), you address the questions which need your attention most, and if you run out of time, the ones you didn’t get to look up will be the questions and answers which you were more confident about. 3) (closely related to reason 2) by answering all of the questions first without looking them up and then prioritizing and going back to look up, you prevent yourself from wasting valuable time double checking answers that you probably don’t have to… and if you do have time to go back and double check the one slash questions, then great…

Hope this helps.

Good luck to everyone.

Reply

181 JustPassedNo Gravatar April 4, 2011 at 10:58 pm

For clarity I should add to the above that when I say answer all of the questions first without looking them up, I mean actually enter the answer into the software.

Reply

182 JustPassedNo Gravatar April 4, 2011 at 10:59 pm

… And by “mark it,” I mean mark it for review in the software.

Reply

183 JustPassedNo Gravatar April 4, 2011 at 11:20 pm

Sorry for the piecemeal post, but a few other things: I shot for about 1 hour to one hour 15 minutes (during practice tests) to answer all questions without looking them up. One the real test, I did it in about 1 hour 30 minutes.

An additional advantage to this method is that when you go back to start looking up answers, your flow isn’t interrupted by reading new questions–in other words, you are working uninterrupted in looking up answers and I found I was more efficient when I worked this way.

Promise this is the last post! Best of luck.

Reply

184 Done and DoneNo Gravatar April 5, 2011 at 7:14 am

Here is another advantage to JustPassed’s approach. If you blow through the questions, when you go back to do lookups, you can hit all the questions in a particular area at the same time. This saves a lot of time fumbling through the MPEP in two ways. First, you will pass by things as you navigate that are questions in your lookup list. You can knock them out when you see them. Second, if you have a flood of questions in a particular area (for me it was Board’s New Rejection questions), you can hit them at the same time. This wasn’t my planned approach, but once I got into the test, it became obvious.

Reply

185 New ApplicantNo Gravatar April 12, 2011 at 8:02 pm

Did anyone take exam today? Please keep us posted. I am planning to take on the memorial day break….

Reply

186 Big Bad Voodoo DaddyNo Gravatar April 14, 2011 at 8:13 am

Following the earlier threads about the new exam. If someone can point out what the KRS, bilski changes were and if the number of repeats dropped, or more emphasis was on a a few select topics – it would be extremely helpful for us future test takers. Planning to take mine in June.

Reply

187 AlexiaNo Gravatar April 19, 2011 at 6:10 pm

Does anyone know if on the actual exam you can search a term in the entire appendix R using the find function or do you have to go into each section and do it that way? Thanks!

Reply

188 SarahNo Gravatar April 20, 2011 at 5:23 pm

Hey everyone!

I’ll be taking the test in a month or so, and this site is amazing. I just wanted to ask everyone that if you see a question in the comments from 2009 that was never answered, please answer it, even though its 2 years later! The new people that look at this site (like me) are reading through all these comments and a lot of good questions were never answered. So if you’re reading along and know the answer, do us all a favor and post a reply.

Also, a lot of people referenced an outline that was removed from the website. Does anyone know why? I’ve started making my own, but its going to be LONG, doesn’t really feel like an outline.

Reply

189 DanNo Gravatar April 27, 2011 at 5:14 pm

Yes, very interested in an outline!

Reply

190 Sarah W.No Gravatar May 17, 2011 at 4:25 pm

Another tip I’ve found helpful when taking my practice exams:

I noticed initially I would get questions wrong because I read the call of the question wrong. I knew the concept, but just answered in the opposite. Now on my written answer choice sheet, when I notice the call of the question is something like “FALSE according to MPEP” or “NOT true” or “all of the following EXCEPT”, etc, I write the word CALL next to the question before I even look at the answer choices. That puts it into my head to look for the right answer.

Then, at the end of the test, when you have extra time to check over the questions, I look at these questions I marked as CALL to ensure that the answer I put down fits the call of the question. I seriously raised my practice exam score by 10% just because doing this forced me to slow down and recheck any tricky questions.

Reply

191 BeckerNo Gravatar May 17, 2011 at 4:27 pm

Alexia, you can search the entire Apendix R.

Reply

192 SarahNo Gravatar May 18, 2011 at 3:17 pm

Just received a prelim pass report. A few things about the test and center:

-Not allowed to access electronic devices.
-Not allowed to leave the testing center outer office except to go to the bathroom.
-Not allowed to go to your car, and you have to leave study materials in your car.

That made my hour break VERY boring and I ended up only using 20 minutes of it. I REGRET it because I started to fade about halfway through the afternoon session, and had to take a break during my timed session to drink an energy drink in my locker.

Reply

193 DonNo Gravatar May 18, 2011 at 8:13 pm

I am about to take the test for the second time and reading comments concerning the testing center (Sarah, JustPassed), my experience was different.

Concerning Sarah’s post, I was able to access everything in my locker on break and leave the building for lunch. I didn’t look at my notes during break, but I presume I could have if I wanted to.

As far as making a list of the question numbers during the tutorial, I began to do that but was stopped by the supervisor and told I couldn’t write anything down before beginning the test clock.

Just a heads up that apparently some testing centers will be different from others.

Reply

194 JuneNo Gravatar June 6, 2011 at 1:05 pm

Hi, everyone.
I passed the patent bar early May.
Unfortunately I paid too much concentration, I do not remember much of the exam.
If you would like to buy used test material for bargained price, let me know.
Patbar material. for 150.

Reply

195 RBNo Gravatar July 13, 2011 at 1:40 pm

Does this material include the April changes?

Reply

196 AnnaNo Gravatar June 7, 2011 at 4:03 pm

My experience of the test, which I think is important to pass on is:
1. Although I was scheduled to begin at 8 am, the exam actually started earlier. I checked in at the front desk, put all my stuff in a locker and waited. The security guard called me to sign in again around 7:35 am, took my picture, verified my driver’s license, had me sign in, checked my pockets and then took me to my assigned “booth” and let me take the tutorial. AS SOON AS THE TUTORIAL IS COMPLETED, THE PROGRAM BEGINS THE EXAM. I was shocked and surprised and worried, but the guard came back and said it was ok to begin the exam. I was a little unprepared mentally because I thought I had another 20 minutes until the exam began (at 8 am), but I took the advice of others and wrote down all the chapter titles, and doing that settled me down, and helped me focus and get into it.
2. RE: Lunch break. Yes, it is for one hour, but know that the hour is also counted down in the program. The program does not pause for your break and then restart. When you begin the test, it counts down the first three hours in the upper right hand corner. Then it closes the screen and notifies you that you time is up, AND THEN IT COUNTS DOWN 60 MINUTES. When those 60 minutes are complete, the program opens up the second half and starts counting down another 3 hours, regardless of whether or not you are in your seat. I was allowed to leave and eat. I came back, signed in, showed my empty pockets, waited to be escorted to my seat, sat down and looked at the screen with just 1 minute and 23 seconds before the end of the countdown of the break. Whew!
3. My facility would not let me bring in my analog watch. I had to leave it in the locker. I only had my locker key and my driver’s license with me. They gave me pencil and paper.
4. The facility performs a lot of other exams. I was the only one there that day doing the patent exam that I know of, most of everyone else was doing some kind of medical certification exam.
5. At the end of the exam, there was a short survey about the facility, and then it gave me my score. I waited to be escorted out, and then they also print out the results, stamp it, and give it to you.

Reply

197 JanelleNo Gravatar June 13, 2011 at 3:04 pm

I passed the Patent Bar on my first try this past weekend!

Going into the test I had all intentions of remembering difficult questions so that I could report back to this website to help, however once the test got going I couldn’t remember anything. However, I can say that the test was very difficult. I had fewer than 10 repeat questions. I had several KSR, Bilski, 112 new questions. Here’s my Patent Bar study/exam experience breakdown:

My Study Technique:
I took the PLI home study course. I studied 12 hours a day for about 4 weeks. I took the first two weeks to get through the PLI lectures and quizes. From my understanding of the PLI Course, the course only requires the student to listen to the lectures and answer the questions. I not only listened to the lectures but I took extremely detailed notes, made flashcards, and took daily quizzes by chapter on the Patware program. I believe this additional studying really helped. The last two weeks I took exams. I cannot stress how important it is to take the exams. I took the exams over and over and over again until I was getting 100%. It is not just that I had the questions memorized but I studied the questions until I understood the reasoning behind the wrong and right answers. I do believe the PLI course prepared me quite well, however just doing the practice test repeatedly is what allowed me to pass. My first practice test after finishing the PLI lectures I made 78%. If that had been the real exam, I would have passed. So, kudos to PLI.

Testing Center:
I had a great testing center. The person running the testing center was very organized and attentive. I was the only person taking the patent bar that day. Everyone else was taking the USMLE Step 1 exam. I was given scratch paper and two pencils for the exam.
My Test Strategy:
During the tutorial I made a chart with the letters ABCDE across the top and numbered 1-50 down the page. Also, I marked at questions 20 and 40 how much time I should have left at those questions. This kept me on track so that I did not spend too much time on a question and run out of time. Another thing I did was I put a check mark next to questions that I knew I got correct. I knew which ones I had correct because the question was a repeat question or I found the material in the MPEP (or of course, I just knew). This helped my nerves because there were so many questions that I just did not feel comfortable with. By the end of the test I counted about 40 questions I knew I had gotten correct. I handled each question as it came. I know some people go through the test answering the questions they know first and then go back to the questions they didn’t know. This strategy did not work for me. If I had used this strategy, there would have been a ton of questions I would have needed to come back to.

The Exam:
I don’t mean to scare people, but I have to be honest. In my opinion, this test was a lot harder than the old exams. Perhaps it is just because I studied the old exams so much that I thought they were easy, but I definitely feel that the questions required a higher level of understanding/analysis.

Questions:
I had 7-8 PCT questions. No claim counting questions. 5-7 KSR, Bilski,112 questions. 5-7 Appeal questions. Maybe 10 repeat questions. None of the questions had answers that were verbatim in the MPEP. For several questions I found myself reading a couple pages to find the correct answer. I had several tiered questions that had options I,II,III, IV,and V and then a combination of those for answers ABCDE.

Despite how difficult the exam was, it was still passable. So, for those of you still studying, be strong and continue your efforts. Best of luck!

Reply

198 RBNo Gravatar July 13, 2011 at 1:42 pm

Do you have your PLI material? Does it include the new April changes? Are you willing to sell?

Thanks!

Reply

199 BigBadVoodoDaddyNo Gravatar July 13, 2011 at 6:38 pm

Took the test and passed on the first try this week.
Thank you to all the people on this website that have posted.
My advice read the recent questions. I am posting on the other sections of this website.I used PLI (selling on eBay currently), USPTO tests from 1999-2003 and this website. On the MPEP, I read chapters 700, 2100 in-depth and skimmed through 600, 1800, 1200 and 200.
The prometric site is very professional, quiet and clean.
No problems checking. I set my time to start at 2 PM – that was my preference. But I showed up at 1 PM and they were ready to seat me. That was great.
I had spent time in the days before testing the prometric software on the website, so I didn’t waste any time looking at in during the tutorial. Its quite straightforward really.
The only thing I didn’t like about the software was that after you were done, if you were reviewing (either marked or unanswered) questions, you could not go back to the previous question. This option shows up only when you choose the Review All question.
The clock doesn’t stop once you finish the morning session. A fresh 60 min timer begins. I didn’t take a long break, just ate a sandwich and drank some water, stretched my legs and used the restroom and was good to go in 40 mins. I suggest staying away from caffeine and other energy drinks, except for the tea or coffee that you might normally drink in the morning. Because, the when I started the test, my heart was beating fast, and you are intensely focused on the test – you really don’t need to stay away. Believe me you will feel jittery enough without any help from caffeine.
So I spent the first 15 minutes (during the tutorial) making my answer grid – with choices ABCDE. I wrote down the grid for both the morning and afternoon. Alternatively, I realized that if you took a shorter lunch break – you could use the spare time then to make your grid. In addition I had practiced with the following method to form a columnar grid
Q.No A B C D E T/F CHPT NOTES
I would make a square around any unanswered/for review question (and strike through that once I was done) and note the call of the question (where applicable) in the T/F column. The chapter column helped me answer all the questions from that chapter together, so I could look it up.
My method was to make 3 passes. In the first pass – I did only the short questions and/or obvious repeats to which I knew the answers. If the choices were lengthy, or the question was filled with fact patterns, I would note the chapter and the call of the question plus and notes and move on. In the second pass, I would hit up all the unanswered questions, with all the questions from similar chapters all together. In the first and second pass I would mark the questions for review and do that in my third pass.
When I was practicing, this was really hard, but later on and especially on test day – by the third pass I knew I was pretty confident about all the answers – I would spend 1 hour on my first pass – that way it would give me a good idea about how the test was structured, 1-1.5 hours on the second pass, and any left over time to review the test.
Good luck guys – don’t forget that preparation is the key, read the questions on this website, do the practice tests and get your basics about the MPEP straight.

Reply

200 BoNo Gravatar July 21, 2011 at 9:47 am

I passed the exam last week, and just wanted to say many thanks indeed to this site and all the people who have posted on it. I don’t think I’ll be much help on what questions were asked- it was a serious blur, but here are some insights:
1- I studied using the Bullseye Patent Materials (which I highly recommend), The Ultimate PAtent Bar Exam Study Guide (a great introduction, but a little out of date, as it doesn’t concentrate on PCT) and ofcourse this site.
2- The Prometric center (San Jose) was OK. The staff were nice and my chair was comfortable :) Unfortunately, when people walked past, (which occured frequently, as there were a lot of different exams going on) the monitor and my chair would ‘jiggle’. I would say get to know Adobe 5 if you think the change will be an issue. It took me a minute to get used to the search feature.
3- I remember only a few repeat verbatim questions. There were a lot that were variants that needed some thought. Only a couple of questions on the new materials, and they were very easily searchable (verbatim in the materials). I don’t seem to remember too many PCT questions, but quite a few appeals.
4- I didn’t really have an answer marking strategy- I just marked all the ones I was unsure of, and came back at the end. On both papers, I had 45 mins for review. I guess do what makes sense to you, and practice in an exam situation a couple of times.
5- Looking back, I would say absolutely study the repeat questions, but know inside and out why the answers are what they are. It makes you go to the MPEP and get to know it. Really this test is about learning your way around the MPEP and it’s nuances.

Good luck to everyone!

Reply

201 Top GunNo Gravatar July 26, 2011 at 5:38 pm

Passed today. As other posters have mentioned, this site was a huge help. And as one poster mentioned, it’s important to pass on the goodwill.

Do the questions. Going through old exam questions ad nauseam makes the actual exam almost routine.

My suggestion for reading comments on old and repeat questions is to read the question at the top of the page, work through what you think the answer would be, then scroll down to the very bottom and start reading there. That is, read from the bottom up. While certainly not a foolproof method, it helped me to avoid reading comments that–while well-intentioned–might be considered misinformation.

I used materials from BARBRI Patent Bar, which almost no one on this site uses. BARBRI no longer offers the course or materials, but I had the materials already so I used them.

Good luck to all.

Reply

202 Jin GeNo Gravatar July 28, 2011 at 12:00 am

Passed today.
I just wanted to say lots of thanks indeed to all the people who have posted on it. This site gave me huge help.

Reply

203 BobNo Gravatar August 3, 2011 at 1:58 am

Took the test today and passed. Thanks to all the people who have posted here. This website is definitely a game changer. I saw many repeats from past exams and also from the “Exam Questions and Concepts” section on this website. In preparing for the exam, I used the Bullseye materials together with an old copy of the PLI course I bought on eBay. I had no previous patent or patent law experience. I went through the PLI course in a couple weeks, and then drilled questions daily for about a month. I would go through the Bullseye outline just to get a quick overview of the MPEP chapters and their contents. The only portions of the MPEP that I read were 1801, to get a grasp of the PCT process, and the 102(e) examples in 700 (706.02(f)(1)). When I drilled questions, in addition to trying to memorize repeats, I would also look up every answer, right and wrong, just to see where it was in the MPEP. I think the most important thing is the get familiar with the MPEP and where you should look to find answers because you will get a lot of questions that you won’t feel 100% confident with and will want to look them up. During the tutorial, I made an answer grid to keep track of my answers for both the AM and PM sessions. I marked on the grid at certain questions how much time should be left when I got to that question, just to keep me on pace. I just started with #1 and went sequentially through the test.

I had about 25-30 repeats or slight variants from previous exams and this website. A majority of the other questions tested the same material as the previous exams, so it is crucial to go through the old exams (2002-2003) at least 3 times. My test seemed to be all over the place. I only saw a few questions regarding the new material, a handful of PCT, and probably 8-10 appeal-related questions. If you study consistently and go through all the questions on this website (including the comments) and the past exams, the test will seem fairly easy. I had about 30-45 minutes at the end of each section to review.

Thanks again and good luck to future takers!

Reply

204 fengyuwuzuNo Gravatar October 6, 2011 at 12:07 pm

1. are you going to sit on the same computer when you come back after lunch?

2. will they new scratch paper and take away the morning ones?

I wonder if I make answer sheet in the morning, whether or not they will take it away in the PM session.

Reply

205 PaulNo Gravatar October 7, 2011 at 3:47 am

I took my exam on 10/3.

1. are you going to sit on the same computer when you come back after lunch?

Yes. Once the first section ends, the break timer begins, and you can just stand up and exit the room (to security). The timer can be ended early if you choose (I actually only took about 40 minutes for my break).

2. will they new scratch paper and take away the morning ones?

What I did was I took my scratch paper out to security after the first section, and then got new scratch paper when I checked in for my second section. Ultimately, you can’t take anything in or out of the test room (except ID and locker key). I wondered about whether I could have left my old scratch paper in the testing area after the first section. Unfortunately, someone else will probably have to answer the question of whether or not they require you to hand over your first book of scratch paper after you finish your first section.

206 BluebirdNo Gravatar August 9, 2011 at 4:28 pm

I took the exam yesterday and passed. Lots of repeats you can find on this site – very helpful! And you can easily search other questions. However, my new guidlines are not searchable but only a couple of questions are related to that. For exmaple, one statement said “Machine or transformation is the sole test…,” and another statement said “A claim of a mere business method is not patentable under 101.”

By the way, could you please tell me how long does it take to get the formal notice from USPTO? Thanks!

Reply

207 BluebirdNo Gravatar August 9, 2011 at 4:32 pm

I have another question about “limited recognition.” I am working on H-1B working visa so even I passed the exam, I will get “limited recognition” to “prosecute as attorney or agent a specified patent application or specified patent applications” per rule 11.9. How does this work? Can I sign papers now? How do I specify patent matters I am working? If any people have similar experiences, your input will be much appreciated.

Reply

208 RalphsNo Gravatar August 19, 2011 at 9:42 pm

I am not a permanent resident of US.
After taking exam in US, I have to move to another contry.
In this case, when I fail my exam, can I try the exam again?
So, is it possible to travel to US to take the exam?

Reply

209 HexheadNo Gravatar August 20, 2011 at 8:41 am

What a great website! Thanks to everyone who has posted info here. I took the test yesterday and passed!

One thing I was concerned about going into the exam was searching the MPEP as I never tried the practice exams with Acrobat reader 5 as many suggested. I was also concerned about the ability to switch back and fourth between the exam and reference materials. I found that neither were issues at all. I found that if I closed the MPEP window and then reopened it, it was right at the exact section I left it.

The first thing I did when I sat down to the 15 minute tutorial was to make an A-B-C-D-E x 1-50 matrix for both sessions which took me about 5 minutes. The tutorial took me about 3 minutes so no problems there. I would highly recommend using the matrix to document all your answers as well as eliminated answers as I was shocked to find at the end of the both sessions I had not filled in the answers to about three questions. It was easy enough to go back and fill in the answer from my matrix versus re-answering the questions.

Best of luck to future test takers!

Reply

210 NotHappyNo Gravatar September 11, 2011 at 5:00 pm

Finished the exam about an hour ago. I failed. Hard. With only a 50% score.

It seem I should have gone with this website’s recommendations. As I was an examiner I figured the material I had from the Patent Training Academy would be sufficient.

It could just be my imagination but it seemed like half the test was on Appeals.

Guess I’ll have to spend even more time and money on this exam.

Reply

211 maggieNo Gravatar October 11, 2011 at 7:21 pm

Weird that searching the Appendix R Patent Rules using numbers “i.e. 1.105 (2)(b)” is very ineffective. You must use words to search it well.

http://www.uspto.gov/web/offices/pac/mpep/index.htm

Reply

212 Mike CNo Gravatar October 14, 2011 at 10:47 pm

I passed the exam on 11/13/11. Even after being accustomed to taking electronic practice tests, the experience was far more grueling than I had anticipated. To make matters worse, I had something urgent come up the night before, which didn’t leave me with very much sleep. But somehow, it worked out and the feeling is just great.

I am very grateful for this site and to everyone that has posted their studying tips and comments regarding repeat questions. Without such a support base, I don’t think I could have passed on my first try. I feel obligated to give back in some way, so I will talk about how I prepared hopefully offer some of my own tips. I’m sure that many things I say below have already been suggested above in this thread, but you can be sure that the following suggestions and strategies worked for at least one person.

As a broke grad student, I was not willing to fork over more than a month’s salary for the PLI course. Furthermore, I was not exactly convinced that I needed this course, even after all of the talk about the changes for the worse in the exam since April 2011. So I decided to put together my own poor man’s study course over the months of June through October. It consisted of the following materials:

1) Longacre Patent Guide to the MPEP (about $70 on Amazon). While the reviews are not impressive, I actually enjoyed this guide very much. I read it through twice, and then read the part about chapter 2100 over again right before the test. The author does an excellent job of pointing out the relevant sections of the MPEP that should be studied, and emphasizes the points that would likely be covered on the test. After my first read-through of this book, I read it again while simultaneously reading through the relevant parts of the MPEP (although sometimes I skimmed. I felt the most critical chapters to read are 700, 1400, 1800 and 2100). My only complaint is that Longacre’s guide is not entirely thorough when it comes to appeal. I had a ton of appeal questions on test day, for which I had to rely a lot on the electronic MPEP.

2) CATPrep software (http://www.catprep.com/exams/patent-bar/index.html). This is a very nice software package that electronically simulates the old tests. For $30, you get 9 of the most recent tests (1999-2003). I am very glad I used this software rather than solely working out questions on paper, since it gave me a “comfort edge” on test day.

3) Read a book on patent law. In particular, read one of Janice Mueller’s books on patent law. I read “Introduction to Patent Law” prior to doing any serious studying. After going through the Longacre guide the first time, I decided to re-read the book to pick up on things that didn’t sink in the first time. It’s a very well-written book, especially for someone like me who didn’t know the first thing about patent law when I started studying.

Other tips:
1) Know where everything is in the MPEP. By your second time through the study guide, you should be able to read a practice question and know (or have a good idea of) exactly which chapter it will be in the MPEP. For example, most questions that recite an examiner making a rejection should immediately suggest chapter 700.

2) Become a master searcher. Once you know where things are, you need to know exactly what to search for. Practice exam questions by not only memorizing the answers, but also locating strings of words or obscure words in each question and search for them in the relevant chapter. For example, if something sounds like legal jargon, chances are it is taken word-for-word from the MPEP. Also, anything that pertains to patentability will be lifted almost or exactly word-for-word from 2100. Being skillful at searching is absolutely critical, as you will need every minute on test day. The faster you can search and find, the more sure you can be of your answers and move on.

3) PRACTICE ALL THE TESTS! Take every test electronically. Go through and review your answers. Know why you got some wrong by reading the relevant parts of the MPEP. Make flash cards of the answers you got wrong.

4) Review and try to memorize the repeat questions on this site. You will be surprised at how many you see on test day. Once you see one that you memorized, you will have that much more time to work on other questions. Also, read the comment thread underneath each question, especially if there is controversy. This definitely helped me to make sense of things that were ambiguous.

5) And now for some psychology: You must fall in love with patent law. You must become patent law. Over the past few months, I have talked about nothing but patent law and now my friends and colleagues are probably sick of me. It has been said that the test is not about patent law, but about knowing the MPEP. I am inclined to disagree, because the MPEP is about how to apply patent law, and my love of patent law has compelled me to study the MPEP. You cannot fail something you love.

That sums up my suggestions. Best wishes to everyone taking this test. May the relief you will feel when you pass justify the suffering you endured to prepare.

Reply

213 maggieNo Gravatar October 14, 2011 at 11:17 pm

Thank you! And Congrats :)

Reply

214 KFNo Gravatar November 3, 2011 at 4:49 pm

KF November 3, 2011 at 4:33 pm
Just got home from the exam. I didn’t pass. Got 60%. I had hardly any repeat questions, maybe 4 that I can remember, Moondust, A PCT one, Foil airplane, and one other I’ll try to remember. I was hoping for a lot more like others had. Not so. The exam was much harder for me then the practice tests. Lots of random things i had never heard of like subcombination/combination. I had to look up almost everything. Lots of appeal and obviousness, handul on the new KSR and Bilksi stuff. I was really hoping to pass. Damn. I studied for hours a day since early August. Damn. Oh well, I’ll take it again! I am working full time and balancing life with studying so I guess it will just take me longer.

Reply

215 Lisa O.No Gravatar November 4, 2011 at 3:01 pm

I’m taking the test on 11/16 and took the advice of earlier posters to go the the Prometric test site (Fair Lawn, NJ) to avoid the panic in finding it on the day of the exam. Glad I did!! I spoke to the staff and was told that they do NOT allow any writing on the 4 scratch papers until the exam starts. No writing the grid A-E or 1-50 as suggested during the tutorial…which I found so helpful in doing the practice tests. I was told that “brain dumping” is not allowed.

Reply

216 KFNo Gravatar November 6, 2011 at 10:58 pm

Hi everyone,

Well, I took the weekend off, moped a bit, and am ready to ramp up my studying to pass for sure the second time. A question for those taking it for the second time-it seems you have to both apply to the USPTO a second time, as well as sign up for prometric. Do you have to wait until you recieved a second approval letter from the USPTO before you schedule your exam for the second time? I know you have to wait for the 30 days to go by, but I’m a little confused what I have to do. Any tips would be great.

Lisa O-I was able to write on my scratch paper during the tutorial, I just had to turn my scratch paper in. I made my answer grids for both the morning and afternoon on the same packet, and then had to redo my afternoon grid. I was either allowed to keep the packet of scratch paper I had in the morning or get a brand new one. So clearly, different testing centers have different protocol. So if I had wanted to brain dump and keep my morning session scratch paper I would have had those notes acessible to me the whole time.

Reply

217 Lisa O.No Gravatar November 7, 2011 at 9:18 pm

KF,
I think earlier posters indicated that you reapply to USPTO – transcripts are kept on file for one year. Are you planning on going to the USPTO to check on your answers?

Reply

218 RickyNo Gravatar November 7, 2011 at 6:24 pm

Hi Guys

How long will it take to receive the confirmation letter?

I mailed by application 3 weeks ago and my credit card was charged 2 weeks ago. Have not received anything from USPTO yet.

Reply

219 Lisa O.No Gravatar November 7, 2011 at 9:19 pm

Ricky – I think I received a response in 2 1/2 weeks…..

Reply

220 MaggieNo Gravatar November 7, 2011 at 11:42 pm

It took me much longer for me ~ 1 month after charging my CC.

221 RTNo Gravatar December 8, 2011 at 11:04 am

I think I got mine back in about 3 weeks. I included a stamped post-card and it seems like my answer from the USPTO was less than two weeks after I received the postcard.

Reply

222 SBNo Gravatar November 15, 2011 at 2:11 am

I passed the exam today. I relied a lot on my experience (I have been working in this area for almost 2 years). However, it is not everyday that I deal with reissue, re-exam and appeal. I also practiced 2003 and 2004 exam questions this past weekend. The process of reviewing each question/answer was very helpful and that’s how I became familiar with resissue/appeal. I did recognize many repeats … so practice with old questions really help. As for relying on the MPEP lookup, it was hard. I had to read a lot of paragraphs to find or confirm a statement. I actually finished a first pass in an hour and a half and then I spent another hour and a half looking for answers…. Honestly I can’t believe I passed! Good luck to everyone else preparing for the test.

Reply

223 SBNo Gravatar November 15, 2011 at 2:16 am

Oh BTW, the PLI patent bar was pretty much useless. I learned the most from the past question/answer review.

Also, can anyone tell me how long it takes to receive the official confirmation letter from the USPTO?

Thanks!

Reply

224 ChrisNo Gravatar November 15, 2011 at 1:43 pm

Yes SB please let me and Lisa know what topics were mostly tested on the exam! It would be a great help!

225 Lisa O.No Gravatar November 15, 2011 at 1:15 pm

Congrats!
Can you give me a sense of what percentage of the exam was on patentability? I’m taking the exam tomorrow – what should I study in my last few remaining hours? Thanks!

Reply

226 Yajaira SchellsNo Gravatar November 16, 2011 at 12:07 am

You have brought up a very wonderful details , thanks for the post. “There’s two heads to every coin.” by Jerry Coleman.

Reply

227 KerstinNo Gravatar November 16, 2011 at 12:15 am

Hi everyone,

I didn’t pass on my first try-came close with a 60. I am debating going to D.C. to review my exam. For anyone who did go do you think it was worth it? It would require me to take unpaid time off work and a $400 plane ticket plus hotel. :( Just trying to decide if it’s worth the expense when the cost is not really in our budget right now. I don’t want to get to the second try and not feel like I did everything I could to pass, you know? Thanks!

Reply

228 MattNo Gravatar November 17, 2011 at 9:09 am

I’m thinking of traveling to DC too. A round trip from Orlando is only about $200. The PTO is rigth next to Reagan National airport with a metro stop that is 5 minutes walk to the office. If I did that, I’d fly in and out on the same day to minimize the costs.

Reply

229 KerstinNo Gravatar November 16, 2011 at 12:15 am

SB-I got my official letter in about a week. Congrats!

Reply

230 maggieNo Gravatar November 16, 2011 at 8:31 pm

Just passed! Feels VERY Good. Will report back tomorrow.

Reply

231 KerstinNo Gravatar November 16, 2011 at 10:58 pm

Congrats Maggie!!! I know you worked so hard for this!

232 JakeNo Gravatar November 16, 2011 at 9:04 pm

Just passed today, definitely harder than I thought, there are about 20 repeated questions, and there are quite a few tricky questions.
I did not use too much time to prepare this test, basically only used the material from this website. Thanks for the creator!!

Remember a few questions:
Moon dust
PTC term
rejection by board under new ground
and quite a few questions about reissue assignment question

BTW, for me, the afternoon session is harder then the morning, lots of new questions in the afternoon session

Reply

233 SBNo Gravatar November 17, 2011 at 11:49 pm

Sorry for the delay in responding back. But I remember all the questions reported by Jake. Others were:

– inventor Bloc (credible utility)
– priority date (PCT)
– which submission will be rejected for presence or lack of signature
– filing date (when uspto receives or when you mail)
– national stage filing deadline
– KSR related question I can’t quite recall
– number of dependent claims (multiple dependency)
– when appellant does not respond to appeal decision within the given time, and when there are allowable claims and rejected claims, the patent will issue with the allowable independent claim and dependents if any
– 103 exception
– a question relating to special definition in the spec

Also, I remember looking quite a bit in 1400 section. Good luck!

Reply

234 SBNo Gravatar November 17, 2011 at 11:51 pm

One more:

102(e) reference date for an application that has a PCT filing and a national stage filing.

235 JakeNo Gravatar November 16, 2011 at 9:35 pm

one more thing, read the choice carefully, there are quite a few choices the sentence is put together in a very awkward way, negative on negative on negative, feel like you are doing last, i don’t think that is the right direction Patent bar should go.

Reply

236 MattNo Gravatar November 17, 2011 at 9:21 am

A lesson for the under-prepared:
I didn’t pass on the 16th of November. I went in knowing I wasn’t prepared enough but tried to keep an open mind (It was my last day to take the test – I had already rescheduled once). The test was alot harder than I expected. Many questions left me baffled. Others I spent entirely too much time searching the MPEP (mostly Ch 700, 1200 & 1800) for the answers. I frankly only spent less about 30 hours preparing. Other things in law school including my upper-level paper kept me distracted. When I signed up in August I thought I’d have more time to devote to studying, but ended up just trying to cram in the last 5 days and I didn’t invest in any prep materials. Worst of all, I didn’t discover this site (which is awesome – thank you to the site author) until about a week ago. I easily need another 40-60 hours of prep time for the next go-around.

Some repeats included moon dust, the foil airplane, abbot & laurel, and others I can’t exactly recall – but less repeats than I expected. Again, frankly I only really studied about about half of the repeats on this site, did only one old exam (I know – pathetic), and otherwise tried to read all of the important chapters. I can attest as many others have that it was very heavily weighted to PCT & Appeals. There was a trade secret question too.

Reply

237 ENo Gravatar November 23, 2011 at 3:48 pm

I passed yesterday. I am so thankful for this site and everyone who posted. It was my second attempt and this site really helped me this second time. My study strategy for the first test was mastering all the material. I spent so much time just reading the MPEP and not enough time doing practice questions. This second time I mostly focused on practice questions. I went through most of the repeat questions on this site, and I really think that is why I passed this time.

I had probably ten-fifteen verbatim repeat questions. The rest were variations of the repeat questions. I think I had a new one- it was about a means plus function claim for computers and algorithms. I picked answer B but it was just a guess. I will post about each repeat underneath the question itself.

I definitely recommend going through all the questions on this site, and getting a good night’s sleep before the test. I also had a “five hour energy” during my lunch break. That really helped me focus during the last hour!

Good luck to everyone!

Reply

238 RoboRobNo Gravatar December 2, 2011 at 8:50 pm

So I took the exam today only to fail. My score was a 68. If they round down, that means I missed by one question (90 scored questions, 62/90 = 68.8), if they round up, I missed by two questions 67.8%. My computer crashed during the exam. Prometric said I could contact them next week and report the crash. Has anyone else had their computer crash, only to miss by one or two questions? I missed about 5 minutes of time, although they couldn’t tell me if the timer stopped during the crash. Note to future test takers, don’t use the enter button on the search function, you need to click the button, otherwise it might crash…

Reply

239 Lisa O.No Gravatar December 4, 2011 at 12:01 pm

I just booked my visit to USPTO to review my exam on 12/16/11 (I had taken the exam on 11/16/11). I’ve been told that no copies can be made or notes taken. Has anyone gone to the USPTO to review their exam? Also, I just received the letter to schedule my 2nd exam and found out that on or about 1/31/12, the exam will cover 2 new rules issued on 9/26/11 in accordance with the Leahy-Smith America Invents Act. Also, the exam will include questions concerning the 11/22/11 rules governing practic in ex parte appeals. Again, this is on or about 1/31/12 (which is now motivating me to schedule to retake the exam sooner than later).

Reply

240 MalvaNo Gravatar December 5, 2011 at 2:20 pm

So I just scheduled my Exam for the 26th given that they will implement the updated rules on or about the 31st of Jan. and I have the old study prep materials. I have been doing practice questions for some time now but have not taken a full practice test yet. My question is, is about a month and a half sufficient time to prep for the Exam while working full time?? What are your thoughts??

Thanks,

Reply

241 Lisa O.No Gravatar December 5, 2011 at 9:41 pm

I just scheduled my exam for 12/27. Doing the full practice tests (especially 2002/2003) are so important as well as going over the Repeat Questions and posts on this site. Working full time and preparing for this exam is extremely tough – I’m struggling and will be taking a few days off from work the week before to study. I would suggest spending as much time as you can preparing.

Reply

242 MalvaNo Gravatar December 6, 2011 at 12:23 pm

Thanks Lisa,

I meant to say that i’m taking it on the 26th of Jan. I’m trying to avoid the new material that will be implemented in the test. Hopefully I will have sufficient time to prepare. Indeed working full time and preparing is really tough. I have been taking practice questions for a while now and they have been extremely helpful, however I still need a lot more prep time. I’m keeping my fingers crossed…

243 GregNo Gravatar December 6, 2011 at 7:47 pm

Malva – unless you have a photographic memory or have time to grind through about 5 runs through each of the old exams, then I would say preparing for 1.5 months is unrealistic. I have been working full time for 20 years and first starting preparing in Nov. 2010 and accomplished a successful effort on the exam this past Sunday, 12/04. It was my 2nd attempt; the first coming on May 30, 2011. However, getting in front of the new material is worth the investment of late night hours. BTW, I spend many hours from 9 – 12 each night grinding after my family went to bed. Most Saturday afternoons from Aug – Dec were also devoted to outlining or old exams.

Reply

244 RobNo Gravatar December 17, 2011 at 11:13 pm

Quick question. I take the exam Monday. Using PRG’s version of the MPEP and test, I found that I couldn’t search for more than one word at a time (no strings). Please tell me that isn’t the case on the actual MPEP provided for the test itself. Crazy!

Thanks,
Rob

Reply

245 Lisa O.No Gravatar December 18, 2011 at 6:24 pm

Rob,
I took the test last month…from what I remember (it was quite a blur!), I think I was able to search using one or two words…Good luck tomorrow!

Reply

246 RobNo Gravatar December 18, 2011 at 6:54 pm

Thanks Lisa. That is what I was afraid of. 1-2 words doesn’t let you search for a lot of different things. Bummer :(. Thanks for the input. Did you pass?

247 Lisa O.No Gravatar December 18, 2011 at 7:31 pm

Unfortunately, no. But, I immediately filled out the paperwork and am retaking it on 12/27.

248 RobNo Gravatar December 18, 2011 at 8:07 pm

Good for you. That means you’ll be able to take it again before they add AIA. From what I understand, they’re bringing the AIA stuff in shortly.

249 RobNo Gravatar December 18, 2011 at 3:17 pm

Here’s another question. Assuming that I can use string searches… Anyone have any suggestions on how to search effectively on this one:

You are a registered practitioner and you have filed a patent application in the PTO on behalf of your client, Wannaberich, on January 7, 1998. In the first Office action, the examiner made a restriction requirement. Although your client disagrees with the restriction, you have made a provisional election with traverse and vehemently argue the restriction requirement. In the next Office action, the restriction is made final and an action on the merits follows. The application is eventually allowed. The client now wants to pursue the non-elected invention. You file a divisional application directed to the non-elected invention before the parent application issues as a patent. In the first Office action in the divisional application, the examiner rejects the claims on the grounds of obviousness-type double patenting over the patent which issued from the parent application. What should be the most appropriate reply to the rejection?

A. File a terminal disclaimer to obviate the double patenting rejection.
B. Amend the claims in the pending application to overcome the rejection.
C. File a 37 CFR §1.132 antedating affidavit.
D. [Correct Answer] Request reconsideration and point out that it is improper to use the parent patent in an obviousness-type double patenting rejection when a restriction requirement has been made by the examiner in the parent application.
E. File a petition under 37 CFR §1.183 to the Commissioner.

The correct answer is (D). 35 USC §121; MPEP §804.01. (A), (B), and (C) are incorrect. The use of the patent as a reference against the divisional application is prohibited by 35 USC §121. (E) is not the most correct answer because the petition does not stay the period or necessity to reply to the rejection. 37 CFR §§ 1.111; 1.181(f).

Hopefully someone is around and willing to help on a Sunday afternoon :).

Thanks,
Rob

Reply

250 usptocheckNo Gravatar December 19, 2011 at 12:08 pm

Anybody know what this illegal Website does. We will check all user information.

Reply

251 0No Gravatar December 19, 2011 at 3:41 pm

….

Reply

252 GlennNo Gravatar January 3, 2012 at 7:01 pm

Can anyone confirm which MPEP version is available during the Prometric MPEP? Mypatbar.com seems to indicate it is still the 8th/Rev4, but the PTO website shows 8th/Rev8.

I’m taking the exam on 1/9 and I’m starting to freak out.

Reply

253 GlennNo Gravatar January 3, 2012 at 7:04 pm

this is where the PTO shows 8th/Rev8 for the patent bar:
http://www.uspto.gov/ip/boards/oed/aia_regexamsourcematerial.jsp

Reply

254 Mauricio AlvarezNo Gravatar January 3, 2012 at 8:20 pm

let me know when you find out…i’m taking it on the 26! Hopefully testing of new rules won’t be in effect by then…

Reply

255 Shaun KNo Gravatar January 4, 2012 at 2:44 am

This website is AWESOME!!

I took the Patent Bar today and passed it on my first try. The best part was I didn’t spend any $$$ preparing for it, apart from printing the study notes on this website. And the second best part was I looked at so many of the Qs from here that I almost knew when I was doing a BETA Q (so just made an educated guess, and come back if I have time). Oh… and BETA Qs tend to have grammatical errors (DEAD giveaway).

The annoying part was the search function on the PROMETRIC computers. You have to search from the beginning of an entire chapter, or so I though. Then I later realized you could type keywords to get to that subchapter, then search for your keyword from there.

For example, lets say the Q was about “Reasonable Diligence” under 102(g), and one of the answer choices was “The critical period for diligence for a first conceiver….” And you want to type this in word for word in the search field. But to save time you want to search from only 2138 and not the whole 2100 chapter. —- (1) you type “2138” into the search field, hit enter a couple of times till you get to subchapter 2138. (2) You change the “2138” in the search field to “critical period for diligence.” (3) You get your answer!

I hate to be the bearer of bad news, but I realized the answer choices in the latest questions have much more paraphrasing and less direct quoting. So you have to be smart; make your search phrases short, and pick out key words that cannot be paraphrased from the MPEP. For example you could search only for “critical period” or “first conceiver” in the EG above.

Also, the PROMETRIC computers are slightly better than obsolete. Long search phrases take longer times to process. I would keep the search phrases to between 2-4 words.

Ok, off to play SC2 for now and then to update my resume!! And finally, it’s really not that hard. For those of you who took the Bar (the State Bar exams), you’ll need only slightly more than the amount of time you spent studying PR to get this done.

–Shaun

Reply

256 Walter FosterNo Gravatar January 17, 2012 at 4:08 pm

Well, I took the Exam today and did not pass. About 60%, which is about 7 questions short. Some of my impressions: Lots 103/KSR questions, Appeals, Double Patenting, Re-Exam, Re-Issue, PCT. Only maybe 5-7 repeat questions. The Prometric Center and computer delivery was just OK. Know your KSR cold because the search engine DOES NOT WORK for additional source material (Federal Register) provided (other than MPEP).
That being said, about 3 days before the exam I did not feel I was sufficiently ready. At least next time I will know what to expect and can plan accordingly. Unless you are scoring 75-80 on the practice exams under real timed conditions, don’t even bother. I was not scoring well on the practice exams so that was an indicator. Regardless, I figured I would give it a shot and see what happened. I also moved too slowly through the exam and left myself only about 30 minutes at the end of each section. Good luck to the rest of you.

Reply

257 Juris PrudenceNo Gravatar January 18, 2012 at 12:12 am

Walter – I am sorry to hear that! Good luck next time and thanks for reporting back to the rest of us. I delayed taking my exam till I could do much better on the practice ones.

Will you be doing a retake?

JP

Reply

258 Walter FosterNo Gravatar January 18, 2012 at 7:48 am

I will be taking again. Probably mid May or earlier depending on my work schedule. Know the 103 material with KSR cold. Impossible to do any lookups at the Prometric here in Bethesda.

259 Juris PrudenceNo Gravatar January 19, 2012 at 1:35 am

Thank you for the update Walter. Seems unjust to me that you and others are expected to take the exam without having complete access to the supplemental materials! Do you think this is a region-specific problem? Is OED planning to resolve it soon?
I think you should be able to get a refund from Prometric if their system was at fault in this case. Good luck in May!
JP

Reply

260 JenNo Gravatar January 19, 2012 at 12:21 pm

You should be able to get a refund as I had the same problem (couldn’t search the supplement during my exam). I reported the problem to OED and Prometric and they credited me the $150 soon after.

261 BagelmanNo Gravatar January 19, 2012 at 1:07 pm

The supplements were also non-searchable for me on 1/18/12. Luckily, I saw that there was a problem on this site a few days before so I was ready for it. Most of the answers you need are in 2100 anyway.

I would have never passed the exam without this site. I tried taking it two months ago the legit way by using PLI and doing lots of questions, but failed with a 68%. I am convinced that looking over the reported questions here and getting the answers pushed me over the top for this attempt.

I will be going through and updating what I saw over the next few days so I can pass on the good karma.

262 maalvaNo Gravatar January 17, 2012 at 11:23 pm

Hi Walter,

I’m taking my very soon. What was your preparation like?

Reply

263 Walter FosterNo Gravatar January 18, 2012 at 7:50 am

I took the PLI course. I put about 100 hours into it. I only did about 4 3hr practice exams.

Reply

264 maalvaNo Gravatar January 19, 2012 at 3:29 pm

thanks Bagelman, i’m taking mine next week.

Reply

265 KerstinNo Gravatar January 22, 2012 at 1:59 pm

Lisa and others-a question regarding new material. How did you find out that this new material is going to be tested starting in January? I did not know about this. I am retaking March 19th, giving myself as much time as possible to study given that I work full time and am a coach in the winter.

Reply

266 MirandaNo Gravatar January 22, 2012 at 6:16 pm

January 31st is the date new material will start being tested per the USPTO and PLI websites

Reply

267 Lisa O.No Gravatar January 22, 2012 at 7:09 pm

Kerstin,
I found out when I sent in my application to re-take the test

Reply

268 KerstinNo Gravatar January 22, 2012 at 10:41 pm

Interesting…I didn’t get any kind of notification when I sent in my application to re-take. I just found all the info on the USPTO website though. Glad I read about it on here!

How did the review of your exam go in D.C.? I decided not to go. I’m retaking 19th. Probably would have taken it earlier had I known about the changing rules but winter for me is really as I coach busy so it’s probably for the best.

Reply

269 Juris PrudenceNo Gravatar January 26, 2012 at 4:50 pm

Does any recent test taker remember a question that involves India and outsourcing? I’ve seen it alluded to a few times, but cannot find the fact pattern on the posts. Thank you in advance.

JP

Reply

270 justpassedNo Gravatar January 27, 2012 at 5:02 pm

I did yesterday. I answered that outsourcing is not allowed. I passed.

Reply

271 pratikNo Gravatar March 8, 2012 at 8:08 pm

I got this question on 3/6. The fact pattern stated that a company outsourced patent drafting work to India, and asked if they needed to obtain a foreign filing license. The fact also stated that the technology would not be subject to a secrecy order.

The following from MPEP 140 was relevant to answer the question:
Note that the export of subject matter abroad
for purposes not related to foreign filing of a patent
application, such as preparing an application in a foreign country for subsequent filing in the USPTO is not
covered by any license from the USPTO. Applicants
are directed to the Bureau of Industry of Security at
the Department of Commerce for the appropriate
clearances.<

Reply

272 JenNo Gravatar February 16, 2012 at 12:29 pm

I just passed the patent bar using PLI’s Exam Focus Patent Bar Review materials. I purchased these materials full price ~$2,000 but I am selling them for only $500 because: they are from 2010, it does not include Patware and there are some highlighting and writing. Patware is now accessed online rather than through CD.

I have ALL the updated supplemental materials starting from 2011 until 2012 because PLI was kind enough to send them to me.

Although I bought the materials in 2010, I used all of the same materials when I passed on Feb. 1, 2012.

The materials include:

All lectures on CD (from 2010)
Course Lecture
Study Guide
Claim Drafting and Amendment Supplement
Study Guide Exercise & Practice Exam Materials
ClaimsLight/Claim Drafting Workbook
Post Course Materials
Key Words Glossary
Updates on the 2011 and 2012 changes

I am located in SF Bay Area so if you’re local, we can meet at a public place for you to check out the materials.

sertag@hotmail.com

Thanks for looking!

Reply

273 MirandaNo Gravatar February 16, 2012 at 12:36 pm

Is this version 8.4 or 8.8?

Reply

274 MirandaNo Gravatar February 17, 2012 at 11:12 am

Is the scratch paper they provide lined or blank? If blank, how did everyone keep questions in order- did you draw in your own lines?

Also been hearing that some test centers don’t allow writing during the 15 min “orientation”. Is there any way to check ahead of time without sounding suspicious?

Reply

275 RickyNo Gravatar February 17, 2012 at 1:13 pm

Is the scratch paper they provide lined or blank?
–It i blank.

If blank, how did everyone keep questions in order- did you draw in your own lines?

–For the am session, you may use that 15 minutues tutoring to make your answer grid sheet.

Once you finish the morning session, the program will start the 1 hour count down. You may choose to end the count dowm immeditely to start the pm session ( I bet nobody would…..). Therefore, you may enter the exam room ~ 10 minutes before the count down ends, make your grid sheet, then start pm seesion.

Reply

276 dcnativeNo Gravatar February 21, 2012 at 4:04 pm

This is a testimonial…

I passed the patent bar exam today (first attempt). Thank you to all of the contributors to this website (MyPatentBar.com). This website was my study guide. Over approximately 10 weeks I estimate that I devoted about 250 hours in preparation. Starting slowly at first and gradually building with intensity as my exam date drew closer. Basically, I studied the information provided under the “Study Guide” tab of the website during the early weeks, seriously, thoroughly studying the subject matter and referring when necessary to MPEP to add clarification. Also during these early weeks I read all of the supplemental source material identified on the USPTO website. This effort consumed the first 6+ weeks. Then, for about the last four weeks I intensely studied this website’s “Exam Questions and Concepts” and the “Repeat Questions” tabs, as well as the “Old Exam” questions/answers from the April and October 2002 and 2003 exams that were not included under the “Repeat Questions” tab. I can’t overemphasize that when you study these exam concepts and exam questions, you should NOT be doing it in an effort to commit the specifics of the question/answer to memory, but to familiarize yourself with just how patent bar exam questions are structured and to, most importantly, increase your understanding of the application of patent law in various scenarios. It is important to not only understand why the right answer is right, but also why the wrong answers are wrong. I spent a lot of time during this effort going back to the MPEP and reading (and understanding) the referenced provisions. This helped at lot because on my exam today there were no more than about 25-30 questions that were recognizably repeat questions (or variants). However, being able to carefully read the exam questions and answer choices and then “reasoning” (deducing) what I felt to be the correct answer based on my understanding of the resolution in closely similar scenarios I am sure benefited me. Finally, I think that the exam was a fair test of one’s understanding of the principles of patent law. I detected no trick questions or questions focused on nuanced minor facts. It benefited me greatly to have put in place a preparation strategy that I tracked closely on a regular basis during the 10 weeks. I wanted to give myself a fair chance of passing on the first attempt and I thank God that my efforts paid off. It was important for me also to not only go into this exam feeling confident about my preparation, but also to take a deep breath, slow down, and be sure to intently concentrate during the exam. I completed the exam one question at a time on a pace that essentially consumed the two full three-hour sessions (100 questions total). Again, special thanks to the MyPatentBar.com website owner(s), and thanks to all the contributors. The questions and answers provided by the numerous contributors (in the forums) were very helpful too. Thanks.

Reply

277 shivaniNo Gravatar March 11, 2012 at 12:31 pm

Hi everyone,
first of all thank you so much for posting such useful information on this website. I am about to sign up for one of the Prep Course as I want to take the Patent Bar in July or August. I would highly appreciate if you can share your feedback regarding OmniPrep vs. Patbar. I cant afford any other courses, so I will be choosing one of these two.
Thank you in advance
Shivani

Reply

278 JenniferNo Gravatar March 19, 2012 at 12:53 am

I took the exam recently and passed on the first try. there were a lot of old exam repeats and new repeat questions from this website. There were also a lot of new rejection questions by the board.

I used the PLI patent materials. Please email me at jrshih@ucdavis.edu if you are interested in purchasing my materials which includes the new sections tested.

Reply

279 MNguyNo Gravatar March 22, 2012 at 5:10 pm

Passed the exam today on my first try thanks to this website! Thank you everyone for the comments/advice/instruction and this site for bringing us all together to prepare for a fairly challenging test. My study plan consisted of

Over the last two months, during lunch, evenings, and weekends,
1. Reading the PLI outline front to back.
2. Listening to all the PLI audio cds (driving to work)
3. Skimming the entire MPEP and reading relevant sections.
4. Doing all the PLI practice problems.

Then the past two weeks I really buckled down,
1. Did 2000-2003 exams in 50-question increments.
2. Read the USPTO supplemental material on Bilski, KSR, and accelerated examination.
3. Got familiar with the MPEP structure by reading all the headings.
4. Re-listened to the PLI cd’s I still had questions on.
5. Read every repeat question posted on this website until I had them cold.

The exam itself was harder than I was expecting and I looked up way more than I was planning to. Probably 50% of the questions I was unsure of and had to look up. For both AM and PM sessions, I went thru the questions putting down my best guess, and marking the questions with a check when I knew I had it right, and a ? when I wasn’t sure. Then I went back and keyword searched the MPEP for the right answer. As for repeat questions, there were about 6-10 in each session. Lots of appeal questions, lots of obviousness questions, a couple means-plus-function questions, and a few KSR questions. During the exam, my computer froze twice and I was moved to different computer. Overall a really good experience, and really really glad to be done.

Reply

280 vertyNo Gravatar April 6, 2012 at 4:19 am

The PLI practice problems are easier than the actual exam questions from what I’ve seen. Based on your experience taking the exam, how would you rate how helpful the PLI practice problems are in terms of preparing for the exam and in relation to the questions you actually saw.

Reply

281 BfusionNo Gravatar March 30, 2012 at 9:30 am

Question about Prometric sites in the Philadelphia area:

Does / has any had experience with these sites (specifically, either the one Downtown or the one in Conshohocken?) I’m awaiting the green light to take the exam, and just wanted to tap the resources here to get feedback (if any) on these sites.

Reply

282 BaronNo Gravatar April 12, 2012 at 12:53 pm

Bfusion – I used the downtown site on 1/10/12. I walked from the PATCO station on Market to the test site. There is a refrigerator to put some lunch in as well as some lockers for clothes and other items but there are not many of them. I just hung my coat up on a rack near the receptionist. There is only one restroom on the floor you are permitted to use, it only holds one person, and requires a key. If someone else is using it, you either have to wait or you might be able to go to the lobby and have one of the info guides give you a key to one of the lobby restrooms.
The Prometric staff was very professional and pleasant. I booked an early morning session and they let me start before my official start time since there was no reason to just sit in the waiting room. I found I needed to use the headphones to cut down on ambient noise. I was surprised on just how little noise I was distracted by. I was able to use the 15 min of “prep” time for anything I wanted so you can make an answer chart, write down time checks, do a brain dump, etc.
I will be using the facility for my second try shortly. Thanks for the spreadsheets. They are keeping me from losing faith in passing.

Reply

283 BfusionNo Gravatar April 12, 2012 at 3:01 pm

Baron,

Thanks for the info! I registered for the Conshohocken site (close to me/easier than getting in and out the city). Hopefully it’s a decent place.

Keep at it (I’m glad I could be an encouragement). We’ll make it through this.

~bfusion

284 OG1No Gravatar April 4, 2012 at 8:31 pm

I passed today – first time. Morning session was bruising experience with few repeats and a lot of obscure look-ups. However, I persevered through a much easier afternoon session with a good number of repeats from 2003 tests, and squeaked through. Although I studied the newer questions indicated on this site (lip gloss,ship’s bell, etc.) I didn’t see many of these today except the Broom question.
I want to credit and thank this site and its contributors (especially Mambo’s repeat question tests, which I reformatted from the PTO site so that I had all the questions on individual sheets along with parallel sets of 50 answers). As others have stated here, it was important to not only do all the repeat questions and 2002/2003 tests repeatedly, but to understand the reasons for right/wrong answers as part of the learning process.
Finally, I found John White’s PLI lectures very helpful, as what I picked up here made a big difference in ascertaining the way to go on questions with subtleties and close variants of answers.
Thanks again and good luck to all those still studying for the exam.

Reply

285 MirandaNo Gravatar April 4, 2012 at 8:45 pm

congrats OG1!!! Did you by chance add any questions to bfusion’s comprehensive question spreadsheet?

Reply

286 BfusionNo Gravatar April 5, 2012 at 7:57 am

OG1…CONGRATS!

Yeah, if you remember any questions, and don’t want to touch the spreadsheet I posted, just list them here and I’ll take care of it.

Also: If you feel like sharing your re-compilation of Mambo5’s exams, you can upload them to google docs that we’re using to share resources.

the login: mypatentbarr @ gmail . com
password: usptoposita

Thanks, and again, congrats!

Reply

287 JTKNo Gravatar September 15, 2012 at 1:14 pm

Google Docs said the password changed! what is the new password, i would like to work through Mambo5’s tests

288 ENo Gravatar September 21, 2012 at 11:33 pm

Hi Bfusion,

Are you sharing these Google Docs? If so, can you give me the link?

Also, what’s Mambo5?

Thanks

289 sgNo Gravatar August 15, 2013 at 8:26 pm

Was the password changed for the google account??

290 MirandaNo Gravatar April 5, 2012 at 2:01 pm

Yeah, it would be really helpful to have more reviews of the prometric testing centers. I’ve heard that some centers don’t allow “brain dumping” during the 15 min introduction.

Reply

291 zanNo Gravatar April 12, 2012 at 7:44 pm

Miranda, It’s good you bring that up. What do you plan on brain dumping in the 1st 15 min? I should do the same but I don’t know where to begin. Any suggestions?

Reply

292 ARosNo Gravatar April 14, 2012 at 10:28 pm

I got a provisional pass this week and this site helped me do it. I came across this site two weeks before the exam, and spent much of the last week before the exam working through the just the repeat Qs identified in Bfusion’s spreadsheet as occurring in the last month, or consistently over the last 6 months. I got a lot of repeats this way, which really helps with time pressure. Now it’s time to do my duty and give back to the community that helped me out. Here’s the repeat Qs I remember seeing (I’m not confident in my ability to add to that spreadsheet):

Old Repeats: Laurel, Abbott, and Hardy; Tommie and Jo; Potter – 2 variations; ABCD/ABCDE/BCDE; Moondust; 103 Obviousness (10.03.16 P); Polypropylene ranges.

“New” Questions: Tables on CD-ROM; Claiming/Deleting/Reclaiming Benefit; Costa Rica; Broom (multiplicity of claims); Death of inventor – fishing device; Indefinite Claim using High;

Also a 112 para. 4 question, where a dependent claim adds no limitations. The MPEP conflicts with the supplemental material (MPEP says object, Supp. material says reject) so I went with the supplemental material..

Reply

293 BfusionNo Gravatar April 15, 2012 at 5:22 pm

ARos,

congrats on the pass! and thanks for sharing what you remember! Don’t worry, I’ll update the spreadsheet.

Reply

294 GDBNo Gravatar April 17, 2012 at 8:34 pm

Can anyone give advice on Bilski questions that have been asked. I’m not sure which supplementary materials to rely on. I’m taking the test in a couple of days and don’t have time to read all of the Bilski materials.
Thanks a lot!

Reply

295 GDBNo Gravatar April 20, 2012 at 12:07 am

Passed first time against all odds with a big 9 month pregnant belly! My computer froze twice during the exam but staff put it back online very quickly and I didn’t lose any time (I don’t think).
Morning session was really hard, afternoon session was much easier. Hardly any long questions. It seems that the trend is now to have shorter questions.

Didn’t get any Bilski questions.

Got 1 or 2 KSR questions
KSR question: asking what examiner would rely on for an obviousness rejection. My choice came down to 2 answers:
One answer said that examiner would not resolve Graham factors and instead would use one of the KSR guidelines. Didn’t choose that one.
I chose the one which said examiner would use one of the KSR guidelines (even though the answer didn’t mention the Graham factors) because post KSR Graham factors are definitely still relevant so first answer was clearly wrong.

Got tons and tons of repeats/new questions that were posted on this site – I would say close on 50% of the exam. I’m going to go through the different sections on this site and try to update them all.

Thanks to everyone for their contributions. This site is clearly what allowed me to pass.

Reply

296 BfusionNo Gravatar April 20, 2012 at 9:52 pm

GDB.

WOW! Congrats, and THANKS for the updates and suggestions.

Reply

297 GDBNo Gravatar April 20, 2012 at 12:34 am

New Question that I hadn’t seen before:

Spec discloses composition A comprising 25-60% x and 6-11% y with a preference for 36% x and 8% y (or something like that)

Claim is for composition containing 30-60% and 6-11% y.

Prior art discloses compound with 30% x and 8% y
Claim rejected on a 102 objection based on prior art.
Applicant amends claim to contain “at least 35% x” and 9% y
Claim rejected on same 102 rejection and a new 112 rejection for no written description.
Applicant amends claim to 35%-60% x and 9% y.
Question: in the next office action, what will examiner do? There were a number of combinations:
1. Remove 102 rejection and keep 112 rejection
2. Remove 112 and keep 102 rejection
3. Maintain both rejections
4. Removel both rejections.
I wrote remove both. I think this might have been a beta question because I think that the 112 rejection should have been on the basis of indefiniteness rather than written description. But I’m not sure.

Reply

298 GDBNo Gravatar April 20, 2012 at 12:39 am

Got a question on 2 inventors who die in a car crash. The question is who will USPTO send the correspondence to. My answer was the FIRST inventor listed. I think that’s right because the power of attorney ends after the death of an inventor. There were no answer choices saying to send to both inventors.

Reply

299 KeenerNo Gravatar April 29, 2012 at 8:28 pm

Did the question say the USPTO knew the inventors were dead. Unless the living inventor or counsel informs the USPTO, won’t the correspondence address remain the same?

Reply

300 GDBNo Gravatar April 20, 2012 at 12:50 am

Question on oaths. There are 3 inventors and three separate oaths are made each with a single name of an inventor and their signature. Each of the oaths leaves blank the line for “second inventor”. The question asks whether the 3 oaths are proper.
My answer was that they were not proper. Separate oaths can be signed but each oath must state the names of all the inventors even if it is only signed by one of the inventors.

Reply

301 GDBNo Gravatar April 20, 2012 at 1:13 am

Question on appeals – oral hearing. When can you get an oral hearing? I think the question was: which of the answers is NOT in accordance with MPEP. One of the answers I remember: once a request is made the Board has to grant the hearing (I think that’s right)

Reply

302 GDBNo Gravatar April 20, 2012 at 1:17 am

Forgot to mention: I’m selling my PLI patent materials VERY cheaply – $150 + postage. They are a 2010 version, are not marked up at all and come with the Patware CD. I’m also willing to throw in the full notes from PRG for a small extra fee but those have been marked up and have the answers to the questions also marked on them. I don’t have the software for the PRG questions.
Please contact me at 4158284517.

Reply

303 S Fay-RichardNo Gravatar May 1, 2012 at 3:38 pm

Are the materials still available? I will definitely purchase the whole package.

Thanks and congrats,
SFR

Reply

304 GDBNo Gravatar April 20, 2012 at 4:41 pm

I had a PCT question on the difference between Art 19 and Art 34 amendments: Priority date was given, PCT filing date was given, Date of ISR (intl. search report) was given – ISR date = 12/07/00. The facts stated that the applicant did not request a preliminary examination (Demand under Ch. II).
The question was: by when can the applicant file amendments and to what. Two of the answers were as follows:
1. By 02/07/01 to the claims
2. By 02/07/01 to the claims and spec and drawings

You needed to know that if no Demand under Chapter II then you could only amend prepublication under Art 19. Deadline is either 2 months from ISR or 16 months from priority date. The MPEP says that under Art. 19 amendments can only be made to the claims.
If a demand is made under Ch. II then the deadline for submitting amendments to the claims, spec and drawings is 3 months from ISR or 22 months from priority.

I read in my PLI notes that it is now possible to make amendments to claim, spec and drawings under Art. 19 but that wasn’t stated in the MPEP so I went with the MPEP.

Reply

305 BfusionNo Gravatar April 20, 2012 at 9:58 pm

GDB:

If it’s not too much trouble, could you make a list of questions you remember? I know you mentioned you’re going to go through this site and update each one, but the “recent comments’ only lists ~ 8 recent comments (and I think I’ve missed some of your updates). I’m going to probably do one more update to my excel spreadsheet posted here, and I’ll use your recent memory if that’s ok!

Thanks!

Reply

306 GDBNo Gravatar April 26, 2012 at 1:13 pm

it looks like a lot of my comments didn’t go in – that is such a shame. I spent hours going through each of the questions and commenting. I’ve just had my baby so I’ll try and find the time to make the list again and hopefully I can do it before I forget. What a bummer!!

307 BfusionNo Gravatar April 29, 2012 at 10:01 pm

GDB: CONGRATS on the baby!! AND the pass. Man, I don’t know HOW you did that. Frankly, us guys can’t hold a stick to you for doing what you did! (and yes, I’m talking about being pregnant at the same time as studying and passing the exam).

Sorry that your responses weren’t posted =( a big loss for us all. I look forward to getting more feedback from you soon.

308 KeenerNo Gravatar April 27, 2012 at 10:43 am

Has anyone tried the Prometric Test Drive?
http://www.prometric.com/TestDrive
Was it a waste of time?

Reply

309 RTNo Gravatar May 1, 2012 at 11:24 am

Yes, I did the test drive. I think it was worth the time and money fro me. It allowed me to get the feel of the place before the test. I got my identity papers in place and saw how it all worked. They explained how I could enter and leave and explained no water at the test table, no jacket, no food, etc.

My test place didn’t allow taking paper in or out, they provided the scratch paper (unlined).

I also saw how the computer screen worked and looked.

You will NOT be able to see the MPEP or see anything about the test. But you will see how the answers can be noted, i.e. how can you note that you want ot go back and check an answer.

Reply

310 SNo Gravatar May 1, 2012 at 5:35 pm

I have to submit my application for patent bar exam.

I would like to know what are the fees I need to pay to qualify and take the exam in USPTO office in .

Are they only 8a. Application fee $40.00
8b. Registration exam fee: Is it for $200 or $400.00?
Is there any other fee?
Thank you

Reply

311 RTNo Gravatar May 1, 2012 at 6:41 pm

$240 is the total required. I had it drawn at the post office. Sent everything thing in, including transcripts, and received my letter in about two weeks.

Reply

312 SNo Gravatar May 1, 2012 at 9:13 pm

Thank You RT!

Reply

313 BfusionNo Gravatar May 15, 2012 at 8:48 am

Don’t Forget, WHEN you pass, you’ll need to pay $100 to register yourself as an agent/attorney as well.

I forgot about this fee…totally worth it, but still…another $100 bucks!

314 RTNo Gravatar May 15, 2012 at 8:50 am

When do you pay this? Before official notice from PTO?

315 BfusionNo Gravatar May 15, 2012 at 9:24 am

RT,

you’ll get an official letter from USPTO saying “You’ve officially passed. Now send the included oath back (after getting it notarized!), with $100, and after they make sure everything is in order, you’ll be officially registered”

316 RTNo Gravatar May 15, 2012 at 9:41 am

Sweet. How long until you got your letter from USPTO?

317 BfusionNo Gravatar May 15, 2012 at 9:47 am

I got my “official letter” about ~1 week from when I took the exam. I’ll get the official document to hang on a wall sometime soon (i expect within 30 days of when they get the payment, etc).

318 RTNo Gravatar May 15, 2012 at 10:28 am

Thanks. Perhaps just a few days then… I passed last Thursday!

I’m still very excited. I do not take this for granted.

319 dcnativeNo Gravatar May 15, 2012 at 11:09 am

$240 to USPTO to register for the exam
$150 to Prometric to take the exam
$100 to USPTO to register as a practitioner after passing the exam
Then once you’ve paid the $100, USPTO publishes your name on their website and in the Official Gazette to check to see if anyone has any unethical or amoral info on you that would keep you becoming a registered practitioner…
Then once you clear this hurdle… USPTO will place you on their registry of registered practitioners on their website…
Then finally they send you your nice Registration Certificate so that your can deservedly gloat over it.
It took from the day I passed the exam until the day I received by certificate in the mail 10 week plus one day…. So be patient … but you will absolutely thrilled to see your certificate and to finally be able to frame it and put it on your wall. Then, get busy… you have work to do!

320 RTNo Gravatar May 15, 2012 at 12:05 pm

Thanks for the info DCNative.

321 dcnativeNo Gravatar May 16, 2012 at 5:40 pm

For those of you who are awaiting their registration certificates and have never seen one… see the link below. This is the website of a patent attorney who displays an image of his registration certificate on his website. The image provides a great depiction of the registration certificate that USPTO will send to you. The dimensions of the certificate are 8-1/2 inches by 11 inches (in case you are intending to pre-order a frame).

http://www.hannip.com/Professionals.php

Reply

322 PDubbsNo Gravatar May 15, 2012 at 8:43 am

Just got a provisional pass yesterday. My exam felt like it had virtually NO repeat questions (had reviewed all the questions on this site, PTO website, and in the PLI materials) and the morning part was definitely harder than the afternoon. Probably 10 or so questions on the new KSR/Bilski stuff as well. Will try to post questions as I remember them: here’s one (easier one):

Small Entity status entitles you to a reduction in all fees except:
(I chose fee for recording title in USPTO)

Reply

323 PDubbsNo Gravatar May 15, 2012 at 8:48 am

Inventor claims priority to German app filed in May 2000, selects USPTO as receiving office, fails to file required materials under 35 usc 371 by November 02.

Options:
(several that said ‘can still enter national stage’)
I chose one which said cannot enter national stage in US b/c 30 month window closed.

Reply

324 BfusionNo Gravatar May 15, 2012 at 9:26 am

PDubbs,

Congrats on the pass! Good work!

A quick question: About how long (or how many times) did you study the “old” exam questions? Once or twice, or would you say you had them down cold?

I’m still trying to get a sense of how this exam is structured, and people’s responses.

Thanks!

Reply

325 AnnaNo Gravatar May 20, 2012 at 8:35 pm

Is there anybody who submitted a certificate of good standing from the California Supreme Court to register you as an attorney? I have never gotten certificate of good standing and no idea where I should do start. I did some google search, but did not clear idea, yet. Any idea? Thanks.

Reply

326 mrsinnedNo Gravatar June 4, 2012 at 8:41 pm

Yes. I submitted a written request to the Cal Sup. Ct. about 10 days ago and I just received the certificate today.

Here’s the language from the letter I sent:

“Re: Request for Certificate of Good Standing

Dear Sir or Madame:
Please send me a Certificate of Good Standing. My name is Dennis James Smith and my State Bar Number is 233842.
Thank you.
Sincerely,

Dennis J. Smith”

Mail self-addressed stamped envelope (along with check for $1.00/ certificate needed) to:
California Supreme Court
350 McAllister Street, Room 1295
San Francisco, CA 94102

Reply

327 khdqqsNo Gravatar May 21, 2012 at 10:42 am

jqsfh , breast actives reviews http://www.pillbreastenlargement.com/

Reply

328 mrsinnedNo Gravatar May 21, 2012 at 8:13 pm

DO NOT take the PTO Reg Exam at Prometric’s Glendale, CA location. I just tried to do so for two days (May 16 and 17) and the MPEP viewer kept freezing and restarting the computer. Prometric’s technical support people could not figure out what was happening and tried to run the exam on at least 8 different computers over that two-day period. The customer support people had to reset my eligibility twice in two days because of this. I then took the exam this past Friday (May 18) at the Culver City location and everything was fine–and I passed.

Reply

329 KeenerNo Gravatar May 21, 2012 at 8:21 pm

The OKC Prometric site is warm, but otherwise a good place for the exam.

Reply

330 DanielNo Gravatar June 5, 2012 at 9:46 pm

I passed today on my first try. Thanks to this website.
I have some experience in patent prosecution, so I have not spent a lot of time for this exam. I started with 2003-2002 exams and understood concepts through reading answers. After that I read repeat questions and marked questions on my printed 2003-2002 based on people’s review. I read exam concept and questions section and tried to condense keywords of questions and answers.
I think I got almost 15-20 repeat and exam concept questions. I got various new types of questions.
Further, I got the same questions twice with slight variation. One correct answer was (A) and the other was (D). They changed answer choices by inserting ‘not’ and changing ‘proper’ to ‘improper’.
One questions includes if you receive OA on February 28. When is SSP May 28 was the answer I think.
I cannot remember specific details since I spent a lot of time for each question. I reviewed MPEP for each question with fixed amount of time. Thus, when I finished #50, I had less than 45 mins in AM session, less than 25 mins in PM session.

I got the same questions what people got recently in 2003-2002 and exam concept questions. So I will not pinpoint what I’ve got. My advice is that mark recently repeated questions and understand those questions thoroughly. It will save enough search time for difficult questions.
It appears that there are more variants and new questions in the question pool.
Understand appeal, KSR, and Bilski well. I got several questions.
reexamination questions were difficult for me, since I am not very familiar with that. I got second reexamination question.
A requested first ex parte reexamination. find true statements

I. A can file reply in two months after patentee filed reply…
II. ‘C’ may request second reexamination when first reexamination is pending if substantially new evidence …
III. A may request second reexamination when first reexamination is pending if substantially new evidence …
Something like that.

Reply

331 PatentlyawsomeNo Gravatar June 6, 2012 at 6:02 pm

I passed!!! Here’s what I did:

Reviewed 2002/2003 questions on and off for a few months
Reviewed questions by TOPIC
Reviewed major flows for 102 rejections, and PCT
Reviewed the guidance docs for ALL the new changes on the test
There were far less repeated questions than others have said
Know the highlights of the new changes, anything with numbers, time limits, etc.

TAKE TIME to study, take 6 months on and off if you need to with a push at the end. Repetition is key.
Good luck!

Reply

332 passedatlastNo Gravatar June 20, 2012 at 11:52 am

Hello! I passed 6/19/12 and wanted to give some warnings about KSR/Bilski questions. I didn’t see a link for it under the New Questions section, so hopefully someone will repost this somewhere appropriate.

I had a Bilski question that started with a method patent involving advertising. The claim is too long to remember, but my recollection is that involved gathering data then using it to target ads. The PTO rejects the claim over Bilski, stating the claim doesn’t meet the machine or transformation test and that the claim is abstract.. The question then is what the prosecutor can argue in response.

A. Examiner should allow the claim because the Supreme Court said the machine-or-transformation test is of no use (clearly wrong answer–Bilski court said MOT is a helpful tool)

B. Something absurd.

C. Examiner should allow the claim because a transformation had taken place (I found this to be weak given that I’m a patent law professor and could think of only one CAFC case offhand involving EKG results where data alone was enough for a transformation, given some medical treatment could be adjusted based on the data).

D. Prosecutor should amend the claim so that it depends on a machine and should emphasize that this claim involves the application of an abstract principle so it’s okay (I also wasn’t impressed with this answer. First, it was unclear whether the spec would support the amendment. Second, Prometheus told us mere applications of products of nature don’t meet 101 and I could see the same idea carrying out to abstract concepts. Though maybe it’s okay in light of Diehr/Flook? Who knows. This is what I picked for lack of a better choice, but maybe C was the better answer?)

E. Something absurd.

If you got this question and failed by one question, it is worth contesting. Granted, it might be a beta, but I got it in the afternoon and would have thought that all the betas would be in the morning (from a test-writing perspective, that would make more sense, but then again, this is the PTO we’re talking about!).

I also had a crazy KSR question. It involved an inventor whose claim was rejected in light of two references (one from the same field, one involving a similar problem) being combined under 103. Was the rejection proper? I remember two of the answer choices:

A. No because the PTO wrongfully considered the teaching-suggestion-motivation test (this is wrong–although TSM is seldom used in litigation, it isn’t dead after KSR)

B. Yes because QUOTE [some made-up but logical principal] ENDQUOTE. (this is what I picked, because it was the least wrong of the group, but I don’t know why it was quoted–I haven’t seen it in any fed case)

I’ve taught patents for several years and was a patent litigator before that, so I was surprised to see just how crappy these Supreme Court questions were.

Overall exam: 10+ PCT questions. No need to read 1800 or the PRG treatise on PCT, just read through the questions posted on this website and read several times the PCT provisions that apply in those questions (such as 1893). The same few provisions showed up multiple times. There was one tricky question involving how to correct inventorship during the first stage that I posted in more detail under “PCT Publication” in the exam questions and concepts section.

10+ Appeals questions. Here my PRG treatise was useful, as were all the questions posted
here. Someone said to read 1214.06 several times and I’m glad I did–I got 3-4 questions from that section alone.

About Prometrics: Although my ticket said 8am start time, Prometrics didn’t really care when people showed up, as long as your exam concluded before closing time. Your local center may vary! I was allowed to bring in 2 kleenex that they provided me with (oddly, only 2, and I had to surrender them after the exam!), and was permitted to use my own pair of foam ear plugs. Which is good because the first half of the exam I tried using their quieting ear muffs, which didn’t work well with my glasses. The chair was so large that I ended up hurting my back and was grateful to a random test taker who shared painkillers. I knew this could happen yet was powerless to prevent it, given we can’t bring in a back cushion or even take off our jacket and use it for lumbar support.

My computer died 3/4 of the way through the exam. I was doing a search, and first the search software crashed and then the whole computer. But no worries–everything saved to a remote server, and they had me switch computers and restored my exam. Nothing was lost and the entire process took less than 10 minutes. Which is good because my 90-day window was almost up.

After the exam, there was 4-5 questions about my test-taking experience that took all of 2 minutes to answer, then I had my pass notification. They printed it out for me and embossed it with a Prometric seal.

Overall, I want to say thank you thank you thank you to this website! The cramming of new and recent questions the last 2 days of studying is what allowed me to pass. You are awesome!

Reply

333 mojoNo Gravatar June 20, 2012 at 5:51 pm

Anyone have a good summary of the new changes that are relevant? I have PLI materials that haven’t been updated, and am working off old questions. In reading the answers, I’m not finding too much that has changed materially. I saw a few references in the previous threads above regarding changes of time limits/numbers and now I’m wondering if I’m missing anything big. Thanks!

Reply

334 passedatlastNo Gravatar June 24, 2012 at 11:43 pm

Another question I remembered:

Which of the following is not a new application?

A. A provisional patent application

B. A non-provisional patent publication [may have listed a CFR provision]

C. Continuation

D. RCE (what I picked–somewhere in the MPEP later on in the exam, I stumbled upon a statement that an RCE is not a new patent application)

E. Something else (maybe CPE? Divisional? Can’t remember)

Reply

335 PotentialNo Gravatar June 25, 2012 at 2:34 am

Congratulations Passedatlast!!

Thank you for posting the questions, I am about to take the test in 10 days, your information is invaluable to me. I am little bit nervous about this, but I will try my best just like everyone else did.

Thanks again!

Reply

336 YoungPassingNo Gravatar July 5, 2012 at 11:47 pm

Took the Patent Bar today (7/5/12) for the first time and passed! No more studying! Yay!

Background:
I just graduated with my bachelors in engineering and am heading to law school to pursue Patent Law. I decided to study for and take the patent bar this summer to try and get it out of the way. My background is strictly technical.

Study Strategy:
I took one day off between my last exam and studying for the patent bar. I bought PLI off of Ebay. I’m currently working as an engineering intern, so I would study for an hour during lunch, go home and study for 2-4 hours a night after work, and 5-8 hours on the weekend. This was done for about 9 weeks. About three weeks before the exam, I started re-reading chapters 100, 200, 700, 1800, and 2100 in the PLI binder. Good stuff. I read those about 3-4 times. When I was going through, I also made flashcards. Personally, I found that this really helped me remember short facts, dates, and rules. In addition to following PLI, I did fake tests on a daily basis, created via Patware. Of course 2002-2003 exams are a staple. I happened upon this site about 2.5 weeks prior to the exam, and reviewed material here daily. I didn’t realize until I took the exam today how helpful this site was, especially when it came to the new material. The majority of new questions that people are talking about (referring to bilski, ksr, bpai), I encountered today. Honestly, it saved me. This site also helped in that the discussions people have about the material really pull everything together. I would just skim the discussions going on in the repeat questions sections. I also read the study guides for appeal, PCT, 103, and 102.
I took work off this week, and went into hardcore cramming mode 4 days before the exam. On Sunday, I discovered that I was one of the unfortunate people who was gypped when they purchased PLI off of eBay. It turns out, what was advertised as the 2012 version of PLI, was really the 2010/2011 version. BEWARE WHEN YOU BUY PLI OFF OF EBAY! Here is the article which enlightened me as to my misfortune: http://www.ipwatchdog.com/2012/05/08/buyer-beware-counterfeit-patent-bar-review-courses-on-ebay/id=24719/.
For the past 4 days, I locked myself in a room and read this site and googled everything that had to do with bilski, ksr, and bpai. I also reviewed Mambo5’s tests, 2002-2003 exams, combed over this site, and used the PLI Patware to make fake exams. I was at the point with the 2002-2003 exams where I didn’t have to even look at the question, I could tell what the correct answer was buy looking at the answer choices.
I will say, even though I didn’t purchase the correct version of PLI, I really don’t think it was that big a deal. John White’s lectures were excellent in teaching me 85% of what I needed to know. Patware was great making fake exams. I just wish I would have known that it wasn’t the most updated version.

Here are some statistics to my studying that I figured out today:
Took 85 practice tests
Answered 2500 practice questions
Created 250 notecards
Studied for 200-225 hours

I share this because it might give you an idea of what to budget for when it comes to studying. Who knows? I might have gone overboard, but I passed on my first try, so there might be a little truth :)

The Exam:
Prometric is sketch and super strict. No lip gloss, water, etc. into the exam room. Every time you enter the room, you have to sign in, turn your pockets inside out, and get waved down by the metal detector. One thing that I read on this site was the suggestion to bring ear plugs. I did that, and it helped immensely. When I practiced at home, it was completely silent. When you go to Prometric, there are people taking all kinds of tests, and some require a lot of typing. Personally, the sound of someone typing blew a bit of my concentration at first. I just put my earplugs in, put the Prometric ear muff things on over top, and I was good to go. The computer did freeze up on me once, but I was up and running on a different computer in no time. One thing that I didn’t really account for was how long it took the computer to search for a document. I wasn’t used to it being less than super quick, and that caused for me to modify how I was searching. Also, when you select a chapter to search in the MPEP, you have to highlight the chapter, and then press ‘Go’. Wasn’t that way in Patware and it took me a little adjusting to realize that the chapters don’t switch unless you press the button. Overall, a decent experience.
Oh, I did go the day before the exam to just figure out where Prometric was located, and I found that it helped ease the anxiety a little bit.

On the first section of the exam, there were about 15 repeat questions. There were also a handful of questions that have been discussed here that relate to ksr, bilski, etc. There was the blood pressure question, and a couple 103 ksr questions. I actually ran out of time and had to guess on 4-5 questions.

The second section was a bit weird. There were only about 5 repeats, but the other questions were easier than the first section. The majority of the questions were found verbatim in the MPEP. It was a little tedious to do all of that searching, but that’s just a testament to how important it is to know your MPEP inside and out. Essentially, figure out which chapter something is in, and search via a phrase that is pulled out of the question or answers. I found that I did a lot better in the second section.

Here are the repeat questions that I remember:
Public Use (10.03.47p)
Ex Parte Rejection (10.03.50p)
Claim Counting (10.03.50a)
NonProvisional Filing Date (10.03.1p)
Foreign Priority (4.03.22p)
Final Rejection Period for Reply (4.03.4p)
Provisional Filing Requirements (4.03.40a)
Beck – Mixture Y with melting point of 150F (10.03.11p/4.03.22a)
File by Fax (4.03.5a)
Notice of Appeal (4.03.8a)
Potter (4.03.12a)
Japan/nonpublication request/45 days (10.03.44a/4.03.2a)
File by Fax (4.03.5a)
Tommie and Jo (4.00.23a/4.00.24a)

I’m sure there were more, but my brain is mush right now.

Here are the topics that showed up on my test a lot:
103 obviousness
appeal
pct
ksr (about 5 or so)
certificate of mailing
signatures (electronic, who can sign what, etc)
reexam
reissue
1.131 and 1.132

It seemed like I was in a never ending cycle of questions that related to those topics.

Here are topics that were tested on very little, if at all:
claims (claim formats, multiple dependent, etc)
foreign priority
102a,b,c,d,e (questions were asked that had to do with these relating to 1.131, 1.132, or obviousness, but not by themselves)

That’s all that I can remember for now. I might post more as they come back to me. Thanks to everyone on this site!

Reply

337 PatentGrrlNo Gravatar July 6, 2012 at 6:45 am

(Sorry, should have posted here, not on Study Guide)

I’ve already posted that I got a prelim pass on my first try, 7/5/12, but I wanted to share more since this website is the biggest reason I passed. First, I have a background in engineering, I graduated from law school in 2011, I passed the July 2011 state bar, I have been clerking for a judge for a year, and I will be joining an IP boutique law firm in August. I only took copyright and one IP seminar in law school, but no patent or IP survey courses. To give you an idea on my test taking skills, for the state bar, I did an online only course and studied all day every day for two months, but I was sick the whole time. I did the typical flash card studying. I scored a 147 on the MBE. I don’t think flash cards would help with the patent bar and I did not do any.

For the patent bar, my law firm bought me the 2012 PLI review course. (They’re awesome! Even if I wanted/thought I could sell it, the videos are all online and can be viewed from only two computers, which I’ve done at work and at home, so selling it would be worthless). It was very up to date and did a great job summarizing the new stuff. The only Patware quizzes I did was on the new material (other than the review quizzes and such at the end of each lecture). Between that and the awesome outline on bfusion’s gmail account, I felt prepared for the new stuff.

I then took off Friday- Thursday from work and went through every question listed on the repeat questions tab here. I spent at least 20-30 minutes on each question, reading the MPEP section for every right and wrong section. I ended up printing page 700-27 (how to overcome a 102(e) rejection) and 700-273 (situations where 131 affidavits are inappropriate) and memorizing them since so many questions have to do with them. That’s all I memorized. I also went through all 68 questions on the Exam Questions and Concepts tab. Then on Wednesday, I retook the October 2003 exam. But that’s all I did. I put in maybe 100-120 hours studying. I spent about a week going through most of the PLI lectures in mid-June while my boss was out of the office. So I basically spent two solid weeks preparing

Testing facility: I took the test at Georgia State. It was perfect. There is parking immediately behind the facility for $8.50. It was very easy to find, both the location and the testing room in the building. There’s a lobby where you can eat lunch (I brought mine and left it in the locker) or there’s a place Georgia State students eat at in the building (you have to go out to Piedmont to find the entrance). The temperature of the room was perfect. I get cold easily, so I wore sweatpants and a sweater and I was very comfortable. Someone who gets hot easily would have been comfortable in shorts and a t-shirt. It was very clean. The chairs were comfortable and the monitors were big. The proctors were very friendly. The computer worked great. The only problem came when I double clicked on the title bar to maximize the MPEP. Then, every time I clicked in the document, it minimized. So I closed the window and reopened (took all of three seconds) and it worked great. Opening the MPEP and searching was surprisingly fast. I looked up every question (just about) as I went and had time to spare at the end of both sessions.

One note: I thought I would be able to select questions individually at the end of the session and go back to them. But if they weren’t marked, I couldn’t go back without going through every single question. And for the ones marked, I had to go to the first one marked and go through every one in order. So I was glad I mostly looked up every question as I went.

I’ll go through the website identifying the repeats individually as best I can. The morning felt much more difficult with very few repeats. The afternoon was easier with more repeats. As for new material, I consciously tried to remember the new questions so I could post them here. But I just can’t. I’m sorry! I’ll keep trying to refresh my memory. If someone starts, I could definitely help fill in the details.

Reply

338 BfusionNo Gravatar July 6, 2012 at 6:52 am

CONGRATS! Job well done. I hope you are enjoying the moment!

Reply

339 kneedragNo Gravatar July 6, 2012 at 10:49 am

Thanks for the great post. I’m actually taking this in about two weeks (at GSU). I am also using PLI’s course, my biggest question is about the actual test software and how it compares to Patware or the other online simulators; I assume they are pretty close?

I seem to get about 1-2 questions per session that I find as clear odd balls (don’t know right away where to find answer in MPEP) so it was good to hear your thoughts on the searching.

I’ve heard about bfusions’s outline in other comments but never actually found it, don’t suppose you could post a link?

340 PatentGrrlNo Gravatar July 11, 2012 at 9:59 pm

Thanks, Bfusion!

kneedrag, I didn’t worry about “simulating” the exam. For reference, I did go to the Georgia bar exam site the day before to get a feel for the place, but I never did a realistic simulation of the state bar exam, since I don’t really find that necessary. It takes a lot to break my concentration once I get my ear plugs in. I did do the tutorial on this website http://www.uspto.gov/ip/boards/oed/exam/index.jsp, but the actual exam felt much different. Finally, patware seemed different to me as well. I can’t really pin point why.

Also, I hope you found the outline of new material on bfusion’s gmail account.

As for searching the MPEP, I should add more details about my individual test taking skills. I finished the MBE an hour or more early both sessions. I’ve always finished multiple choice early on every exam I’ve taken and I never go back because I find that I change right answers to wrong answers far more often than I correct answers. Thus, I expected to have ample time to look everything up in the MPEP, and I did. Also, keep in mind that I spent six solid days looking up every single answer to the test questions, mostly as a method of making myself get faster at searching the MPEP. In practice, I used this website for the MPEP: http://www.uspto.gov/web/offices/pac/mpep/index.htm. The exam is different in that you have a drop down list of the sections, but that didn’t trip me up.

341 tappylilyiNo Gravatar August 5, 2012 at 2:47 pm

Hey PatentGrrl,

Will you be willing to sell the outline that came with your 2012 PLI study materials? Let me know. You can email me at rachelthwu@gmail.com.

Thanks!

Reply

342 PotentialNo Gravatar July 6, 2012 at 1:31 pm

Bfusion’s material and email address/pw:

mypatentbarr@gmail.com
password: usptoposita

Reply

343 LucasNo Gravatar August 25, 2012 at 6:38 pm

Hey Potential,
Thanks for the information. I tried to log in, but it shows that the password was changed. Could you update the information?

Thanks

Reply

344 George BisharaNo Gravatar July 7, 2012 at 12:12 pm

What is the link for Exam registeration form?

Reply

345 PatentGrrlNo Gravatar July 11, 2012 at 10:02 pm
346 kneedragNo Gravatar July 8, 2012 at 11:52 am

I have a question to those of you that have taken the exam; how close is the software to the simulators like Patware and Cat prep? Will you have multiple windows open (meaning the text, and the MPEP) that you can ALT-TAB between or do you just have one open at a time? Its certainly easier when I have both and am comparing language – which has been possible for printed tests and the patware tests I’ve done, but I want to get out of the habit if thats not possible on the real test.

Thanks

Reply

347 BfusionNo Gravatar July 8, 2012 at 4:41 pm

It’s not possible to have multiple windows open at the same time. This also includes chapters of the MPEP.

Remember, you cannot “Search” the entire MPEP (meaning, you cannot enter a term and have the software search the entire MPEP). You have to select a chapter from a drop down menu. This opens a window of that chapter, and THEN you can search.

Don’t forget people though…you CAN open the index (of the MPEP) and search here first.

Reply

348 kneedragNo Gravatar July 9, 2012 at 12:48 pm

Its an either/or situation then? So if I’m flipping back and forth – it will be closing the MPEP chapter each time and starting me at the beginning when I re-open the chapter?

When I do it in Patware it opens the PDFs in a second window, better practice would be to actually close the MPEP when I want to look back at the question and re-open each time’?

Thanks

349 BfusionNo Gravatar July 9, 2012 at 8:47 pm

Actually…

I found the software on the exam itself to be EASIER to use….for some reason. I wouldn’t worry TOO much about mimicking “real” exam scenarios…at this point it’s more important to learn “HOW” to search using keywords, or using the appendixes or index first, and then searching the specific chapter.

But yes, you can have either the test up, or one window of search.

350 LeighaNo Gravatar August 13, 2012 at 7:19 pm

I used the CatPrep simulator and the real one for the test is nicer. For the real test, access to the MPEP and other relevant documents is on a tab at the top of the test window. The reader in the real test does not use any keyboard shortcuts so when you practice, practice getting around using your mouse only. It’s possible to minimize the MPEP document and ‘squeeze’ it to fit on the screen so you can read some of the answers to the questions while the MPEP is also open.

Reply

351 mecheng88No Gravatar July 12, 2012 at 4:58 pm

Hello all. I just passed the exam today first try by studying the program from PatBar. I have the entire kit which can be found here: http://www.patbar.com/patent-bar.shtml
The study books are in great condition with no highlighting/marking/tearing. I found the books, lectures, and practice tests very helpful and easy to understand. If you are interested please email me at nathan.mutter@gmail.com. Thanks and good luck to you all!

Reply

352 kneedragNo Gravatar July 19, 2012 at 3:54 pm

Hi All,

Took the bar today and received my preliminary pass (first attempt); since I already had a solid 4 days of drinking lined up (it was going to work either way) I thought I’d post a few thoughts I had going in to the test that can hopefully be helpful to some in the future.

PREP – I took the PLI live course (last week). As such I got the PLI home study materials and spent the last two months going through all of their materials and lectures and then getting in to practice tests. Last week I took the live course in Boston and today received my preliminary pass. I read through most of the 68 questions on here and did all of the ’02 and ’03 exams in Patware + a few others on paper + the PLI practice questions.

GENERAL THOUGHTS – First – I think I did the course backwards and it was PERFECT. Getting that dedicated live time right before the exam helped focus a lot of concepts in my head that are hard to pull together on your own. All in all I think the price paid for the PLI course was WELL worth it.

A few things of note:
1) I looked up almost EVERY question. Even ones that I was sure on; I looked in the MPEP to confirm. I think this was important; with some many details separated by subtle differences, this can make the difference.
2) Prior to the test I did two things; the first was creating a pacing reminder; which I wrote across the top of my scratch paper 10/20/30etc and then the times I should be at (in decreasing increments from 3:00hours) so I could make sure I didn’t goof up. It helped. The second was writing down 1-50. My strategy for marking and reviewing questions was this
-anything I realized I wasn’t finding the answer in the MPEP right away; I skipped and left blank
– any claim counting questions (or really long questions that didn’t have an obvious call) I skipped and left blank
– any question I answered but wanted to double check I used the built in “MARK” function
– any question I answered and verified at least half of the answers in the MPEP I crossed off on my scratch paper (with a single line)
– any question I answered and verified ALL of the answers in the MPEP I crossed off completely on my scratch paper (with an X)

After I got to 50 I went back and did Unanswered, Marked, Other questions (in that order).

COMMENTS ON PREP

GET COMFORTABLE SEARCHING – You can, why not? Seriously, this is a no brainer.

DO NOT MEMORIZE QUESTIONS FROM HERE OR OLD TESTS – If you have done much review you have invariably gotten to the point that you’ve answered a question, looked at the answer (which said you were wrong) and then nearly went crazy. Well, remember tests from 2003 and comments from 2008 aren’t using the same MPEP as you are; UNDERSTAND why answers are right or wrong and be able to recognize when YOU are right and the ANSWER key is wrong. This is important.

I had lots of questions that I recognized; there were a few that were verbatim; but many that had slightly changed calls or answers that completely changed the questions. If I just was looking for an answer I can think of at least two that I had that I would have gotten wrong.

That reminds me – WHEN READING THIS SITE – I always started at the bottom of the comments (the newest ones) and worked up until I felt I understood what was being discussed. There is QUITE A BIT of misquoted or outdated information on here; you have to take everything with a grain of salt!

PROMETRIC

The prometric center was great. One thing I realized this morning that you may want to do is that you can actually take a “test run” at your center and go through the check in and take a sample exam. If you’re nervous it may be worth it just to take one more thing off your mind.

The software was good which was my biggest concern. Given my desire to search; I was worried a clunky interface may slow me down. It was very close to PATWARE (from PLI).

I was able to see both the question and MPEP at the same time. When I closed the MPEP and stayed on the same question; it would open up back where I was. Each new question you start at zero.

One thing that some computer savy folks may find annoying is that you can’t use “CTRL-F” or hit Enter or F3 to find next; I had to click the button each time. I think I even did it on the last question showing how hard it is to break habits.

WRAP UP

All in all I think this site certainly helped me; esp. some of the big frequent posters like Bfusion and Bigbadvoodoodaddy.

I had some weird questions in the AM (which I thought was harder), I can only assume they were betas.

I know I can’t remember all the questions I had, but I thought this may be valuable (since I didn’t personally get much out of the recitation of questions).

Good luck – BE CONFIDENT! You’ll get it!

Reply

353 sibaNo Gravatar July 19, 2012 at 8:14 pm

I would like to take the exam around August 15th at PTO, VA. Is it hard to get the appointment date fixed for the same in the weekday? I am still not sure whether I will be ready by 15th. Thank you for your suggestion(s)!

Reply

354 kneedragNo Gravatar July 23, 2012 at 4:20 pm

You can schedule the exam with Prometric once you have recieved your approval from the PTO. It is very easy. You can cancel or reschedule any time up to 48 hours prior to the test.

Be careful; the test is set to change October 2nd – lots of people are trying to get in before that date so seats are filling up fast and you won’t be able to retake prior to the test change if you don’t get it the first time. Be sure you’re ready.

Good luck.

Reply

355 sibaNo Gravatar July 24, 2012 at 9:00 am

I need to take my patent bar exam around in a few weeks. Is there any site for review on prometric test centers? Or we can take at any center based on the availability of date?

Thank you

Reply

356 Joker16No Gravatar July 25, 2012 at 9:10 am

Hey gang, took the exam yesterday and got a 67%. I was wondering if anyone here knows what happens next? I know in two weeks or so I will get the official letter from the USPTO saying I didn’t pass. How long til I can take the exam again? I’ve heard 30 days and 60 days. And is that measured from when you took the exam or when they send you the letter? I’d really like to take the exam again before the October 2 changes.

Reply

357 UpnorthguyNo Gravatar July 31, 2012 at 10:29 pm

See the registration bulletin, section XI(B):
“An unsuccessful applicant, after receiving written notice of failing the examination, may reapply to again take the examination. Such an applicant may reapply for admission to the examination upon receiving notice of failure from OED, but must wait 30 days after the date of the last examination before retaking the examination. See 37 CFR § 11.7(b)(1)(ii).”

Reply

358 rossco993No Gravatar August 7, 2012 at 6:31 pm

Just passed today 8/07/12. First try. Bad news though, almost no repeats. There were maybe 10 that seemed familair. A few verbatim from this site and the rest variations. And I memorized all of the repeats and exam questions. I basically just had to look every question up in the MPEP. Thats the key. I was hoping to rely on repeats but I dont recommend it now. I used the PLI 2006 DVD lectures that I got from a friend of mine. Went through them in a week or so. Then I went through the questions on this site for about a week. Thats all there is to it. The rest you just have to look up. Type keywords into the subject matter index. It will take you to the right place. Lots of PCT, Appeals, Obviousness. I think one of the only repeats i got was the 147 claim counting question. Dont get too crazy with studying. Maybe 1 month max of part time study. Its mostly just look up. I saw some people on here saying they studied for like 250 hours. Way overboard. I figured Id get the exam out of the way before the major changes. Good luck guys.

Reply

359 LeighaNo Gravatar August 13, 2012 at 7:04 pm

I passed the test yesterday. In the first half of the test I got about 5 repeat questions. In the second half of the test I got 15-20 repeats. Some of the repeated questions were: Prior art date of a declassified reference, petition to make special (65 years old), Federal court decisions binding the Office, the provisional double patenting question where you have to look at the flow chart in Ch 8, Laurel, Abbot and Costello, the cell phone, a few questions about express mail, nonpublication request (45 days), the multiple dependent claim counting question (147), cancelled matter as prior art, 5 steps to cross the road, IDS after Notice of Allowance where inventor knew all along, several of the appeal questions (if the applicant lapses out their time then what happens), supplemental oath as an amendment (in a reissue), and probably one or two others that I’m forgetting right now. This website was invaluable in achieving my goal of passing this test. Here are some of the things I did that I found helpful in my preparations:

1) I probably spent about 300 hours over about 4 months preparing. I started off with 1-2 hours per day, but things didn’t really start sticking for me until I did 4-6 hours per day. By the time I took the test I felt pretty prepared.
2) Check out each section of this website. There is SO MUCH useful information here!
3) Several of my friends who have taken this test before me told me to take old tests. This was great advice. I read here that you should re-take the tests until you can get 90% or better on them, which is also pretty good advice. I took all of the 2002 and 2003 tests, most of them at least once, and it was an excellent preview of what to expect. It was useful because you get to answer in a safe environment some of the questions you’ll actually encounter on the test and because it gives you an idea of how the USPTO formulates its test questions.
4) In the words of another friend, “Become a MPEP search Jedi”. The CatPrep website has a version of Adobe Acrobat 5, the version which most closely mimics the searchable MPEP in the test. Download it and use it for practice when you take the tests. If you can easily find what you need, you don’t have to have the text of the MPEP committed to memory.
5) Practice taking the test computerized. You can do this for free on the CatPrep website, but you have to pay them money if you want them to score it for you. I did, I thought it was worth the small fee. However, if you make an answer grid for taking the test (which I highly recommend doing), providing the test is from 2002 or 2003, the corresponding answer key and explanations document is available for free on the USPTO website. You can easily correct the test yourself and you’ll learn a lot by doing it.
6) I used the Bullseye study materials. They are very affordable and very helpful. The full package includes 12 tests and the answer key and explanations for each test as well as a test that is 60 known repeat questions and many other useful things. It was the best money I spent on preparing for the patent bar.
7) On the day of the test, I spent my 1 hour break between sections at my car eating the lunch I packed with me and reviewing my test prep materials. I saw some questions that I answered correctly in the first part (confidence building!) and refreshed some things that came up in the second half of the test.

Thank you to everyone that makes this website such a valuable resource and good luck to future test takers!

Reply

360 MichaelNo Gravatar August 13, 2012 at 11:12 pm

Hey Leigha. Thanks for your comment. Could I ask you (or anybody else) how to get better at searching the MPEP?

I’m using the PLI version, downloaded by chapter section, on a Mac. Whenever I search for terms etc, I get tons of matches. How do you know/strategize where to jump to without memorizing the MPEP??

Thanks so much.

Reply

361 LeighaNo Gravatar August 14, 2012 at 1:40 am

Hi Michael,

Here are a few of my thoughts on tips for good searching:

– Know which chapter to search (anything about examination will be in Chapter 7 or 21, that includes pretty much any question about 102, 103, 101, and 112 rejections, things about the application contents will be in Chapter 6, things about appeal will be in Chapter 12, etc)
– Search the Index for general concepts if you can’t think of which chapter to start in. Also the Index includes chapters where 37 CFR x.xx items are referenced. Search the Index for 1.131, 1.63, or 1.48 and see what you find.
– Use info from the answers and from the questions in your searches. Two words are better than one unless the one word is very unique. If the question is about if a claim can be considered ‘in a vacuum’, searching Chapter 7 for the word ‘vacuum’ will probably give you the exact thing you’re looking for. If you’re wondering about the use of Trademarks in a claim, a search for the word “Trademark’ is pretty useless as the word Trademark is the T in USPTO and appears pretty much everywhere in the MPEP. The answers will likely have more clarifying info and most of the answers are verbatim from the MPEP.

The answer key and explanations from the old tests tell you exactly where the answers are cited for answering each question. Becoming more familiar with where information is cited will help you make more targeted searches. If you know that what you need in somewhere in 608.01, go to Chapter 6, click 608.01 on the Bookmark tab and search for your specific words from there – the search will begin from the section that is selected, not from the beginning of the chapter.

Some of it is recognizing what the USPTO ‘likes to say’. The more times you hear “commonly owned”, “unobvious difference”, or “through error and without deceptive intent” you can learn the types of things that the USPTO ‘says’ in the MPEP and your searches become far more targeted.

I hope these tips help you get better search results.

362 vibhaNo Gravatar August 15, 2012 at 9:38 am

Hi guys,
I am taking my exam on October 26th , little scared abt the recent change , any idea how to approach for recent changes and what changes to expect in exam.

Reply

363 kneedragNo Gravatar August 19, 2012 at 11:55 am

Test is supposed to change Oct 2nd, I would HIGHLY suggest taking it before, or waiting much later. There is a LOT of new information.

Another option would be to pay to take a PLI or similar course as they have access to a lot more info a lot sooner than it will filter out to the rest of us. AFAIK they already have their supplemental materials put mostly together and they are huge.

Reply

364 FoojNo Gravatar September 26, 2012 at 1:41 am

I’ve heard an interesting strategy for taking the exam post Oct 2 — that is to take it as quickly as possible. Here’s the rationale: The USPTO runs new questions as non-scoring beta questions during actual exams first. As such, exams taken within soon after Oct 2. will all new material questions as beta questions, and very few will count towards your score. If one waits to take the exam, an increasing number of new material questions will start counting towards your actual exam score.

Thoughts on this strategy?

Reply

365 Mr. AzulNo Gravatar September 26, 2012 at 2:15 pm

Fooj;

It may well be that the OED does slowly test the new questions on the latest AIA over time. However, there is trasparency with this test, the OED, and how its set up with Prometrics to be able to make any viable plans other than to simply take it until they evetually pass. I would note that there would be political fall out if suddenly no one can pass the exam, at least with the small but powerful DC/VA based IP lawfirms who depend on young lawyers to be able to pass it to become associates.

Post AIA the new reality will be that NO ONE CAN PASS THE EXAM ON THE FIRST TRY and the process to take this exam will become one where its accepted in the patent bar that one has to take it at least five or ten times, and therefore the question pool becomes known and defined one exam veteran at a time. Good time to buy stock in Thomson, the parent of Prometrics! These questions can not be protected by copyright as they are developed by the USPTO staff (government publication) and are outside of copyright proections as a government work product. The test preperation courses, such as PLI and PRG and others know this (the old exams are not subject to copyright) and there will be a strong effort to decode and define the newer AIA questions through their courses. Right NOW my advice would be to not waste thousands on either PLI and PRG for the next YEAR or longer, let the AIA have its impact on the exam, and instead use the two thousand you save NOT TAKING A COURSE to pay for the five Prometric test sessions everyone will have to use to pass this new mutant vampire zombie maker of an exam. Those that take the exam in the next two years are swimming with sharks in deep water, and neither PLI or PRG or anyone else has a clue on what is being tested and what is and is not beta versus actual questioning.

Additionally, the value of presently having a reg number just went up measurably since the threshold for passing the exam has at least doubled, so folks that gamed this exam rather than actually knowing the substance of the patent law are out there and doing real damage to the public. That might have an impact with the USPTO. My take is that given what we all expect to be utter chaos resulting from the huge changes there will be a reason for the OED, after they watch the chaos ensue, to revamp the exam to take it back the way it was with a claims drafting/written portion and a Q&A section, with a huge increase in exam costs to the applicant.

366 VenuSareenNo Gravatar August 24, 2012 at 4:16 pm

What the ?

Reply

367 patentgirlNo Gravatar August 26, 2012 at 1:06 pm

Does anyone know the new password for bfusion’s gmail account? it said it was changed nine days ago. Thanks!

Reply

368 ceejNo Gravatar August 26, 2012 at 11:23 pm

He posted this in Exam Questions and Concepts a couple of days ago:
__________________________________________
Thanks for updating the post roll. It seems my post back on July 26, 2012 is now long buried and so people aren’t seeing that yes, my materials now cost a little.

This was the post:

Hi All,

If you are interested in the spreadsheet, you can email me directly:

bfusion.uspto@gmail.com

(I’m currently asking $85 for my materials….which includes the spreadsheet, and two word documents (one is ~100 compiled questions of new material being asked on the exam, and one is ~10 pages of concepts to know for the exam being tested).

Best!

~Bfusion
__________________________________

I emailed him very early this morning and have not heard back from him yet.

Reply

369 LisaLNo Gravatar August 27, 2012 at 2:45 pm

Where can I find mambo5’s exams?

Reply

370 VenuSareenNo Gravatar August 29, 2012 at 3:38 pm

I’ve also been looking! Searched Mambo and Mambo5, did not come up with anything.

Reply

371 JTKNo Gravatar September 15, 2012 at 1:10 pm

Agreed, where can i find this compilation of old exams?

372 timmyNo Gravatar August 27, 2012 at 8:05 pm

This is crazy. Just turn him into the California bar.

Reply

373 RobinNo Gravatar September 2, 2012 at 1:42 pm

Hi! Anybody can tell me what is the typical time for OTD to approve my application (category A)? And in chicago or nearby area, what is the current reservable date?

Obviously I am trying to get tested before the new law been tested. Oct 3rd, correct? Any idea how different it could be? And is there any review course that already started to cover that? Thanks very one!

Reply

374 tappylilyiNo Gravatar September 7, 2012 at 5:21 pm

You can actually check on the Prometric website to check for availabilities in your area without getting a eligibility code from USPTO. I just scheduled mine and I looked through seven test sites (in NYC area) before I found one that had an availability before 10/2 (the date that the test is supposed to change). PLI apparently has the new patent bar materials, but I don’t know that for sure. If you didn’t send in your application yet, I am fairly sure you won’t get your eligibility code in time to schedule your exam before 10/2. It takes the USPTO at least 3 weeks to issue you an eligibility code. I sent in my application (and I was reapplying to take the patent bar, so they have all my information already) in early August and just got my eligibility code.

Reply

375 Mr. AzulNo Gravatar September 7, 2012 at 7:56 pm

The post October 2nd materials are posted on the OED website. Here is the link.

http://www.uspto.gov/ip/boards/oed/exam/aia_regexamsourcematerial2012sep04.jsp
I am going to wait until next year to take my exam, since its too late to get a test date before October 2nd, and I am of the opinion no one really knows what the OED will draft into the exam. Anyone who takes the exam after October 2nd is at best an involuntary beta test subject. I for one do not want to be part of that experiment. I prefer to wait until the experts at PLI or PRG get the necessary feedback from their students and adopt the preperation materials (at those students expense!) for the rest of us. Additionally, I am just a regular student and not an expert like so many on this blog, so I would rather someone else become the statistical base population during the next year for the USPTO to use as the post AIA testing baseline analysis.

GOOD LUCK!

376 patentgirlNo Gravatar September 11, 2012 at 9:42 am

I took the exam yesterday (preliminary pass), and I wanted to post regarding the questions. There were not many repeats, and very few KSR/Bilski. But for the repeats, there were the 2 Potters, Costa Rica, moon dust, and ABCDE. There were a bunch of questions on access, signature requirements, and facsimile. Good luck to all those studying.

Reply

377 tappylilyiNo Gravatar September 12, 2012 at 12:04 pm

Hi patentgirl, how long did you study for? I’m taking my exam on 9/25/12, and I would estimate that I put in around 150 to 200 hours over the last four months. Taking the exam for the second time, and I’m pretty nervous about it.

Reply

378 kneedragNo Gravatar September 17, 2012 at 6:50 pm

I just came back to post that I think I’ve forgotten everything I learned….. and I still haven’t even gotten my reg # yet! Taking their sweet time!

Reply

379 ShinNo Gravatar September 17, 2012 at 7:48 pm

I am registered to take the examination on Sept. 20. I noticed that OED had updated their announcement last week stating that the Sept. 16 changes had been implemented, even though the source materials still refer to “on or about” Oct. 2. Paranoid as I am, I called OED, and they stated that the changes would be reflected on the exam on Sept. 25…just a heads up.

Reply

380 tappylilyiNo Gravatar September 17, 2012 at 7:55 pm

WAIT…so they will be implementing the changes starting 9/25/2, not 10/2?? I am going to call OED tomorrow as well to confirm, because I’m taking the test on 9/25, and the registration information I got said changes will take place on or about 10/2.

Reply

381 GeppiegirlNo Gravatar September 17, 2012 at 10:59 pm

I hope that this isn’t true – but with my luck it is. I’m also signed up to take it 9/25, should have taken that slot one day earlier apparently. Will be very interested to know what you find out, tappylilyi, because my registration information said the same thing re: 10/2.

382 tappylilyiNo Gravatar September 18, 2012 at 10:11 am

OED said that the last day to take the “old” exam before the changes are reflected is 9/25, and there is a blackout period between 9/25 and 10/3 in which the exam will be updated with the new information. The changes will be reflected on the exam starting 10/3.

Reply

383 GeppiegirlNo Gravatar September 18, 2012 at 10:28 am

Thank you for the prompt follow-up Tappylilyi – and best of luck to you on 9/25!

384 NaanaNo Gravatar September 17, 2012 at 7:55 pm

Thank you very much Shin! I am going to take the exam on 23rd!

Reply

385 MJNo Gravatar September 18, 2012 at 11:35 am

Does anyone have Mambo5’s tests or Bfusion’s tests that they can share with everyone?

Reply

386 tappylilyiNo Gravatar September 20, 2012 at 2:11 pm

MJ,

Here’s the link to Mambo5’s test. http://d.pr/f/QuyD

If you want any of Bfusion’s materials, you have to email him at bfusion.uspto@gmail.com. He’s very helpful.

Reply

387 CGNo Gravatar September 20, 2012 at 12:50 pm

Has anyone taken the exam starting at 1 pm? I just reserved one for 10/11 and the exam time is 1 pm. And the exam center confirmed the time with me. I’ve never seen anyone taking it that later.

Reply

388 MwhitNo Gravatar September 20, 2012 at 3:11 pm

I am taking it at 12:30 tomorrow…..i thought the same thing.

Reply

389 ShinNo Gravatar September 20, 2012 at 5:22 pm

Sorry for the confusion earlier for those taking it Sept. 25. I must have misinterpreted OED’s comment to me that the changes would take effect Sept. 25. Good to hear that the first day the changes take effect is Oct. 3! I just finished the exam and got a preliminary pass! Good luck to everyone.

Reply

390 Mr. MPEPNo Gravatar September 20, 2012 at 6:00 pm

Congrats on passing Shin. Can you list some questions that you can remember for everyone? Thanks!

Reply

391 MaxNo Gravatar September 23, 2012 at 2:14 am

Hey all. I have a mac, and have no idea how searching on acrobat is going to be.

Can you search terms/phrases by putting “——–” around them? Any other searching tactics?

Thanks!

Reply

392 fluidNo Gravatar September 23, 2012 at 8:51 am

I passed 9/23/2012 and had 15-25 verbatim repeats. I used a PLI course from 2006 and then did the 2003 and mambo5 past exams twice. This website has been essential!

Reply

393 EmilyPNo Gravatar September 23, 2012 at 11:26 am

Related searching question to above: Can you search a few words in a row that are in proximity to each other (like, same page), but not strictly in sequence?

Reply

394 HSUNo Gravatar September 25, 2012 at 6:11 pm

Just passed today on my second try. Such a relief to have passed it before October. Had easily 30-40 repeats or variations. With BFusion’s material I would say there really wasn’t anything out of left field. Some new questions though:

PCT:
IA filed designating US, accorded filing date by US/RO properly. Inventors then decided to add one more inventor, and filed the correction w/in 20(?) months of priority date. No demand for Chapter II was filed. Now they want to enter US national stage. Question is what must they file.
A. Only basic national fee w/in 30 months of priority date
B. something
C. Only basic national fee and a copy of IA if it hasn’t been transmitted from IB to US/RO w/in 20 months
D. Only basic national fee and a copy of IA if it hasn’t been transmitted from IB to US/RO w/in 36 months
E. Something.
Went with C for lack of a better answer. Nowhere did it say whether the copy of IA was submitted or not. Also, not sure how the correction of inventorship is relevant in this question. Would be an easy one if it’s “30 months” in C or D.

KSR:
Which is true: when the examiner fails to use one of the 7 factors in KSR to make a rejection, the applicant may point this out and the examiner should “modify or withdraw” if he/she thinks is right. I couldn’t find the exact quote in the KSR Guideline. No where did it say whether the 7 factors are exhaustive. I know you should still consider Graham, but Graham is not a rationale for making 103 rejections. I went with this choice b/c it sounds reasonable.
Also, one choice talked abt examiner must give reasoning under KSR. Thought it’s right.

Bilski:
A business method claim that recites abstract idea. To overcome 103 rejection, applicant can amend the claim by adding machine element into claim so as to make it an application of the abstract idea.

Double patenting:
Which can be subject to double patenting or provisional double patenting rejection:
Application and patent, not patentably distinct, 1 common inventor, provisional double patenting rejection based on the patent;
Application and patent, not patentably distinct, common assignee, provisional double patenting rejection based on the patent;
2 pending applications, not patentably distinct, 1 common inventor, provisional double patenting rejection based on the first applicaiton;
something…
went through 804 but they all seem to be OK, just picked one randomly.

Requirements of information:
All is true except:
A. Prior to first office action, examiner may ask applicant to provide info is any search has been performed on the application;
B. Prior to first office action, examiner may ask applicant to provide info as to which commercial database was used for drafting the application;
C. Requirements for information should be made when examiner thinks is necessary for examination;
D. A petition to withdraw requirements for information is a reply to 1.105 requirement, and it tolls the time for reply.
E. All the above is not in accordance w/ PTO rules.
Chose B per 704.14(c).

Accelerated examination:
which of the following is true: must be filed to make special, age is not considered, something, something, must have ADS. Chose 1-5. Not sure about the ones in the middle, couldn’t find exact quote under 708.02(a).

Restriction between combination and subcombination:
When is proper to make restriction between the two: Look to 806.05(c) 2nd paragraph. Answer choice not exact quote though regarding “the subcombination can be shown to have utility either by itself”, it added something else, which is little confusing.

A variation on trade secret: stuff material to patentability should not be presented (wrong answer?).

Agent in California, pacific time 10 PM of the last day of reply to OA. Fax is available, but transmission would take hours. what’s his option? RCE, abandon, fax, priority mail under 1.8, express mail under 1.10. Chose express mail, thought it’d give a local filing date before midnight.

Application showed applicant had possession at time of filing, but no disclosure of how to make the invention; prior art reference same situation: non-enabling but anticipates the invention. Question is what rejection? Chose “only reject under 112 first paragraph for non-enabling.”

Know that “market force” can be used for 103 analysis.

Know supplemental O/D is considered amendment only during a reissue after allowance.

New standard of inter partes reexam: reasonable likelihood of at least one claim is patentable. …

Can’t use certificate of correction is all inventors are not in agreement, but an assignee of entire interest can request correction of inventorship over the objection among inventors.

A continuation can be used to correction inventorship.

When making corrections in an application, all inventors must be identified, but it’s OK for one inventor to sign if only his claim is involved.

Prior assignment covers divisional and continuation, but NOT CIP or substitute.

You can appeal when any claim is rejected twice, doesn’t have to be the exact same claims.

Non-Provisional app submitted w/o a claim, what to do at one year deadline? Chose convert non-provisional to provisional, b/c provisional doesn’t require a claim.

What should not be considered for utility: live and inanimate.

When is IDS a proper submission w/ RCE? Answer: after allowance. Got it wrong and chose “none of the above”. Read this question during the 1-hour break on Bfusion’s material. Cold sweat. Thought I was gonna fail.

Some of the repeats:
One hundred forty-seven;
Japan – 45 days;
Mexican PCT in Spanish;
131 antedates;
Velcro (maintain rejection b/c no structure no described);
Good faith error in claiming small entity status;
Reply to OA w/in 2 months, AA w/in 3 months (SSP still counts);
Electric fan and lamp;
When piecemeal examination is not proper;
Can’t use 103(c) common ownership exception to overcome a 102(e) rejection;
RCE is not a new application;
No need to file English translation when claiming foreign priority(?);
Canceled matter can’t be used for 102(e), but can be for 102(a);
PCT designating US only, no request for publication (answer IB won’t publish but upon national entry will publish by US);
Third party submission;
RCE filed w/o fee after appeal = treated as withdrawal of appeal, back to examiner to abandon to rejected claim and allow the allowed ones (one answer choice says examiner doesn’t have to do anything, claims abandoned automatically, but it’s not right, examiner must take the action after reopen of prosecution);
When district court found claim valid = non-binding, PTO can still find it invalid;

There were numerous old exam questions. So you should really have them cold. At least 24 questions were on Bfusion’s material. Really made whole lot difference. Thank you to all on this website. Can’t do it without you guys. Good luck to everyone!

Reply

395 tappylilyiNo Gravatar September 26, 2012 at 9:48 am

HSU, sounds like we had the same test. Just some comments: 1) ADS doesn’t need to be filed in an accelerated exam. 2) With the PCT question, I went with basic filing fee by 30 months to enter the national stage, because if I remembered correctly, the USPTO was the RO in that question, so it already had a copy of the IA. To enter national stage, all you need is a copy of the IA and money. Everything else can be filed later. 3) provisional double patenting, must have the same assignees or at least one common owner. I was confused by this one, because there were answers with app and patent, app and published app. I picked app and published app, because that’s what I found to be the right answer in 700. Between app and patent with a common assignee or inventor, I THINK it had to be a obvious rejection (nonstatutory) to be a provisional double patenting rejection, and all answers said patentably indistinct inventions.

Reply

396 HSUNo Gravatar September 26, 2012 at 11:17 am

tappylilyi, re question 2), I think your answer makes sense to me. Just remembered another PCT question that had to do with using fax to send an authorization to charge fees, there was a previous filing (missing the request?), question is when is the filing date, the previous one or when the fax is sent. Not sure about this one, but did find in 1800 fax can be used for payment authorization. Had this one in the morning session.

397 JPizzleNo Gravatar September 25, 2012 at 7:46 pm

I passed today.

I watched the PLI lectures and did most of their material. Sept. 25 was the first day that the PTO said I could take the exam and I scheduled it (after wandering onto this website to find out that there was a blackout with no test dates until after the exam changes). So without doing any practice exams until about 3 days ago I took the 2003 tests (april and october) and memorized what people were reporting on this site. I know that I would not have passed without the contributors here.

Questions:

PCT Question with added inventor (named “Riggs” on the exam) that HSU mentioned earlier today.

I put answer A: that only the fee needs to be paid. I said this because since the US was the RO I the MPEP said that a copy of the IA didn’t need to be sent.

KSR Question that HSU mentions: I had it too but don’t remember much. I do remember choosing something to the effect that the examiner has to have a clear rationale based on KSR???

It looks like I had pretty much the same exam as HSU, who commented previously. So, I’ll add where I can.

Bilski Question that HSU mentions: the question recites a claim that reads very similarly to the actual Bislki claim (no mention of any machine/computer in it). I also chose the answer that said something like amending the claim to add machine/transformation.

Double Patenting Question that HSU mentions: He is correct but there was one more answer choice (the one that I picked, but am not completely sure about):
Application and patent, not patentably distinct, common inventive entity, provisional double patenting rejection based on the patent;

Accelerted Examination: had this but don’t remember much. This was in the MPEP, not the supplement.

Restriction: Sub/Combination: the answers were right out of the MPEP

Chemical application where applicant claimed something. It wasn’t enabled. Prior art had it, but a PHOSITA couldn’t make it w/o undue experimentation (I think this was a previous exam question). How should examiner reject? one answer had “enablement and written description are separate and distinct”.

Inter party re-exam: in supplement. Easy answer – must have “reasonable liklihood…”

Provisional A filed T1 by A. Provisional B filed T2 by A, B, C. NonProvisional C filed at T3 which claims benefit of A and B. NonProvisional D filed at T4 (more than a year after both A and B) which claims benefit of both A and B. There is a prior art reference that anticipates all claims. And it looks like the claims in A and B are the same, and both disclosures support the claims. There is a rejection. What can applicant do???? Something like that – hopefully others can flesh out. It basically had to do with asking about claiming priority to provisionals and maybe swearing back.

In addition to what HSU mentions, there was:

Laurel and Hardy
moondust
broadening reissue
a few on appeals that asked what should examiner do when the file is returned to him when, for example, claims 1-3 are rejected, 4 and 5 are objected to but otherwise allowable, and 6-10 are objected to.
laminate claim
PCT “ccar”

Thanks to everyone on this site! This is a great resource.

Reply

398 Mr. AzulNo Gravatar September 26, 2012 at 2:39 pm

Correction to my earlier post; There is NO transparency with this test, nor the OED on how its set up and managed, as evidenced by their lack of distribution of old questions after 2003, and the seemingly evergreen contract with Prometrics. There does not appear to be any OED accountability for this exam and the way its adminstrated. My only suggestion post AIA is to to simply take it until you pass it. The USPTO appears to be above congressional or judicial oversight at least as to the test. Real patent reform would have included making this exam equitable and open for review by those that have take it and the created a due process review rulemaking on the exam as a requirement for USPTO practice, but I am sure those with a stake in the way this exam is set up intervened. A lawyer in good standing can appear before the Supreme Court, but not speak with an examiner absent passing this arbitrary and misguided exam.

Reply

399 MalissaNo Gravatar September 30, 2012 at 9:08 pm

Hi everyone, I just wanted to confirm a few things before I send in my registration application. 1. I am a category A applicant and want to make sure that I do not need descriptions of the courses I’ve taken. 2. Is it a requirement that I submit my application by certified mail? 3. Do I need to also send in my law school transcripts? (I did not think so from reading the Bulletin, but the application itself asks you to list every institution you’ve attended, so now I’m not sure).

Reply

400 October TakerNo Gravatar October 1, 2012 at 8:35 pm

Can anyone who took the exam in the afternoon tell me if there was anything different about the exam schedule? Do you get an hour for dinner? Or straight through?

Reply

401 Mr. AzulNo Gravatar October 1, 2012 at 8:44 pm

You get an hour break between the two three hour sections no matter what time you start the exam. Best to bring a lunch or dinner with you, as well as a couple of cans of REDBULL!!

Reply

402 brianNo Gravatar October 3, 2012 at 12:27 pm

Does the computer test include:
1. just the MPEP
or
2. the “Source Materials for the Registration Examination” from ustpo (MPEP, AIA, etc)

Reply

403 JasNo Gravatar October 3, 2012 at 5:33 pm

Hi,

If I fail the exam, can I re-take it (with $150 fee to Prometric) in the same 90-day window without re-applying to OED?

Reply

404 HopeNo Gravatar October 3, 2012 at 6:11 pm

Jas, You will need to reapply to OED and I think after a month from your first exam you can retake it again.

Reply

405 JasNo Gravatar October 3, 2012 at 6:13 pm

thanks

406 Lisa M.No Gravatar October 3, 2012 at 7:34 pm

I will do a bigger brain dump soon, but I wanted to briefly let anyone who is taking the “new” test in the next week or so know you do NOT need to panic! I took it today (October 3) and am very pleased to report the new material is constrained to only 10 questions, most if not all of which are easily identifiable due to conspicuous dates (e.g., Sept. 16, 2012, dates in the future, etc.) or give-away terms (e.g., inter partes review). It would appear the PTO has not abandoned their new-questions-must-be-beta-until-we-statistically-analyze-them policy. So, for the first several days or weeks–however long it takes for them to get enough stats to determine what betas go live–it is my belief that you don’t actually have to know ANY new material to pass or even to get a perfect score. The 90 questions that are scored are all old material. Furthermore, if you pay attention to the dates and identify the betas, it appears you actually can safely focus on just the 90 questions that you know will be scored. That actually makes the test EASIER than normal, since you can avoid wasting time on 10 non-scoring questions. Of course, it’s hard to say how long this will hold true (and I obviously can’t swear it’s true since they didn’t state they were beta, so please don’t treat this post gospel!), so I’d be nervous with that strategy too far down the road. If your date is in the immediate future, though, I wouldn’t stress.

It is also my understanding that each question in the PTO database is given a “difficulty score” or something like that. The questions you see on your test–which is unique to you or perhaps a subset of test-takers and not the same one everyone else on the same day takes–are psuedo-randomly selected from that database. I say “psuedo” because they do select in such a manner that your total difficulty score is within a given acceptable “total difficulty” range for the sake of normalization. So, you won’t get all 10s while your buddy gets all 1s. Basically, if that’s all true, what I hope that means is that when the new material goes live, they’ll be balancing out some of the higher difficulty new material with new material gimmes (I had one today that simply related to knowing that the Oath/Dec no longer requires “original and first” statements and citizenship) as well as some easy novelty, obviousness, etc. questions that we all know from the old exams.

To Brian: There is one button that opens up the MPEP and another that opens up the supplemental material (final rules, KSR stuff, etc.). You’ll have everything you need. :)

Reply

407 GeoffNo Gravatar November 8, 2012 at 3:59 pm

Lisa,

Where did you do your brain dump of the questions on the “new” exam? I’m taking the exam 11/28 and would really like some incite on new questions being asked especially with the AIA. Thanks!

Reply

408 RobbyvegasNo Gravatar December 5, 2012 at 4:49 pm

Geoff,

Were you able to collect information on the new AIA questions? I’m scheduled to take the exam on 12/11. Could you pass along any notes that you were able to collect on these?

Thanks,
Robbyvegas

409 gotta get through thisNo Gravatar October 5, 2012 at 1:22 pm

HSU,

You think the repeats you have mentioned will show up in an exam being taken in late November? Thanks for the ample amount of information.

Reply

410 EAGNo Gravatar October 10, 2012 at 12:33 pm

All:
I am selling my PLI 2012 Patent Bar Review Course materials. This includes updated AIA material! Please see the following link:
http://cgi.ebay.com/ws/eBayISAPI.dll?ViewItem&item=150921441993

Thanks!

Reply

411 JasNo Gravatar November 2, 2012 at 11:59 am

Is there a “Recess” button in the Prometric Exam? If I go to bathroom or drink some water/coffee, will the clock stop?

Also, is coffee provided in the exam center?

Reply

412 AnnoniemouseNo Gravatar November 2, 2012 at 9:16 pm

no to both – at least at my exam center.

Reply

413 OliveNo Gravatar November 5, 2012 at 7:56 am

My relatives every time say that I am wasting my time
here at web, but I know I am getting experience everyday by reading such fastidious articles or reviews.

Reply

414 rickNo Gravatar November 5, 2012 at 8:48 am

PLI is completely worthless, dated material from 10 years ago. The Cd sample test questions do not work.

patentbar.com is garbage. Just a monotone Ben Stein guy reading the MPEP chapter by chapter. The CD’s skip or fast torward themselves ahea for no reason. The material is dated from 20 years ago.

Reply

415 OrdinaryskillNo Gravatar November 5, 2012 at 12:25 pm

Rick:
Both PLI and PRG as well as the other patent review courses are totally lost given the recent AIA based questions. The question pool has been rewritten. I suggest reading any of the old PLI or PRG courses for back ground and just plan of taking the real prometrics exam a few times, one time after another, picking up from each session until you make a 70. I agree with you about both PLI and PRG as well as the other lessor bar review courses. They are all too expensive and now are obsolete given the AIA. Rather than paying a few thousand for a new PLI or PRG course, buy a used old one on ebay for a few hundred dollars, read the old exams, and spend the money you would have spent of a new course taking the prometrics exam two-three times.

Just my opinion. You may be a genius. If you are, why are you not already rich and why would you have to take the patent bar to get a better job?

Reply

416 man'neNo Gravatar November 5, 2012 at 3:35 pm

how do you recommend studying? by patentbar.com, do you mean this site?

Reply

417 man'neNo Gravatar November 5, 2012 at 3:33 pm

so, what version of the MPEP is being tested right now? how up to date is the material on this site for the current patent bar exams being administered?

Reply

418 OrdinaryskillNo Gravatar November 5, 2012 at 5:23 pm

Man’ne:

All your questions on what is on the exam can be answered here:

http://www.uspto.gov/ip/boards/oed/exam/aia_regexamsourcematerial2012sep04.jsp

This site is not the same as patentbar.com. That site is for a exam review service. This site is mypatentbar.com, and is a useful forum for discussions regarding the USPTO exam.

My best suggestion is to use an inexpensively obtained PLI or PRG course, (it can be an old course, so long as it covers the basics of 101, 102, 103, 112, and the PCT). Then study the AIA materials linked above to the USPTO’s updated test materials. Save the money you might spend on a new preperation course and budget to pay for what is most likley going to be several runs at the exam at ($240 plus $150) for about $400 eash attempt. I expect very few will be passing the new rewritten exam out now on the first attempt going forward with these new questions.

Good Luck!

Reply

419 CODE REDNo Gravatar November 21, 2012 at 3:08 pm

FYI: The OED is now announcing that the final AIA Phase III materials will begin testing in early April 2013. Strong incentive to take and pass this exam ASAP in the next four months before they rewrite the entire examination to include the rest of the AIA!
Here is the link: http://www.uspto.gov/ip/boards/oed/exam/index.jsp

Reply

420 RobbyvegasNo Gravatar December 5, 2012 at 8:05 pm

Does anyone know how to search the MPEP for a text string (multiple words, e.g. “extension of time”)? I’m using PRG’s Examware right now, but I recall having this problem when I took the test last time – only being able to search for one word at a time. Double and single quotes don’t appear to work.

Thanks,
Rob

Reply

421 TJNo Gravatar December 18, 2012 at 9:06 pm

Just type in the string without quotes. The prometric patent bar pdf viewer will only display results with the entire string. It will not break up the words like newer versions of Acrobat. Do yourself a favor and install and practice with Acrobat Reader 5.0. I could not tell the difference in the search capability.

Reply

422 VVNo Gravatar December 15, 2012 at 12:14 pm

Does anyone know if the MPEP currently being offered during the test is still the 8th revision with notices or has it been updated to the 9th revision?
Thanks!
Vicki

Reply

423 PHRNo Gravatar December 16, 2012 at 3:35 pm

As of last week when I took it the reference material was MPEP 8/8 with the additional supplemental materials.

Reply

424 TakingExamSoonNo Gravatar December 16, 2012 at 10:39 am

I have the same question as VV. Any feedback on MPEP v8r9 or v8r8 at Prometrics. This is important because they include updates to the Statuts per AIA and have incorporated many of the earlier notices, such as KSR, into the MPEP.

Reply

425 TJNo Gravatar December 16, 2012 at 6:53 pm

It still lists v8r8
Goto http://www.uspto.gov/ip/boards/oed/exam/index.jsp
Click “Source Materials” link to watch for updates.

Reply

426 BfusionNo Gravatar December 17, 2012 at 10:43 am

Hi All,

I talked with the USPTO this morning. Prometric (and the test) STILL use ER8 on the exam. That means when studying (especially getting used to searches) you should be using V8ER8 to get used to studying (along with searching all the annoying supplements).

This kinda stinks since it’s a pain to search the supplements, but, as a heads up for you all!

So, be prepared. On exam day (until the USPTO changes over) you’ll encounter version ER8 + Supplementals.

427 TJNo Gravatar December 18, 2012 at 9:03 pm

Adobe Acrobat 5.0’s search is very similar to the patent bar search, as has been stated above. I did not notice a single difference in my searches. You can search for a string without quotes.

When you have the exam open, at the top right there are two buttons, one for the ‘supplemental materials’, one for the ‘MPEP’ (including appendices, etc). Clicking either button opens a pdf viewer in a new window that is not maximized by default, so you can drag it around to view the question underneath. In the top left of this window is a ‘find’ and a ‘find next’ button and a drop down menu with an ‘open’ button next to it. The drop down menu has the names of each chapter and supplemental materials (so you don’t need to memorize the 201#####.pdf names!!!)

Once you have a pdf opened in the MPEP for a question, if you open a supplemental file and then reopen the MPEP, it will return to the same file at the same position. However, it defaults to no open file each time you progress to the next question.

Make sure you’re used to clicking find next instead of hitting enter!!!!! As mentioned above, this can (will?) crash the program, requiring a staff member to re-log you in, taking up up precious time (not sure if the time is counted or not when you’re logged out. I would assume so, since it’s networked in their systems).

At the end, you will have a short survey (<5 multiple choice survey on location and staff). There is an end button, so you could theoretically skip it, but it takes less than a minute.

Be careful when you sign in/out for breaks. I sign quickly, and apparently my signatures did not look the same. I had a staff member tell me that we'd have to go over and verify the signatures when I got back from my restroom break. She didn't hold me to that thankfully.

If you pass, they give you a printout on your way out with an embossed stamp and a signature. I didn't actually look at it until I got home, but it has the same statement that you see on your screen at the end: "Preliminary test results show that you passed the U.S. Patent and Trademark Office Registration Examination for Patent Attorneys and Patent Agents. You will receive your official examination results from the USPTO in due course."

Reply

428 KarmaNo Gravatar December 19, 2012 at 6:15 pm

Does anyone know how to get in touch with the Admin on here?

Reply

429 BfusionNo Gravatar December 19, 2012 at 11:30 pm

Karma,

this site really hasn’t been monitored, or updated, in over ~2 years. I got in touch with someone who owned the site ~5 months back (when the site went down). It’s one of the reasons I started up my own website, to keep monitoring the new material, to better organize data, comments, information. Yeah, it’s brand new so there isn’t a ton of activity, but I’m hoping that with oversight, and as traffic increases, it’ll be a helpful resource for future test-takers.

Reply

430 DarbyNo Gravatar December 30, 2012 at 6:30 pm

This is the perfect web site for anyone who hopes to understand this topic.
You understand a whole lot its almost hard to argue with you
(not that I actually would want to…HaHa). You certainly put a new spin on a topic
which has been written about for many years. Excellent stuff, just great!

Reply

431 RicksterNo Gravatar December 31, 2012 at 4:41 pm

What were you scoring before you took test?
I am at around 30-32 per half exam on exams given 2000 and later.
I am about 25-29 per half exam on exams given before 2000.

Any one feel that the exams before 2000 are harder than the recent exams?

Reply

432 JakeNo Gravatar December 31, 2012 at 7:24 pm

I wouldn’t bother with pre-2000 unless you’re completely out of other tests and have gone through all the “exam questions and concepts” and “repeat questions” sections of this site.

I only took the 2003 and 2002 exams, scored in the 40s on the last two, and found the real exam to be easy (though still nerve-racking) because I was fast and efficient at searching the mpep.

I would push the exam date back (if you can w/in your window) if you’re getting less than 35 per half exam in a timed setting. Chances are that you will score lower on the real deal, so you need to be getting that 70%.

Reply

433 RicksterNo Gravatar January 3, 2013 at 3:09 am

I wouldn’t mind taking it much later but tests given after early April 2013 will include the 3d implementation of AIA.

I wonder if current tests are easier than the 2003-2001 tests.

434 BfusionNo Gravatar January 3, 2013 at 9:00 am

Rickster,

I agree with Jake If you can push your exam back a bit till you are scoring higher on the 2002-2003 exams, that would be better.

Of course, there are a LOT of factors to consider. How efficient are you at searching the MPEP, can you read questions fast and understand what the question is asking, etc. A lot of this “skill” comes from practice.

Stay away from pre 2000 exams (but yes, they did seem to be harder to me).

As far as current tests go, it’s really subjective. Some people study for hours and hours and get slammed on the exam, others study for 2 weeks and pass. A lot has to do with what and how you study, how you think, etc. Yes, depending on what test you get on your exam day, will also make a difference. BUT, some of the prior mentioned factors can be changed / learned.

Hope that helps…if you have any questions, feel free to email me!

bfusion.uspto@gmail.com

435 RicksterNo Gravatar January 4, 2013 at 5:24 pm

Repeat questions – to bfusion and Jake

I think knowing the Repeat questions on 2003 -2001 will boost anyone’s score by a couple of points.
I am just starting to “know” or memorize the repeats.

Reply

436 JakeNo Gravatar January 5, 2013 at 5:08 pm

You’ll probably get at least a few repeats, but it seems to be a 50-50 chance whether you’ll get a few or a lot.

Memorizing the old exam questions will only improve your score if either 1) You would have otherwise got the question wrong, or 2) You use the extra time to get additional questions right.

In my opinion, you’re better off working on speeding up your mpep search time by actively looking up each and every question. Chances are you’ll recognize the questions and the answers for a lot of them without trying to memorize them, and you’ll speed up on searching for other questions.

Reply

437 BfusionNo Gravatar January 7, 2013 at 9:23 am

Rickster,

It’s tricky saying “know the old questions” because behind that statement are a bunch of assumptions (that Jake touched on).

1) Just memorizing right/wrong answers for questions might get you a point or two. So, is it worth it? Well, maybe…but, if time is short, you could be spending that time more efficiently.

2) In “knowing” the questions, i think the key is to not only know the answer, but the “why’s” of it being right. This…as freaky as it sounds, happens rather organically the more you study. For me, as I studied, I easily memorized the right/wrong answers, but it wasn’t until probably 1-2 months after looking at the question that I “got it” / understood why it was right/wrong.

3) In “knowing” old questions, mentally, when you see similar facts, there will be a sense of familiarity. This is HUGE on the exam day. If you encounter 100 questions, and all are things you’ve never seen, ouch. BUT, lets say you get 50 questions that are similar to things you’ve seen before or are repeats. That’s a HUGE confidence booster. It’s small things like this that can make/break you on exam day.

WHICH (I know..shameless plug here) is why I started to help other test takers when I was going through the exam/studying myself. Often people focus so much on the material itself, neglect the smaller things, and fail. With some minor tweaks and focusing (and encouragement!) you can pass.

Which is what I and the team are doing over at the Wysebridge website. Yes, there are both free and paid memberships. But, with the forum access being part of the free membership, what i’m hoping is that by organizing what people discuss (unlike on this site, where it’s just streams of posts) into distinct topics and categories, current and future test takers will be able to find information easier and faster. I’ll continue to keep trying to help people here, but I’d be remiss if I’m not hoping that we can build a more organized and specific way to share and catalogue information.

here’s the site: http://www.wysebridge.com

438 Thank God I'm DoneNo Gravatar January 6, 2013 at 6:39 am

I’m selling my 2012 PLI material. You won’t get the online access with it. It is just the hard copy stuff (outline of the MPEP, practice questions and answers, glossary, etc.). Email me an offer at chemeval -at- hotmail -dot- com.

Reply

439 JasNo Gravatar January 8, 2013 at 10:09 am

How long does it take to register with USPTO?

I passed the patent bar exam on Nov 21, received the letter from USPTO a couple of weeks later and submitted the registration form and fee right away. The check was cashed on Dec 11. Still no further response from OED. Is it normal? When can I expect to receive the approval letter?

Reply

440 JakeNo Gravatar January 8, 2013 at 11:40 am

Your name has to be posted in the gazette for 2 months for the moral character check. See http://www.uspto.gov/ip/boards/oed/practitioner/notices/index.jsp

Reply

441 mikoNo Gravatar January 14, 2013 at 12:05 am

Since there were holidays (Thanksgiving/Christmas/New Year) in between, it may delay in your case, but the following is my time schedule for receiving my official certificate:

9/5/2012 – passed the patent bar
9/10/2012 – received the official letter from USPTO
9/11/2012 – sent in the forms/fee
10/31/2012 – saw my name/registration number on the USPTO website
11/15/2012 – received the official certificate
11/16/2012 – applied for the digital certificate
1/10/2013 – received the authorization codes for the digital certificate

Hope this helps… and congratulation on your passing, Jas!!!

Reply

442 pbarNo Gravatar January 10, 2013 at 9:58 pm

How accurate are prometric preliminary pass results? I want to celebrate but am afraid it is too good to be true.

Reply

443 ArthurNo Gravatar January 11, 2013 at 9:01 pm

I felt the same way after passing as you do now. You’ll get a letter from the USPTO director within 1.5 weeks. I took the exam on 12/21 and got the letter confirming my passing grade before 1/1 (I arrived home from a trip on 1/1 and the letter was in the mail). Congrats on passing too, pbar.

Reply

444 JakeNo Gravatar January 12, 2013 at 7:58 pm

You’ve got it. If you look around the site, someone mentioned that John White from PLI said no one has ever reported a preliminary pass that turned into a fail.

Reply

445 steveNo Gravatar January 13, 2013 at 4:34 pm

See comments 287 – 291 on the “Study Guide” page of this website.

Reply

446 pbarNo Gravatar January 13, 2013 at 9:02 pm

Good to know thanks. Congrats to you as well.

447 MartyDNo Gravatar January 24, 2013 at 9:19 am

Hello –

From what internet site can I download MPEP in a state most similar to what I’ll see on the Prometric exam (currently, that would be e8r8)? It appears from reading comments in various places that test preparation should only be using pdf files and Acrobat Reader 5.0, but while I have Acrobat 5.0 installed, I can’t find a zipped (or otherwise) MPEP file in pdf that I can download to simulate Prometric. Does anyone know how to get this?

I’m hoping to take the test in about a month. I’ve studied with “The Ultimate Patent Bar Study Guide” which I have read through in its entirety 2 times. I’ve also gone through the supplemental material provided on the USPTO website (KSR, Bilski, new third party preissuance, etc.). I haven’t spent much time yet going through old questions. I’ve done one test exam from October 2003 and got 62 correct – but that was with limited searching.

I’m taking the day off from studying today as my brain is a little fried. I’ll work on finalizing a preparation strategy as I head into the home stretch. My plan for the final 4 weeks is to hopefully master the MPEP search process in the format most closely resembling what I’ll see with Prometric. I envision this to be extremely important to assure success. I’ll also review the old test questions starting with October 2003 and work my way backwards, knowing the answers and why.

Another question: should I fail this time around (Feb 22), will I have enough time to turn around another application to squeeze in another test before the April 2, 2013 changeover? If anyone has any input or comments on getting the MPEP download, what my preparation strategy is, or whether I can get another test in, I’d appreciate his or her thoughts. Thanks very much.

Reply

448 SBNo Gravatar January 24, 2013 at 4:05 pm

E8R8 is here:

http://www.uspto.gov/web/offices/pac/mpep/old/mpep_E8R8.htm

PDFs for each chapter and appendix are separate, not zipped, and note that the page title (in your browser tab only) will be wrong but I guess someone forgot to update that when posting E8R8.

Reply

449 UnipecNo Gravatar January 24, 2013 at 4:58 pm

You can get the E8R8 at the archives, here: http://www.uspto.gov/web/offices/pac/mpep/old/index.htm

You’re right to think quick searching is important. Thus, in my opinion, taking a test with limited searching isn’t helpful, even if you know all the answers. I would spend more time focusing on searching while you’re doing the tests, for every question. I would go through all the old exams over and over, searching for the answer to each question each time you do. It’ll take forever at first, but you’ll get far faster.

I’d also go through the “Exam Questions and Concepts” posts several times. These are mostly posts of questions that post date the 2003 exam, so there’s a lot of debate in there, and you’ll have to search through the MPEP in several different places before you understand most of the posts and comments together.

If you fail on the 22nd, getting in before April 2 might be tight. I think reapplying is far faster, but I’m not sure. Even so, assuming the USPTO turned over your app in 2 weeks from Feb 23, you’d have just above 2 weeks before April 2, which, around my area, is generally too short notice to get an appointment at a test center. Maybe it’s different in yours. Either way, I’d have the application ready to go in the mail the next day, even the night of the 22nd if you can.

Depending on how confident you are, I’d suggest 1 of 2 alternatives. The first is to take your exam sooner, so you’ll have more time to get another app in. This might be a waste of $150, though, if you wont be ready. The second is to push your exam back until the end of February or the beginning of April. If you study hard until then and crush the exam, there’s no reason to care about getting a 2nd app in. This might cost you $30 to reschedule, but that’s cheaper than $390 for app and exam fees.

Reply

450 MartyDNo Gravatar January 24, 2013 at 10:01 pm

Many thanks to SB and Unipec –

I will download the pdf’s and begin practicing the searching process in earnest. I’ll have to give some thought to a strategy on when to take the first exam.

My impression from looking at the USPTO website is that to sign up for a retest will cost the full amount of $390 each time. Is that the case or does it depend on the timing of the retest? Thanks again for your input.

MartyD

451 UnipecNo Gravatar January 29, 2013 at 4:07 pm

MartyD,

I know that you have to play the full $390 each time if your window expires, but I don’t know if you fail and apply again during the same window. I’m pretty sure there’s a section on that in the application materials.

Unipec

452 GuyNo Gravatar January 29, 2013 at 4:59 pm

I know from experience that after a failed attempt, you can apply immediately (I sent in my application the next day). You need to fill out the two page application from the registration bulletin and pay the fee $240) and then later the $150 to Prometric when you schedule the test. You do not need to submit anything other than the application (including the info on when you last applied to take the test). You need to pay the full amount ($240+$150) each time you take the test regardless of the reason (expiration of 90 day window or failure).

The PTO will send you your letter letting you know that your 90 day window starts 30 days after you fail date (e.g. if you fail your 2//22 test and get your application in right away your next window could start no sooner than 3/22). The trick there will be that the Prometric seats will start to get tight, especially because of the big change coming up with regards to the exam (which also happened at the end of September before the exam added new material starting the first week of Oct. ’12).
If you fail (knocking on wood that you don’t) I would get your app mailed the next day so you can get your Prometric date ASAP before all the openings before 4/2 fill up.

453 Stephanie HeegNo Gravatar January 24, 2013 at 4:29 pm

Arnold & Porter LLP has an opening for a Patent Law Clerk in the Washington, D.C. office. The Patent Law Clerk will be directly supervised by an attorney and get individualized patent law training. Ideal candidates will be organized; detail-oriented; and able to follow up on instructions independently. Responsibilities include: preparing and filing new patent applications and other patent prosecution documents; conducting client technology and patent portfolio assessments; supporting business development; managing large domestic and foreign dockets and other duties as assigned.

Candidates should have a Ph.D. and significant graduate level experience in plant biology, molecular genetics, immunology, or biochemistry as well as excellent academic credentials and strong verbal and written communication skills. The ideal candidate will have passed the Patent Bar. Law school or prior legal training or enrollment in law school is not required. Benefits include a competitive salary and a tuition reimbursement program when applicable. Please include a cover letter, resume, school transcripts (please include each school), and writing sample with your online application.

Arnold & Porter LLP is an equal opportunity and affirmative action employer that does not discriminate on the basis of race, religion, color, national origin, sex, veteran’s status, age, disability, sexual orientation, gender identity or any other characteristic protected by federal, state or local laws. Our Firm’s policy applies to all terms and conditions of employment. To achieve our goal of equal opportunity, Arnold & Porter maintains an affirmative action plan through which it makes good faith efforts to recruit, hire and advance in employment qualified minorities, females, disabled individuals, and covered veterans. EOE M/F/V/D

Click here to apply: http://www.arnoldporter.com/jobs.cfm?u=PatentLawClerk&action=view&id=472

Reply

454 Tom IsenbargerNo Gravatar January 31, 2013 at 9:02 pm

Is there a PDF reader for Mac (OS X 10.8) that is similar to the ProMetric search utility?

Reply

455 JakeNo Gravatar February 1, 2013 at 10:36 am

I could not find one. I tried old versions of Acrobat for OSX, but they did not work. I eventually installed acrobat 5 for windows on my vmware install.

Reply

456 NewbieNo Gravatar February 8, 2013 at 3:37 am

Dear all,
I was able to schedule to sit for the USPOT exam on March 29, 2013. I had a choice of any day during the week of March 25 to March 29, 2013. And I am not prepared for taking an exam with the latest implementation of the AIA of March 16, 2013 (Phase 3).

My questions are:
– On March 29, 2013, will I be tested for the new materials, which are related to the March 16, 2013 implementaton of the AIA? In other words, will I be subjected to the “newest” exam?

– Is there supposed to be a blackout period betwen the current exam and this “newest” exam? I thought there was going to be blackout period in between. And I was surprised thal all dates at the end of March (right before the “newest” exam will begin) are available at a nearby Prometric tes stie.

– Is it possible that the USPTO or Prometric will notify me in a few weeks that my already-scheduled March 29, 2013 date will no longer be available because it happens to fall during the blackout period, so that I will be forced to pick another date?

Thanks!

Reply

457 GuyNo Gravatar February 8, 2013 at 10:41 am

The PTO is not going to start testing the March 2013 changes until sometime around April 2nd:

http://www.uspto.gov/ip/boards/oed/exam/index.jsp

“Examination for Registration to Practice in Patent Cases will be updated again in early April 2013 to Include New Provisions from the Leahy-Smith America Invents Act (AIA)
The United States Patent and Trademark Office will be updating the content of the registration examination on April 2, 2013. The updated examination will include provisions of the Leahy-Smith America Invents Act (AIA) that will take effect March 16, 2013. Specifically, the updated examination will additionally cover the following topics: Derivation Proceedings and First-Inventor-to-File. For further information concerning AIA, please refer to the AIA Implementation web page. Candidates for the registration examination, see Source Materials for the Registration Examination.”

I know some co-workers were trying to schedule exam dates back in September (before the October 2012 update to the exam) and there seemed to be no dates from 9/26 through 10/2 or something which felt like a blackout period, but maybe Prometric was just really really full.

I’ve never heard of anyone having Prometric rescind a test date after it was already scheduled, but never say never…

Reply

458 BfusionNo Gravatar February 11, 2013 at 1:12 pm

Newbie,

1) On March 29, 2013, will I be tested for the new materials, which are related to the March 16, 2013 implementaton of the AIA? In other words, will I be subjected to the “newest” exam? The answer..yes and no. There are for sure going to be some “beta-questions” on your exam (10 of them), and you can probably count on some of them being directly related to the March 16th AIA changes. While these don’t count towards your score…well, you don’t want to bank on trying to determine which are beta/which aren’t.

2) As far as blackout periods go, same as what guy mentioned. There’s been no indication this will occur. BUT, count on exam dates filing up
FAST for that week before April 2nd, as everyone is going to be trying to take the exam before then.

3) Rescheduling….yeah, this would be a new one, and I haven’t looked through prometric’s literature/disclaimers, but it wouldn’t surprise me if they hold some right to reschedule.

I’ve also put together some materials on another site (wysebridge.com) and have a blog going discussing the exam (wysebridge.wordpress.com) to try and help people as they take this exam.

All the best to you as you study!

459 WilsonNo Gravatar February 14, 2013 at 6:06 pm

If I were you I would try to schedlue earlier than March 29th, just in case there is a black out. Your concerns are valid!

Reply

460 NewbieNo Gravatar February 15, 2013 at 2:28 pm

Thank you all so much!!

461 MoeNo Gravatar February 14, 2013 at 4:56 pm

In case anyone is curious, it took 13 business days from the day I sent my application for a re-test to receiving an approval. My ID number is still not in the Prometric system, but my first go around it took a day or so before I was able to schedule.

Thought this may be important to people wondering if they would have another shot before April.

Reply

462 MoeNo Gravatar February 14, 2013 at 5:00 pm

I will report back once I can actually see my test center availability.

Reply

463 MoeNo Gravatar February 15, 2013 at 1:45 pm

Was able to schedule today.. So 14 *business days* from the mailing of my re-application to being able to schedule.

Good news is that the only weekday NOT available at my location before the major changes was April 1st, but all of the Saturdays were booked. This eases some fears I had of a “mad rush” to take the exam before then.

464 Ramon GlydeNo Gravatar February 14, 2013 at 10:41 pm

Preliminary pass today! Such a relief.
Here is my brain dump. I will try to do more later.
There were around 25 repeats total (out of 100). Since you all know what those are, I will try to put the new stuff here.
Standing in inter parte review: The asked about use of court documents (did not say deposition) such as statements the inventor made in infringement and a criminal matter. I wasted maybe 15 minutes trying hunt down the answer, to no avail. I just guestimated that any statement in an adverse proceding would not be usable.
Inherent function question using a bicycle. Claim is for handle bars, frame, wheels, and spokes. Later the applicant tries to add rubber tires but is rejected as new matter. I went with arguing that the tire is an inherent part of a wheel.
Claim count where answer was 8. All I had to do was count the number of dependent claims and add in multitples when something depended on more than 1 preceding claim.
A question the new post-grant review (up to 9 months after) and inter parte review. Was pretty basic if you know the cutoff.
Another question where you had to know that a party can initiate an inter parte review even though first involved in a civil action in the patent (must be the defendent in the matter- not the one who initiated the civil action) This also took about 10 minutes to find – I initially had the wrong answer
Electronic filing question that wasted some time. I never found the answer but just went with “the person filing an e-signature is certifying that they are in fact that person” (it seemed to good to be true so I wasted time trying to disqualify the other answers)
Tricky one: Applicant gives partial assignment to practitioner and dies. I think the correct answer was the power of attorney is revoked on his death. (Yes, the practitioner can still prosecute the petition because of the partial assignment but, the power of attorney is no longer in force.)

You need to know 102abcdefg and 103 but, don’t bother memorizing. There are many questions regarding these and they are easy to find in the MPEP (you all know where). Just open the MPEP to the section when reading these questions. – I was very happy to see these numbers when I opened a new question because I knew I would not waste a lot of time searching in remote areas of the MPEP.

Know the difference between anticipation and obviousness as they try to trick you by asking an obviousness question and giving an answer having to do with anticipation, and vice-versa.

Oh yes: a 30 months PCT -> national stage question I have not seen. What happens if someone passes through 30 without providing what is needed for the national stage? I initially had the wrong answer but, on review found somewhere that they can petition to revive. Other answers were Give up, you will never get it, and 2 other options involving extensions to file in 31 months (not possible in this case).

Anyway, I welcome your comments telling me I am wrong on something. I really don’t care because I am soooooooo happy I passed. I used Omniprep because I signed up with them last summer (didn’t get anything done for so long due to a move and other things). They are not very up to date but the practice tests and MPEP summary were good. If I had it all to do over again I would probably use wysebridge. I used the list of new and old repeat questions from the free side to create a list of questions to test myself with.

The best thing: Read every sample question here and all of the commentary. AS you read think about who you agree with and who you don’t. Look up the stuff in the MPEP.

Reply

465 Ramon GlydeNo Gravatar February 14, 2013 at 10:51 pm

I forgot to add: Thank you again to whoever is maintaining this site. It was a lifesaver.

Reply

466 ObviousNo Gravatar February 18, 2013 at 10:42 pm

Ready set Go!! USPTO announces the fun new rules everyone must master to pass this exam in about a month:

http://www.uspto.gov/news/pr/2013/13-10.jsp

The Federal Register publication – Changes to Implement First Inventor to File Provisions of Leahy-Smith America Invents Act – can be found at http://federalregister.gov/a/2013-03453 and the Federal Register publication – Implementing First Inventor to File Provisions of Leahy-Smith America Invents Act: Examination Guidelines – can be found at http://federalregister.gov/a/2013-03450.

Makes me wonder if I really want to take this exam!

Reply

467 BfusionNo Gravatar February 22, 2013 at 11:02 am

As a REMINDER: For anyone looking to take the exam prior to April 2nd, 2013, if you haven’t registered yet and submitted your application, DO IT NOW!

Keep in mind it will take 1-3 weeks to get your official acceptance to take the exam, and then you’ll have to ensure you can get a spot at a Prometric Testing Center!

Just some thoughts to keep in mind if you haven’t registered yet!

If anyone has questions or needs some feedback, help, etc….I’ll try and help as much as I can. you can email me (bfusion.uspto@gmail.com) or submit a request over on wysebridge.com

Reply

468 SeanNo Gravatar March 1, 2013 at 2:01 am

Hello…quick question on the cheaper prep courses:

What do folks currently think of OmniPrep or PatBar.com?

Thank you!
Sean

Reply

469 SatgoankNo Gravatar March 2, 2013 at 5:16 am

best for you [URL=http://cheaplouisvuitton23.devhub.com/]cheap louis vuitton bags[/URL] online shopping PfxaXkPy [URL=http://cheaplouisvuitton23.devhub.com/ ] http://cheaplouisvuitton23.devhub.com/ [/URL]

Reply

470 joshjtaNo Gravatar March 10, 2013 at 9:18 pm

Is Prometric still using a PDF viewer that most closely resembles Adobe Reader 5.0? The most recent comments about this I can find are from 2008, though there are later references to the viewer still being out-of-date.

Reply

471 joshjtaNo Gravatar March 10, 2013 at 10:43 pm

Also, so far as I can see, to look at the MPEP using Adobe Reader, I would need to open a directory containing all the little files in the MPEP and then select which file to open with Reader. But on the real exam, we will be able to select a topic/file from an index after clicking on the “MPEP” button, right? So how do we reproduce that so that we can pick a file from an index and don’t have to select a file from the windows explorer window?

Reply

472 PatentMidgetNo Gravatar March 17, 2013 at 11:35 pm

Wondering if you ever received an answer. I have the same question.

Reply

473 PatentMidgetNo Gravatar March 17, 2013 at 11:35 pm

As someone above addressed, can someone who has recently taken the exam describe the operation of Adobe and the MPEP? How much search function did you have and etc.? Very much appreciated!

Reply

474 MoeNo Gravatar March 25, 2013 at 7:15 pm

There are two buttons at the top of each question, one for “MPEP,” and one for “Supplemental Information” or something similar.

When you click on MPEP, there is a dropdown menu at the top which lists each chapter and their titles. Once you select a chapter, there is the standard navigation list along the left listing each section within the chapter, which you can click on to navigate quickly.

For searching, there is a “Find” and “Find Next” button at the upper left once you open a chapter. The find dialogue window has only basic functions on it. You can choose to keep the find window open as you search through the document, match the entire word, or search up/down (probably something else I’m forgetting). One annoying thing for me was not being able to go back very easily when i skipped passed something via “Find Next”. I’d reverse the find function to search “Up” in these instances.

Another annoying thing is that the search appears to pick up wherever it is stopped (likely a way around this, but I did not waste time finding out). For example, if you search for a word and get too many hits and decide to search for something else, it will automatically start at the point where you are at.

Other than those two minor annoyances, the search is fine once you get used to it. Just remember to use the “Find Next” and “Close” buttons rather than the Enter key and “X’s” to avoid crashing the program.

Reply

475 PatentMidgetNo Gravatar March 26, 2013 at 4:25 am

Thanks! I appreciate the info. I’m taking the exam on Thursday and really really don’t want to slip over into post-April 2nd world, so passing is a must. Feeling good about non-AIA material. New material seems a bit of a black box to me, other than just some basic high level trivia that you can pull out of the supplementary material. We’ll see how it goes I guess.

Saw below that you passed. Congrats!

476 zeplieksNo Gravatar March 19, 2013 at 3:43 pm

best for you designer outlet ytSDtTBq [URL=http://www.cheap-designer–handbags.weebly.com/]knockoff handbags[/URL] and check coupon code available mLhTdglO http://www.cheap-designer–handbags.weebly.com/

Reply

477 MoeNo Gravatar March 25, 2013 at 3:17 pm

Preliminary pass today!

19-20 repeats (Kept a tally)
1 Bilski
2 KSR
2-3 PGR/Inter party
4-5 total with dates after October 2012, easy to spot

Quite a few 102 and 103 questions; this is what I imagine the test was like before the late 2012 AIA changes. The afternoon session was a bit tougher, but at least I got to start day with some easily recognizable repeats.

Some notable repeats:
-Submitting tables on CD-ROM
-Smith laminate (negative variant)
-Broom multiplicity
-Best mode
-Combination/Sub-combination
-French Martinet
-Potter
-Japan 45 days
-Laurel, Abbot, Hardy
-DRAM
-Claim counting (low number variant)
-Dead inventor POA+assign
-Foreign design patent priority

There were many more that I can’t remember off the top of my head, but I was mostly familiar with the “new” repeats and 2003 Exams. I spent a lot of time on here and Wysebridge, and would have paid for the full version over there if I hadn’t already bought the PLI set.

Reply

478 singing guideNo Gravatar March 28, 2013 at 7:23 am

Hi, I do think this is a great blog. I stumbledupon it 😉 I’m going to return yet again since i have book marked it. Money and freedom is the best way to change, may you be rich and continue to help other people.

Reply

479 diet_cokeNo Gravatar March 29, 2013 at 3:15 pm

Since there have been a couple of questions on accessing the supplemental materials – I figured I’d throw in my 2 cents worth. In short, its just like accessing the mpep (drop down list – each one is in individual pdf that loads in a new window). The materials are in fact the pdf’s you can download from the PTO’s list of source materials (they even have the GPO certified document symbol at the top!)

http://www.uspto.gov/ip/boards/oed/exam/aia_regexamsourcematerial2012sep04.jsp

Reply

480 BobSmithNo Gravatar April 2, 2013 at 10:32 am

Passed: March 27, 2013.

Had at least 50 repeats (25-30 from new questions, 15-20 old exams), one KSR, a couple 2012 type questions.

I remember the following:

CCAR
Suspended practitioner
Sweden PCT
Trade Secret
Germany PCT
Electric Fan/light
Dead inventor – friend as executor
Chemical Claims – headaches
Z to treat cancer
Indefinite “High”
Rejection after allowance
Appeals: dependent claims rejected
New ground of rejection by board
RCE filed with no fee
When board affirms rejection + new grounds for rejection
Cancelled matter
Foil airplane
Japan 45 days
Beck
Able
Potter

I did the 2006 PLI course, studied old test to the point I could recognize the answer without reading the questions. I knew the wrong answers and why, studied old exams back to 2000, knew all the issues involved in the reported new questions. I did not read any sections of the MPEP in their entirety however I read each section outlined in the model answers in depth when reviewing practice tests. The new questions will not appear exactly as they do on this web page but the issues are framed very well and you will know the correct answer. I was able to look up about 5 questions in each session. The search feature worked as expected,

Thanks tp everyone who has contributed to this and other web pages. Make sure you check out the WyseBridge web page.

Reply

481 sax toyNo Gravatar April 20, 2013 at 6:30 pm

This post presents clear idea for the new visitors of blogging, that actually how
to do running a blog.

Reply

482 CDNo Gravatar April 26, 2013 at 5:20 pm

I passed the Registration Exam yesterday. I definitely had a lot of repeat questions from the old list and the new list. It is essential to study these.

I also spent the 15 minute tutorial in the beginning creating a matrix for my answers, it looked like:
1. A B C D E
. . .
50. A B C D E
I also created a pre-AIA 102 matrix to refer to (didn’t need it, I knew 102)

I used the PLI guide, which was expensive, but I passed on my first try. However, before I looked at repeat questions I was only getting about 70% on the 2002-2003 exams. I reviewed PLI’s AIA packets and the repeat questions and I was fine.

Good luck everyone.

Reply

483 GottaTryNo Gravatar April 27, 2013 at 10:39 am

CD,

Congratulations on passing. It’s refreshing to hear that the USPTO hasn’t removed the old questions from the data bank completely. I know the NDA is making impossible for anyone to divulge any specifics about the exam, but are you allowed to comment as to the number of “AIA” questions you saw on your exam? Not asking for specifics, just how many AIA questions….thanks.

Reply

484 ENo Gravatar May 13, 2013 at 1:19 pm

Hi CD,

Congrats.

I have a stupid question. When you say “new list”, what do you mean by that?

Thanks,
E

Reply

485 hunter wellies cheapNo Gravatar May 23, 2013 at 1:38 am

I’ve been surfing online more than 3 hours today, yet I never found any interesting article like yours. It is pretty worth enough for me. Personally, if all web owners and bloggers made good content as you did, the web will be a lot more useful than ever before.|
I could not refrain from commenting. Perfectly written!|
I’ll immediately snatch your rss feed as I can not find your e-mail subscription link or newsletter
service. Do you have any? Please let me recognise in order that I may just subscribe.
Thanks.|
It is appropriate time to make some plans for the future and it is time to be happy.
I have read this post and if I could I wish to suggest you few interesting
things or suggestions. Maybe you could write next articles referring to this article.
I desire to read even more things about it!|
It’s the best time to make a few plans for the future and it’s
time to be happy. I have read this publish and if I may I want to
recommend you some fascinating issues or tips. Perhaps you
could write next articles referring to this article. I wish
to learn more things approximately it!|
I have been browsing online greater than three hours nowadays,
but I never discovered any interesting article like yours.
It is lovely value enough for me. In my opinion, if all site owners and bloggers made just right content as you did, the web might
be a lot more useful than ever before.|
Ahaa, its pleasant discussion regarding this article at this place at this
blog, I have read all that, so now me also commenting at this place.
|
I am sure this article has touched all the internet
viewers, its really really nice post on building up new web site.
|
Wow, this piece of writing is fastidious, my younger sister is analyzing these things, therefore I am going to inform
her.|
Saved as a favorite, I love your site!|
Way cool! Some very valid points! I appreciate you writing this
post and the rest of the website is also really good.
|
Hi, I do think this is an excellent site. I stumbledupon it 😉
I am going to come back yet again since I bookmarked it.
Money and freedom is the greatest way to change, may you be rich
and continue to guide other people.|
Woah! I’m really digging the template/theme of this site. It’s simple, yet effective.
A lot of times it’s tough to get that “perfect balance” between superb usability and visual appearance. I must say you have done a awesome job with this. Additionally, the blog loads very quick for me on Chrome. Exceptional Blog!|
These are actually enormous ideas in about blogging. You have touched some good points here. Any way keep up wrinting.|
I love what you guys are up too. Such clever work and coverage! Keep up the wonderful works guys I’ve added you guys to my personal blogroll.

|
Hello! Someone in my Facebook group shared this website with us so I came
to check it out. I’m definitely loving the information. I’m book-marking and will
be tweeting this to my followers! Great blog and wonderful design.
|
I really like what you guys are usually up too.
This kind of clever work and coverage! Keep up the wonderful works guys I’ve added you guys to my blogroll.|
Hello would you mind sharing which blog platform you’re working with?
I’m going to start my own blog soon but I’m having a difficult time deciding
between BlogEngine/Wordpress/B2evolution and Drupal. The reason
I ask is because your design and style seems different then most blogs and I’m looking for something completely unique. P.S Apologies for getting off-topic but I had to ask!|
Hello would you mind letting me know which web host you’re using?
I’ve loaded your blog in 3 different web browsers and I must say this blog loads a lot quicker then most. Can you suggest a good hosting provider at a reasonable price? Thanks, I appreciate it!|
I love it whenever people get together and share views. Great site, stick with it!|
Thank you for the good writeup. It in fact was a amusement account it. Look advanced to far added agreeable from you! By the way, how could we communicate?|
Hey just wanted to give you a quick heads up. The text in your content seem to be running off the screen in Ie. I’m not sure
if this is a format issue or something to do with browser compatibility but I thought I’d post to let you know. The design and style look great though! Hope you get the issue fixed soon. Kudos|
This is a topic that is close to my heart… Many thanks! Exactly where are your contact details though?|
It’s very trouble-free to find out any topic on net as compared to textbooks, as I found this post at this web site.

|
Does your website have a contact page? I’m having problems locating it but, I’d like to shoot you
an e-mail. I’ve got some ideas for your blog you might be interested in hearing. Either way, great site and I look forward to seeing it grow over time.|
Hello! I’ve been reading your blog for a long time now and finally got the bravery to go
ahead and give you a shout out from Humble Texas!
Just wanted to say keep up the fantastic work!|
Greetings from Colorado! I’m bored to death at work so I decided to check out your website on my iphone during lunch break. I enjoy the information you provide here and can’t wait to take a look when I get home.
I’m shocked at how fast your blog loaded on my mobile .. I’m
not even using WIFI, just 3G .. Anyways, superb site!
|
Its like you read my thoughts! You seem to understand a lot
about this, like you wrote the book in it or something.
I feel that you simply could do with a few p.

c. to power the message house a bit, but other than that, this is
wonderful blog. An excellent read. I’ll certainly be back.|
I visited various blogs but the audio quality for audio songs current at this website is truly superb.|
Hello, i read your blog occasionally and i own a similar one and i was just curious if you get a lot of spam feedback? If so how do you protect against it, any plugin or anything you can recommend? I get so much lately it’s driving me mad so any support is very much appreciated.
|
Greetings! Very helpful advice within this post!
It’s the little changes that make the greatest changes. Thanks a lot for sharing!|
I truly love your site.. Very nice colors & theme. Did you develop this web site yourself? Please reply back as I’m looking to create my own personal blog and would like to know where you got this from or exactly what the theme is named.
Cheers!|
Hi there! This post could not be written any better! Looking at this article reminds
me of my previous roommate! He continually kept talking about this.
I am going to send this information to him. Fairly certain he’ll have a great read. Thank you for sharing!|
Whoa! This blog looks exactly like my old one! It’s on a completely different
topic but it has pretty much the same page layout
and design. Outstanding choice of colors!|
There’s certainly a great deal to learn about this subject. I love all the points you’ve made.
|
You have made some good points there. I looked on the net for more info about the issue and found
most individuals will go along with your views on this web site.
|
Hello, I check your new stuff daily. Your writing style is awesome, keep up
the good work!|
I simply couldn’t depart your site prior to suggesting that I really loved the usual information an individual supply for your guests? Is gonna be back steadily in order to check up on new posts|
I need to to thank you for this very good read!! I absolutely loved every bit of it. I have you saved as a favorite to look at new stuff you post…|
What’s up, just wanted to tell you, I enjoyed this article.
It was practical. Keep on posting!|
I comment each time I appreciate a article
on a blog or I have something to contribute to the discussion.
Usually it’s a result of the sincerness displayed in the post I browsed. And after this article Prometric Patent Bar. I was actually moved enough to post a thought 😉 I actually do have a couple of questions for you if it’s allright.
Could it be simply me or does it look like a few
of these comments look like they are written by brain dead folks?
😛 And, if you are writing at other social sites, I’d like to follow you. Would you list the complete urls of your social pages like your linkedin profile, Facebook page or twitter feed?|
Hi there, I enjoy reading all of your post. I wanted to write a little comment to support you.|
I every time spent my half an hour to read this web site’s content everyday along with a cup of coffee.
|
I every time emailed this website post page to all my contacts, because if like to read it afterward my friends will too.

|
My programmer is trying to convince me to move to .
net from PHP. I have always disliked the idea because of the costs.

But he’s tryiong none the less. I’ve been using
WordPress on numerous websites for about a year and am nervous
about switching to another platform. I have heard excellent things about blogengine.

net. Is there a way I can import all my wordpress posts into it?
Any help would be greatly appreciated!|
Howdy! I could have sworn I’ve been to this site before but after looking at some of the articles I realized it’s new
to me. Regardless, I’m certainly delighted I stumbled upon it and I’ll be
bookmarking it and checking back often!|
Wonderful work! That is the kind of info that are meant to be shared across the net.
Shame on the seek engines for not positioning this submit higher!
Come on over and consult with my website . Thanks =)|
Heya i am for the first time here. I came across this board and I find It really
useful & it helped me out a lot. I hope to give something back and aid others like you helped
me.|
Hi, There’s no doubt that your blog may be having web browser compatibility problems. Whenever I look at your blog in Safari, it looks fine however, if opening in Internet Explorer, it’s got some overlapping issues.
I just wanted to give you a quick heads up! Besides
that, excellent website!|
Someone essentially lend a hand to make severely articles I might state.

That is the first time I frequented your web page and up to
now? I amazed with the analysis you made to make this particular publish extraordinary.
Great job!|
Heya i am for the primary time here. I found this board and I find It truly useful & it helped me out much.
I am hoping to offer one thing back and help others like you helped me.
|
Hello there! I simply wish to give you a huge thumbs
up for your excellent info you have right here on this post.
I’ll be returning to your web site for more soon.|
I all the time used to read article in news papers but now as I am a user of internet so from now I am using net for articles, thanks to web.|
Your method of describing all in this article is in fact good, every one can effortlessly be aware of it, Thanks a lot.|
Hi there, I discovered your site by the use of Google even as looking for a related matter, your website got here up, it appears great. I have bookmarked it in my google bookmarks.
Hi there, just changed into alert to your weblog via Google, and found that it’s really informative.

I am going to watch out for brussels. I will be grateful in case you proceed this in future.

Many folks can be benefited from your writing. Cheers!
|
I’m curious to find out what blog platform you happen to be using? I’m having some minor
security problems with my latest blog and I’d like to find something more safe. Do you have any recommendations?|
I’m really impressed with your writing skills and also with the layout on your blog.
Is this a paid theme or did you customize it yourself?
Either way keep up the nice quality writing,
it’s rare to see a great blog like this one today.|
I am extremely inspired along with your writing talents as neatly as with the layout on your weblog. Is this a paid subject matter or did you modify it your self? Either way keep up the nice high quality writing, it is uncommon to peer a great blog like this one these days..|
Hi, Neat post. There is a problem with your site in web explorer, could check this? IE still is the marketplace chief and a large part of other people will miss your wonderful writing because of this problem.|
I am not sure where you are getting your information, but great topic. I needs to spend some time learning more or understanding more. Thanks for excellent info I was looking for this information for my mission.|
Hello, i think that i saw you visited my blog thus i came to “return the favor”.I am trying to find things to improve my site!I suppose its ok to use some of your ideas!

Reply

486 basketnewbalanceplaphyGlyncNo Gravatar May 31, 2013 at 8:58 am

Homer strolled as part of your just like a terribly created automaton as well as their functions had been occur the firm, mechanised smile. He’d their pants upon more than their nightgown as well as a part of this strung from their open up travel. Within each associated with their fingers had been luggage. Along with every action, he or she lurched to 1 aspect then your additional,
He or she had been attempting to determine how to proceed in the event that he or she were not able in order to wake up Homer whenever, all of a sudden he or she noticed their mind bobbing over the actual group. He or she rushed towards him or her. Through their look, it had been apparent which there is some thing certainly incorrect.
sunlight isn鈥檛 sufficient. These people obtain fed up with grapefruits, actually associated with avocado pears as well as enthusiasm fresh fruit. Absolutely nothing occurs. These people don鈥檛 understand how to proceed using their period. These people haven鈥檛 the actual psychological gear with regard to leisure time, the cash neither the actual bodily gear with regard to enjoyment. Do these people servant such a long time simply to visit an intermittent Iowa have a picnic? Exactly what otherwise can there be? These people view the actual surf are available in from Venice. Presently there wasn鈥檛 any kind of sea exactly where many of them originated from, however following you鈥檝e observed 1 influx, you鈥檝e observed all of them. Exactly the same will additionally apply to the actual planes from Glendale. If perhaps the airplane might accident every now and then so they might view the actual people becoming eaten inside a 鈥渉olocaust associated with fire, 鈥?since the papers place it. However the airplanes in no way accident.
Tod was upward. Throughout the 10 min’s he’d already been seated about the walls, the actual group experienced developed 30 ft as well as he or she had been scared which their get away may be stop in the event that he or she loitered considerably longer. He or she entered towards the additional aspect from the road as well as began back again.
Their own monotony gets increasingly more horrible. These people understand that they鈥檝e already been fooled as well as burn off along with bitterness. Every single day of the life these people browse the papers as well as visited the films. Each given all of them upon lynchings, homicide, intercourse offences, explosions, accidents, adore nests, that will fire, wonders, cycles, battle. This particular every day diet plan created sophisticates of these. The sun’s rays is really a laugh. Grapefruits can鈥檛 titillate their own experienced palates. Absolutely nothing may actually end up being chaotic sufficient to create tight their own slack thoughts as well as physiques. They’ve been scammed as well as tricked. They’ve slaved as well as preserved with regard to absolutely nothing.

________________________________

Reply

487 ルイビトン 格安No Gravatar June 1, 2013 at 8:47 am

Have you ever thought about including a little bit more than just
your articles? I mean, what you say is important and everything.
But think about if you added some great graphics or video clips to give your
posts more, “pop”! Your content is excellent but with images and
videos, this website could certainly be one of the best in its niche.
Terrific blog!

Reply

488 robert bottNo Gravatar June 8, 2013 at 12:37 am

Is Cover Business Methods i.e. CBM tested?

Reply

489 john wunder scamNo Gravatar August 8, 2013 at 7:39 pm

Hi, Neat publish. Likely to situation with your site with web traveler, could test out this particular? Web browser nevertheless is definitely the sector boss and also a substantial piece of others can take out the fantastic producing for this problem.

Reply

490 RLNo Gravatar August 26, 2013 at 7:36 pm

For past test takers, will the exam will have the Federal Register publications available for searching (i.e, for the new AIA enactments)?

I’m presuming the answer is “no.” Then, for information being tested in those publications, will just have to be memorized as they have yet to make their way into the MPEP, right?

Reply

491 MikeNo Gravatar December 1, 2013 at 1:13 pm

I am not sure whether I can post this here.

We are looking for freelancer Patent agent who has normal registration number before USPTO. No experience required and we provide basic training how to do actually do patent prosecution work. If you are interested please give me an email.

Mike
patent.nexus@gmail.com

Reply

492 Kansasman2014No Gravatar December 17, 2013 at 5:00 pm

HI All,
I have my new PLI materials for sale (have constructive hand notes on the side). Great course. Please let me know if you want to get it. contact me kansasman2014 at yahoo
Thanks

Reply

493 FreeTheBarNo Gravatar December 19, 2013 at 11:39 pm

RL.
Yes all material listed for the current exam is available and searchable.

Reply

494 ChuckNo Gravatar February 9, 2014 at 3:57 pm

I know this has been asked several times, but I can’t find the answer. How does the search function work on the prometric test? If I do a search and find the section I’m looking for, if I enter in another search term, will it start searching in that section?

Reply

495 Bfusion.No Gravatar February 15, 2014 at 9:26 pm

Hi Chuck,

So, on the exam, the MPEP and any additional material (Federal Registers) will be in a drop down menu. You’ll be able to select one of those, and then once that section is open, you can search within that section.

This is unlike the online MPEP (where you can search the entire document).

All the best to you as you study!

~Bfusion
http://www.wysebridge.com/

Reply

496 ChuckNo Gravatar February 15, 2014 at 1:45 pm

How does the search function work? if my initial search term brings me to a section and i enter a new search term, will it start from the current position or from the beginning?

Reply

497 ZhangNo Gravatar March 6, 2014 at 4:35 pm

Hi All,

I am new here. I just submitted my exam application. Can anybody tell me how long it usually takes to get a notification from the USPTO. Thanks!

Reply

498 JustenNo Gravatar March 6, 2014 at 5:25 pm

I just got my reply from the office. It took three and a half weeks.
But I applied under “plan A.” If you applied under “plan B” it will take a lot longer.

Reply

499 ZhangNo Gravatar March 7, 2014 at 2:08 pm

Hi Justen,

Thanks much for your answer. I applied under “Plan A”, but not sure if that’s OK. My bachelor’s degree is on Materials Science and Engineering, but did not find the exactly same subject in the bulletin.

By the way, which source are you using for the exam? (PLI, OmniPrep or something else?) I am trying to find a not expensive, but helpful way to start my preparation. Thanks.

Reply

500 ChucklesNo Gravatar March 18, 2014 at 12:40 am

Just took and apparently passed the exam today, March 17, 2014.

My experience mirrors many of those above. It is still critically important to drill the old 2003 exams. I had a good 15 repeats. Lots of 1200 and 1800 questions with several questions about RCEs – my guess is that it was an unusually high number.

The afternoon session hit the latest AIA stuff hard. You MUST become familiar with the newest PLTIA stuff and post-grant review procedures. These are NOT yet in the MPEP but are being tested heavily. Know the differences between inter partes review, post-grant review, the transitional program for covered business methods and the older reexam procedures. These are laid out in the USC but if you are not familiar with them you will be stupefied on the exam. Know the rules for transitional applications and whether pre-AIA or AIA applies. The newest stuff comprised a good 20+ questions.

I actually had a chapter 900 question but it was an easy look-up.

Regarding look-up, it is a very good idea to develop good look-up strategies. The MPEP at the Prometric site can be accessed very, very fast. The search function works well. I had some glitches with the chapter index on the side taking me to the wrong parts of the CFR and the USC which was frustrating (the location it took me to was not even close in several instances) and forced me to manually navigate the documents, which wasted a little time. I knew my chapters well and was able to do a look-up or at least confirm an answer in about 40 questions. No joke.

Reply

501 tomegoNo Gravatar March 18, 2014 at 8:35 pm

Thank you for the information. I am taking my exam next week. I will spend some more time with the PLTIA and post-grant review procedures. I have heard that there are less repeat questions than in past years so it is good to hear that you still got a fair amount of repeats.

Reply

502 Bfusion.No Gravatar March 19, 2014 at 8:40 am

Chuckles,

Congrats on the pass! Sad to hear that the software glitches are STILL happening on the exam (I heard form another individual their program crashed twice). This has been going on for over 2 years now. I contacted Prometric a few times to let them know (It crashed on me as well).

SO: if your program freezes on the exam, crashes, etc…don’t panic. Just raise your hand/get the proctor ASAP and they’ll rest for you. In 99% of the cases you won’t loose your test (I only heard of this happening once where the system deleted everything from an individuals exam).

Again, many congrats!
~Bfusion

Reply

503 RonNo Gravatar May 19, 2014 at 1:00 pm

Hello,

I am not up on the acronyms but what is PLTIA?
I would like to review the material that you mention but was
a bit confused.

Regards,

Ron

Reply

504 TexcoNo Gravatar May 7, 2014 at 5:22 pm

Got a preliminary pass today.
Read the 2013 PLI guide twice, read “the ultimate patent bar guide” twice, and went over all the questions posted on this site 3 times. Saw quite a few repeats from both new and old questions. Got by with only less than 10 search of mpep with both sections combined.

Some of the ones I can remember
Moon dust
Lipgloss
Potter

Tons on appeal, can’t recall the details, have to clear my mind.

Got PLI from eBay and tupbg from amazon. Willing to sell them cheap, if anyone interested. mdd200@qq.com

Reply

505 KansasmanNo Gravatar May 8, 2014 at 11:57 am

HI All,
Anyone has a recent LSAT course and is willing to exchange it with PLI updated course? Passed 01/2014

Reply

506 Bryan AdamsNo Gravatar June 3, 2014 at 12:35 am

I have some LSAT materials. Powerscore. I would be willing to exchange. I would have to mail them though.

Reply

507 Bfusion.No Gravatar May 13, 2014 at 9:03 am

Hey all,

A few patent bar exam thoughts for you all studying:

Content: Rumor on the street (can’t say which streets) is that the exam still has a lot of “familiar” content on the exam (can’t disclose details, but, there are a number of variations of previously tested questions still showing up).

Personal: depending on who you are/how you like to studying, sometimes just knowing someone is around/available to chat with or even bounce questions off of (not related to studying even) makes a world of difference. Again, this exam is part knowledge and part skill. You do need to know SOMETHING about the MPEP and rules/laws, and you do need to know HOW to take the exam. BUT, you also need to be mentally prepared, and sometimes this takes having someone talk with you and email you to re-affirm your abilities. Forums are a great place for this, and if you want directed, focused attention as well, there are places to get this in a study program.

Pass rates. The national average last year reported by the USPTO was 47%. That means of everyone who took the exam (which also means, used the various study programs and resources available..some of them $500-$3000), only 47% passed.

Wysebridge Patent Bar Review had an 84% pass rate (for our first time test takers) in 2013.
http://www.wysebridge.com/passrates

If you have any questions, please feel free to email / reach out (info@wysebridge.com)

All the best to you as you study,
~Bfusion

Reply

508 WelddoctorNo Gravatar July 8, 2014 at 7:19 pm

Hey All,

I’m glad I found this site thanks to a recent Patent Agent conversation. Has anyone been getting errors and typos as they have been studying with Omni?

I hope its not just me. I’ve found over 30 errors. No one is perfect but you would think they would correct them as students found them. Is there anyone out there using Omniprep?

How are the other Patent Bar Review Courses for Errors?

Reply

509 Bfusion.No Gravatar July 31, 2014 at 8:18 am

Hey all,

A few months back we had been discussing about an updated forums/place to collaborate, work on questions, discuss the exam, etc.

We are finally up and running (www.forum.wysebridge.com), with a simpler, easier to use interface and way to better keep track of questions, answers, and discussions (instead of long threads like here).

This site has been instrumental in a lot of individuals paths while studying, but as time has gone by and threads and information have become outdated, the need for a better place to collaborate and work on materials has grown.

So, please feel free to utilize, share, collaborate on our new platform!

All the best,
~Bfusion

Reply

510 nancyNo Gravatar September 23, 2014 at 5:45 pm

Hi , I have a question and I am pretty sure that the answers will be beneficial to others too. I read on USPTO website mentioning that one has to score 63 marks out of 90 to consider as pass as the 10 beta questions are not considered in marking. So 63 marks out of 90 is 70% marks in order to pass. So now at the end if someone sees their score as 57% that means the person is short of 6 marks to make it 63 as passing no. for making as 70%. Is 57% marks means one has scored 57 marks out of 90 or marks are different than percentage? Please confirm. Just wanted to clarify and consider review if needed. I in anyways appearing again but will be good to know. Thanks in advance for replying.

Reply

511 Bfusion.No Gravatar September 24, 2014 at 9:50 am

Hi Nancy,

Good question. Yes, you are correct, in the following:
1) 90 questions are scored
2) 10 are unscored
3) Thus, 63 correct out of 90 scored = 70% (Pass)
4) which means you can miss 17q out of 90 scored questions.

If you see a 57% at the end of your exam, this means that:
1) you correctly answered 51 (if the OED rounds up) or 52 (if they round down) out of 90 scored questions
2) This means you would have incorrectly answered 38 or 39 out of 90 scored questions.

Hope that helps! If you have any further questions, we’d be glad to help..and feel free to post on our forums as well at Wysebridge Patent Bar Review.

All the best,
~Bfusion
http://www.wysebridge.com

Reply

512 nancyNo Gravatar September 24, 2014 at 12:18 pm

Thank you, Byfusion. I really like your participation to keep everyone aware with your prompt reply . I can see you are very proactive towards everyone. Thanks in tons!!!
BR

Reply

513 PirloJuveNo Gravatar January 5, 2015 at 5:51 am

Does an appeal brief need to be signed? (not the notice of appeal)

Reply

514 PirloJuveNo Gravatar January 13, 2015 at 9:12 pm

§ 41.67
Appellant’s brief.
(a)
(1) Appellant(s) may once, within time limitsfor filing set forth in § 41.66, file a brief and servethe brief on all other parties to the proceeding inaccordance with § 1.903 of this title.
(2) The brief must be signed by the appellant,or the appellant’s duly authorized attorney oragent and must be accompanied by the requisitefee set forth in § 41.20(b)(2).

Reply

515 SamNo Gravatar January 21, 2015 at 12:53 am

I passed the exam on January 5. I took old exams (esp 2002/2003) and used Wysebridge. Some topics I remember from the exam:
– EFS, ePAS (I was unprepared for these)
– post grant review and inter partes review (know the differences)
– patent prosecution highway
– PCT and appeal
– patentability (particularly from MPEP chapters 700 and 2100).

While taking the exam, I had the impression I was not doing well and was surprised when I found out I passed at the end. So – don’t give up even if you feel like you blew it after the first half.

Regarding Wysebridge, I probably would not have passed without them. They have good downloadable AIA materials, which was the main reason I signed up. I also benefited from their basic guidance on how to take the test. An important point is to make sure to answer all questions and mark difficult questions for later review. If you have a feel for questions that would be easy to look up, mark those to review first. It helped me to try to answer all the questions from my own knowledge, and then use the MPEP after I answered everything. I also liked and used the answer sheet template.

Wysebridge also has a long list of repeat questions and a test database of old exam questions. The former was more helpful than the latter.

On the negative side, the Wysebridge forums were not particularly helpful, mostly because they are infrequently used. Also, there are broken links in the answers to questions and a few questions had unhelpful answers or even answers that seemed to be wrong. Further, while the AIA materials are good, many of their MPEP chapter summaries are not current. Overall, while Wysebridge is not perfect, it helped me pass the exam and I would recommend it to others.

Reply

516 NYSeaNo Gravatar January 21, 2015 at 4:00 pm

What was the test like in terms of repeats? I imagine there would be little to no verbatim repeats from the old exams, but would expect to see some AIA variants of them.

Congrats btw! Taking my exam on Sunday, hoping to have the same outcome as you!

Reply

517 Becky HureyNo Gravatar January 26, 2015 at 4:18 pm

Arnold & Porter LLP has an opening for a Patent Law Clerk or Patent Scientist in the Washington, D.C. office. The Patent Law Clerk or Scientist will be directly supervised by an attorney and get individualized patent law training. Ideal candidates will be organized, detail-oriented and able to follow up on instructions independently. Responsibilities include but are not limited to: preparing and filing new patent applications and other patent prosecution documents; conducting client technology and patent portfolio assessments; supporting business development and managing large domestic and foreign dockets.

Candidates should have a Ph.D. or significant graduate level experience in plant biology, molecular genetics, or biochemistry as well as excellent academic credentials, strong oral and written communication skills. Law school or prior legal training or enrollment in law school is not required. Relevant legal experience is preferred. Benefits include a competitive salary and a tuition reimbursement program when applicable. Please include a cover letter, resume, school transcript(s) and a writing sample with your online application.

Arnold & Porter is an equal opportunity and affirmative action employer that does not discriminate on the basis of race, religion, color, national origin, sex, veteran’s status, age, disability, sexual orientation, gender identity, genetic information, creed, citizenship status, marital status, or any other characteristic protected by federal, state or local laws. Our Firm’s policy applies to all terms and conditions of employment. To achieve our goal of equal opportunity, Arnold & Porter maintains an affirmative action plan through which it makes good faith efforts to recruit, hire and advance in employment qualified minorities, females, individuals with disabilities and protected veterans. If you would like more information about your EEO rights as an applicant under the law, please click here. Arnold & Porter LLP is an EO Employer – M/F/Veteran/Disability

Click here to apply: http://www.arnoldporter.com/jobs.cfm?u=PatentLawClerk&action=view&id=617

Reply

518 Becky HureyNo Gravatar January 26, 2015 at 4:19 pm

The Washington office is seeking a mid-level registered patent associate for the Intellectual Property Practice Group. Qualified candidates should have a minimum of 3 years’ experience in patent prosecution and preferably exposure to inter partes matters and evidence of excellent writing skills. Candidates must have a PhD in the life sciences, material sciences, or biomedical engineering fields. DC bar or ability to waive into DC bar required. All applicants must have excellent academic credentials, writing skills and references.

Arnold & Porter is an equal opportunity and affirmative action employer that does not discriminate on the basis of race, religion, color, national origin, sex, veteran’s status, age, disability, sexual orientation, gender identity, genetic information, creed, citizenship status, marital status, or any other characteristic protected by federal, state or local laws. Our Firm’s policy applies to all terms and conditions of employment. To achieve our goal of equal opportunity, Arnold & Porter maintains an affirmative action plan through which it makes good faith efforts to recruit, hire and advance in employment qualified minorities, females, individuals with disabilities and protected veterans. If you would like more information about your EEO rights as an applicant under the law, please click here. Arnold & Porter LLP is an EO Employer – M/F/Veteran/Disability

Click here to apply: http://www.arnoldporter.com/jobs.cfm?u=LateralPatentAssociate&action=view&id=565

Reply

519 AnnieNo Gravatar February 24, 2015 at 2:01 pm

I have questions regarding the various Patent Bar Courses available in the market. I cannot afford the PLI and therefore was thinking what is a good alternative to the PLI. So far I have had mixed and confusing reviews for OminiPrep, Wysebridge and PatBar courses. Would highly appreciate suggestions for a good review course.

Reply

520 julieNo Gravatar February 24, 2015 at 4:07 pm
521 yeonaNo Gravatar March 23, 2015 at 4:36 pm

I have to re-apply. I sent a new application about 15 days ago. How long will it take to get a New ID number?

Reply

522 OliNo Gravatar April 22, 2015 at 6:48 pm

I got mine back about 2 weeks. You will have a same Id number. And, schedule early if you want review last test!!! I just miss the date (65 days- 2 weeks) and I am so regret now…

Reply

523 Becky HureyNo Gravatar April 13, 2015 at 11:29 am

Arnold & Porter LLP has an opening for a Patent Law Clerk in the Washington, D.C. office. The Patent Law Clerk will be directly supervised by an attorney and get individualized patent law training. Ideal candidates will be organized; detail-oriented; and able to follow up on instructions independently. Responsibilities include: preparing and filing new patent applications and other patent prosecution documents; conducting client technology and patent portfolio assessments; supporting business development; managing large domestic and foreign dockets and other duties as assigned.

Candidates should have an advanced degree, preferably a Ph.D., and significant graduate level experience. Training in the fields of plant biology, genetics, microbiology or entomology are preferred though candidates having experience in other biological science fields also will be considered. Successful candidates will have excellent academic credentials and strong verbal and written communication skills. The ideal candidate will have passed the Patent Bar. Law school or prior legal training or enrollment in law school is not required. Benefits include a competitive salary and a tuition reimbursement program when applicable. Please include a cover letter, resume, school transcripts (please include each school), and writing sample with your online application.

Arnold & Porter is an equal opportunity and affirmative action employer that does not discriminate on the basis of race, religion, color, national origin, sex, veteran’s status, age, disability, sexual orientation, gender identity, genetic information, creed, citizenship status, marital status, or any other characteristic protected by federal, state or local laws. Our Firm’s policy applies to all terms and conditions of employment. To achieve our goal of equal opportunity, Arnold & Porter maintains an affirmative action plan through which it makes good faith efforts to recruit, hire and advance in employment qualified minorities, females, individuals with disabilities and protected veterans. If you would like more information about your EEO rights as an applicant under the law, please click here. Arnold & Porter LLP is an EO Employer – M/F/Veteran/Disability

CLICK HERE TO APPLY: http://www.arnoldporter.com/jobs.cfm?u=PatentLawClerk&action=view&id=617

Reply

524 ChrisNo Gravatar April 29, 2015 at 7:16 am

I would like suggestions on the best study method and materials for taking and passing the Patent Bar the first time. I work fulltime and do not have money and/or time to waste. Please give me any and all suggestions.

Reply

525 frustratedNo Gravatar May 20, 2015 at 1:13 am

I took the Reg Exam for the first time this weekend. I am curious if what I experienced, while TRYING to search the MPEP during the Prometic test, is normal. If you have taken the exam please share your experience.

The MPEP I had was slow to open and slow to change pages. The so-called search function failed to search for more than one word at a time, except when I entered “new use”, and it also failed to find words that were in a particular chapter. I called the OED and they told me to email the OED. So I did and am waiting for a response. I have asked that they waive the fee for my next attempt.

Has anyone had a similar experience or heard of the OED making amends for a inoperable MPEP?

Much thanks.

Reply

526 julieNo Gravatar May 21, 2015 at 3:12 pm

I can search but it so slowed… annoying

Reply

527 JulieNo Gravatar May 30, 2015 at 1:04 pm

I am not sure if I am correct:
pre-AIA separate oath/delcaration, each oath must state the names of all the inventors even if its only signed by one of the inventors.
But under AIA, separate oaths are proper with a single name of an inventor if ADS has all the inventors’ info.

THanks

Reply

528 shanthameenaNo Gravatar July 9, 2015 at 12:50 pm

It appears that the prometric center in 4030 Moorpark Ave #280, San Jose is closed and the prometric site shows up a new address 2665 north first street suite 207 san jose.

Please share your reviews for this location to take Patent bar exam.

Reply

529 ExhaustedNo Gravatar July 22, 2015 at 5:26 am

Just took the exam with the most recent updates. Repeats from 2000-2003 exam were super rare (maybe 2-4). However, I would definitely still recommend going over them until you can answer each without a problem because they are the basic concepts and will help you organize a map of the MPEP in your head when you do need to search during your exam. Obviously a lot of questions on AIA stuff, including the fact that if any claim is added to an application after March 2013 it will be analyzed under AIA (even if those claims are later canceled). There were a couple of questions (5-10) that were the same as mentioned above by others and throughout this site. Even had 2 questions regarding protests. From my recollection, majority of questions dealt with either chapters 600 or 700 with emphasis on oath/declaration, IDS, inventorship, 102&103 rebuttals and examiner’s responses/answers (falls in 1200 too). Overall there’s no easy route, it’s going to take lots of time and work before you can actually answer correctly without looking up every question and wasting time. I didn’t think I was doing great but received a provisional pass in the end so don’t give up in the middle and try to limit yourself and move on if you get stuck on a question and waste 5 minutes looking up the answer (chances are it’s a beta question anyway). I only used this site and some outlines of the MPEP I found online along with continuously doing questions and reading people’s comments on this site. Put in the work and do as many questions on as possible. Don’t take the test cause you want to get it over with, take it when you’re ready (unless you like spending $390)

Reply

530 JNo Gravatar August 12, 2015 at 7:37 am

TIPS: I WOULD LIKE TO SHARE MY EXPERIENCE AS TO HOW I PREPARED THE EXAM, AND HOW I TOOK THE EXAM. THE FOLLOWING CONTAINS BRIEF TIPS AS TO HOW YOU SHOULD PREPARE AND TAKE THE EXAM.

I just passed the exam yesterday on my first try. I have been studying for three months, and used PLI book without registering for the class. I also have studied 2003 April and October Old exams (but not 2002 exams)

OLD EXAMS: Just want to let you guys know that Old Exams (at least 2003) are mostly consistent with current MPEP. I have gone through each question, and compared the explanation of the answers with current version of MPEP, and have found 6~7 questions (per morning+afternoon exam) that are not consistent with current MPEP. That means that more than 90% of the questions are still relevant.

I have seen 5~7 questions exactly from the 2003 exam. I do not know how many questions are from 2002 since I have not studied those.

Though there are only 5~7 questions that are exactly same as those of old exam questions, I saw a lot of variant from old exam questions. Therefore, when you are actually studying old exams, do not try to merely memorize the correct answers, but try to understand why the answer is correct one. Once you understand every question in 2003 exams, you would be able to handle 20~30 questions in actual exam.

STUDY MATERIAL: I have bought 2015 PLI STUDY GUIDE without recent update, but it was not a problem to understand and spot the issue for questions. The only question that I guessed is about PPH system (I chose the lengthy one since correct answer tends to be lengthy than others).

For those of you planning to study for patent bar exam, I would highly recommend going over PLI 2015 many times since PLI is concise, but enough to contain all the relevant issues that will be on the actual exam.

ACTUAL EXAM: There are a lot of questions regarding obviousness, and I have really not touched the material since it is lengthy. However, search would be the key in solving those obviousness questions since obviousness questions tend to quote exact sentence or terms from MPEP (and those sentences and terms are rarely shown in other sections). Whenever you are facing obviousness question, open MPEP 2100, go to obviousness section, and search for the sentence or term quoted in the question.

So together with PLI 2015 + Old Exam + Searching Skills for Obviousness, I was able to understand all questions (except one about PPH). Most importantly, don’t get exhausted while studying, but try to refresh yourself often. I wish all of you the best luck, and hope this TIPS help.

Reply

531 RelievedNo Gravatar August 14, 2015 at 10:14 pm

the computer and the center told me that I passed today. So, I’m assuming that I passed. First, huge thanks to this website. Second, the old exams are by no means irrelevant; third, the AIA has two implement dates, remember them. And I think the thing is to really get behind the reasoning of things, for analyzing purposese.

My test center actually called me a day before the test, asking me to be there an hour, give or take, before the actual scheduled time. The process was as pleasant as tests could get, but the thing was that at my center, you have to start a new search of MPEP for each question. And loading of the material was slow, so be patient and wait for it to load. More clicks only cost more time, because the computer would think of each click as a new command. The center provided scratch paper booklet, which was more than enough for me. However, I still think writing down the question number and a quick note, along with the mark function of the test software, made it quicker for me to look for relative sections in MPEP.

Anyways, I read the MEPE page to page once, selective chapters twice, and repeated questions and concept discussion on this website until I got them. Hopefully, my experience might help.

Good luck to all of us.

Reply

532 LNKNo Gravatar September 23, 2015 at 4:27 pm

How do I look at all of the comments on this website?

Reply

533 Patent AgentNo Gravatar November 19, 2015 at 10:20 pm

I passed the exam today, and no longer need the PLI material I ordered about few months back. The online access is valid until mid June of next year. The material has all the latest material tested, and helped me immensely in passing the exam. I relied entirely on the material to pass. Email me if you are interested in getting all the material: pat.agent21 (at) gmail (dot) com.
Thanks!

Reply

534 JenNo Gravatar May 14, 2016 at 9:01 pm

Hi, I have a question for recent test takers, I notice when I look up things in MPEP, different web browser , (IE or chrome), gives different search function, what frustrates me is that in chrome the search starts with begging of the chapter again whenever I type in a new word, but I would like to search chapter heading, jump to the chapter then search for specific information, but chrome doesn’t do that.
anyone can tell me what exact browser used in Prometric bar exam? I think I need to practice with the same browser

Reply

535 BLawyeredNo Gravatar May 20, 2016 at 6:35 pm

It’s not a browser, it’s an older version of Adobe Acrobat, with each chapter as individual files. And it is SLOW as snails, so don’t depend on being able to look up more than a couple of questions. The search function will start looking from wherever you are currently at, with the option to search from that point up or from that point down.

Reply

536 BLawyeredNo Gravatar May 19, 2016 at 9:17 pm

I am looking for a Patent Bar tutor! email me if interested: BLawyered@outlook.com

I took the exam in April and did not pass. I am considering signing up for the Q&A review session for $195… has anyone done this??? Is it worth it?

Thanks!

Reply

537 JenNo Gravatar May 22, 2016 at 9:36 pm

Thanks a lot for your reply above. Do you still remember some questions on top of your mind, at least in what area are those questions?
If you do the exam again, do you need to mail transcripts and apply again like the first time? I hope not, I only have one copy of transcript.

Reply

538 BLawyeredNo Gravatar May 24, 2016 at 6:30 pm

I felt like every question was new AIA material with many many questions having relative dates just before AIA AND just after. So like… this thing happened on March 15th, 2013 then this other thing happened on March 17th, 2013, so which answer applies? Also, I’ve been using Wysebridge to study and often I can get the question right just looking at the answer options. This is NOT the case on the exam… the answers are either so similar or all completely different or more than one answer is technically correct so which is MORE correct.
Ugh.

Reapplying for the exam is exactly the same as applying for the exam, with the only exception being that you don’t need to resubmit your transcripts unless it’s been more than 1 year since you originally applied.

539 JenNo Gravatar May 25, 2016 at 12:09 am

Thanks Blawyered! I suggest you get PLI, it is more expensive, but it’s very good and it’ll give you a lot of practice on that kind of question.

540 kmpatbarNo Gravatar June 1, 2016 at 8:39 pm

Hi BLawyered,
I did not pass the exam in April as well. I did pass it today (second attempt). On my first attempt, I think I wasn’t prepared enough especially with AIA. I did go by the assumption that the exam is still primarliy/heavily pre-AIA tested. That doesn’t seem to be the case anymore.

Having said that, I did take the review session. I knew I had missed by just 1 question, I got a 68% on the April exam. My intent of taking the review session was to try and solve the incorrect questions, once I had studied enough and since the database is not that huge,I was hoping to find some repeats from the incorrect ones on the second take. And I did get around 4-5 repeats from the incorrect ones. That saved me some time today and some lookup work too. Although, I think the review session is too expensive. I’d rather just study well (PLI is awesome for studying).

Thanks !

Reply

541 JenNo Gravatar May 24, 2016 at 12:09 pm

Hi everyone,
Just to share with you a search tip, for Derivation, PGR, IPR, and CBM, I find it easier to search the rules instead of search the federal register. Hope this helps.

Reply

542 JenNo Gravatar June 2, 2016 at 11:54 am

when I search I like to use the bookmarks appearing at the left side of window, it has titles and easy to jump around topics, can anyone tell me during the exam whether this bookmark window available? anyone knows which version of pdf they are using? the version I use are not highlight the searched word, it is a bug for certain version.

Reply

543 BLawyeredNo Gravatar June 7, 2016 at 6:26 pm

Yes, there is a bookmark window that you can jump to sections with. It is PDF viewer, but does not indicate which version (I tried to click around to see if a properties tab would pop up, but no luck). I took the review session today. I missed 40 questions. They give you 2.5 hours to review your missed questions and the correct answer to those. I received my new OED ID# today, and have from 6/10 to 9/8 to take the exam (again). Anyone want to study with me?

Reply

544 CraigNo Gravatar June 8, 2016 at 9:34 am

Wow Blaweyred you are at least in striking distance and don’t be discouraged as many people don’t pass.

Are you and Jen interested in forming an email group? We could post topics to study and hard deadline dates to meet our goals.

545 MeilinNo Gravatar June 8, 2016 at 1:29 pm

Hi Jen, I like your idea of studying together. I’m planning on taking the exam in a few weeks. How about starting a group on google hangout? If you let me know your gmail account, I can start one. Thanks, -MH

Reply

546 JenNo Gravatar June 15, 2016 at 3:54 am

Let’s start a google hangout! or any other way to have a discussion group is fine. Sorry, I haven’t been to this website lately, coz I was on a trip. do I have to have a gmail ? or any other email will also do?

547 JudyNo Gravatar June 5, 2016 at 10:37 pm

Hi! I’m applying for the patent bar exam for the first time and there is a form 2038 that I need to submit in order to use my credit card to pay for the application fees. Does anyone know how to indicate “description of request and payment information” last section on the form? The General Requirement Bullet says to check “other fees” and write application fee, but I don’t see a blank space on the form to do that.

Thanks!

Reply

548 CraigNo Gravatar June 8, 2016 at 3:29 pm

Hi Meilin,

Please start one! my email is all one word “cbellon36” and I am at gmail.

Reply

549 PatagentToBeNo Gravatar June 8, 2016 at 6:08 pm

Craig, I just invited you on the google hangouts. -MH

Reply

550 kmpatbarNo Gravatar June 13, 2016 at 5:30 pm

Hi,
I passed the exam on June 1, 2016. I have PLI materials to sell, if any one’s interested. I bought the materials in 2015, and I do have the mar 2k16 updated supplement from PLI. I do have some markings on the material, so ready to sell for a pretty cheap price. If interested, please email me at mail_at_krish@yahoo.co.in

Reply

551 MichelleNo Gravatar June 13, 2016 at 8:34 pm

If anyone has the updated PLI materials for sale, please email me, ychen1026
I am at gmail.

Thank you!

Reply

552 JenNo Gravatar July 2, 2016 at 12:00 pm

hi Blawyered, how long do I need to wait to hear back fro OED about my application for registration exam? anyone knows? I sent in my application two weeks ago, still didn’t hear anything from them.

Reply

553 DaveNo Gravatar July 2, 2016 at 4:18 pm

Jen, give it a little longer than that. If you have a simple app (science or engr degree) I bet you will hear back in the next couple weeks. If you had to go Option B, maybe a little longer.

BTW, have you found any good sources for practice post-AIA questions. I am taking it in a couple weeks and am running out of post-AIA questions. From what people are saying, it doesn’t seem like those old exams are super helpful anymore.

Reply

554 JenNo Gravatar July 2, 2016 at 8:43 pm

Hi Dave,
Thanks for your reply on the time frame, I’m going for the option A, hopefully I’ll hear back from them in a couple of weeks, didn’ t expect for them to take that long.
I use PLI study material and they do have lot of post AIA questions, I know wysebridge has post AIA practice material, I think old exam are still helpful for study, whatever they test in old exam, you need to know the new and old, in that regard, I think it still quite helpful.

Reply

555 olddognewtricksNo Gravatar July 14, 2016 at 3:43 pm

Had a question on the prometric exam. Understand you have to search for chapters using control F function and its like an old version of adobe. My question is do the search terms have to be single words or two words that are consecutive? Are any boolean searches allowed such as a word one within 5 words of word 2? etc. Thanks for the help.

Reply

556 Austin PatentNo Gravatar July 19, 2016 at 4:25 pm

If anyone is interested, I am selling my PLI Patent Bar Materials (purchased February 2016), which I found really great but decided not to take the Patent Bar after all. If interested, email me at austiniplawyer (at) gmail (dot) com.

Reply

557 CubingNo Gravatar July 31, 2016 at 9:26 pm

I am selling my 2016 PLI Patent Bar Materials. It is recently purchased and has all the most up to date supplements. I passed my exam and don’t need it anymore. Email me at cubing98 (at) gmail (dot) com.

Reply

558 CraigNo Gravatar August 1, 2016 at 9:45 am

Sorry, I’ve been away for a bit. I’m sending in my application papers tomorrow under option B. My undergrad was non-science, but I have a ph.d. in mo bio. I don’t get why they rely only on my undergrad. I’m sending in my ph.d. transcripts regardless. I hope they don’t give me any problems….

Reply

559 SteveNo Gravatar August 19, 2016 at 5:52 pm

Passed on the first try!!! I used PLI, which I thought was a great program for teaching you the nuances of the MPEP material, which is what I found useful for answering the questions correctly. I am now wanting to pass along the program to another for a significantly reduced cost (Paid 3K). Anyways, I am only selling the hard copies of the program that I purchased early 2016. I cannot give you my access to PATWARE, the online portion of the program, which has the video lectures and the extra practice questions. However, the hard copy of the program is extremely thorough, has a ton of practice questions, and I feel one could readily pass the patent bar through studying this material and working either the questions found on this website or possibly using the Wysebridge question bank, which I spent some time using before I switched to PLI (not a statement against Wysebridge). Please e-mail me at sandrewrogers(at) gmail (dot) com with a reasonable offer if you are interested in purchasing the course from me. I also don’t mind sharing some tips because I wish I had someone to personally advise me through this silly exam. However, prepare for a lot of material if you do because it is a great course.

Reply

560 IP JohnNo Gravatar August 29, 2016 at 5:34 pm

Hello,

I am new to posting in this forum. I have failed the exam twice now. Once in June and then today. My exams both times have had ~5 repeats. Is this normal? I have been using OmniPrep and this website to study. Today, I am at a loss…how should I move forward? I have seen lots of posts by people saying that they saw a lot of repeats. I certainly have not. I believe that everyone gets a random exam generated by the USPTO. Maybe I have been really unlucky or maybe not!

Reply

561 AlexNo Gravatar September 20, 2016 at 8:15 pm

Hi folks, I just passed this patent bar yesterday. It is not as difficult as I expected during my preparation.

Now I want to sell my 2016 PLI study guide. I believe it has all the latest changes since I bought it this June. Please offer me a price at alex.patent.1991@gmail.com if you are interested.

This study guide has over 1200 pages study materials, including guideline and teaching points for each MPEP chapter, practice exam and exercises and the supplement material for Post AIA stuff. Spend 2-3 months with it and you should be able to pass easily. The only things that are not included are the online video lectures and the Patware software, which I think are not very important. I spent too much time listening to the stupid lectures…

Reply

562 R--ANo Gravatar September 21, 2016 at 11:18 am

Hi all,
I did my test yesterday and unfortunately failed. I did what my friends did about 5-6 years ago and studied all the questions (almost 1800) and the class material from Patent Resource Group, INC. plus attending their class which is a week long class and cost around $3000. According to my friends most of the questions would be repeat questions but they were not. I just saw one question which was repeat. I never felt so stupid before because I actually spent good amount of time preparing for this test. I did not just show up for the test and I have a very good record of education so I cannot be that bad. Am I doing something wrong? Should I study other materials? Appreciate your help!

Reply

563 R--ANo Gravatar September 21, 2016 at 11:35 am

Hi,
Both me and my friend did this bar test in the past month and failed. I personally saw one repeat question and he saw only 2. So you are not alone. Another friend took the exam back in 2010 and he said 70-80 percent were repeat questions. I guess we have either been extremely unlucky or the bar exam is not the same as it used to be, or the class we took by Patent Group Resource, INC. is not as good as PLI classes which are talked about by many in this forum. Who knows?

Reply

564 Marie!No Gravatar October 1, 2016 at 9:49 pm

@steve I am currently using Wysebridge, can you elaborate on what made you change to PLI?

Reply

565 dunnoNo Gravatar October 5, 2016 at 8:48 am

Took the patent bar at prometric yesterday (10/4/16), and got preliminary pass on the first attempt! Yea!! Thanks for PLI, and this site. Great resources. Some reflections:
– AIA is HEAVILY HEAVILY tested. 102 questions are almost COMPLETELY AIA. Very few pre-AIA, in terms of 102.
– Very few repeats from prior exams. For me, <10 out of a total of 100 Qs.
– Some very complicated and convoluted fact patterns, with lots of dates involved. Had to draw timelines on the real test, even though I never use scratch paper when preparing the test!
– 102/103, PCT, appeal, O/D were heavily tested. Some post grant proceeding questions too, not many and not so difficult.
– MPEP pdf sucks. Not clear prints, limited functionalities. I did search some (<10) and corrected my answers based on the search (~4-5 max). Personal take: do NOT rely on searching through MPEP for this test!
– AM session and PM session were SO different! I guess all 10 beta Qs were in my AM session. Several Qs really throw me off! Thanks god I kept going and stayed for the PM session, which was WAY easier than the AM session.
– Some advice: do the PLI patware as much as you can. Remember the 102 dates inside and out as best as you can… AND, stick to the test even if AM session drives you crazy!!

Good luck guys!

Reply

566 Will take in 3 weeksNo Gravatar October 13, 2016 at 2:05 pm

Hi R-A,
I took PRG too and find most of their stuff is old. At the same time, a lot of there questions test areas of the MPEP that have not been closed off. Thus, I think PRG questions still have a level of relevancy – I will see soon. Most prep courses out there are weak on AIA. From what I’ve heard from others, most of the studying comes down to knowing the MPEP, knowing where to go, and quickly eliminating wrong answers. PRG doesn’t teach the MPEP at all, which is what I found to be a major gap in my study. Thus, I dived really hard into the MPEP. We’ll see what happens.

If the MPEP pdf sucks on test day, I’m sure the pass rate would be even lower than it is.

Reply

567 Will take in 3 weeksNo Gravatar October 13, 2016 at 2:20 pm

In other words, if you really study PRG well, you should easily score 50%. If you also study the MPEP just as well, you may score at least 80%. This is according to people who share a similar experience as yours…. maybe PLI just does an overall better job of combining both.

Reply

568 PLI RocksNo Gravatar October 21, 2016 at 4:10 pm

All:

I recently passed the exam on my first try, thanks to PLI. I am selling my up-to-date 2016 PLI materials (hard copy stuff only, purchased in July 2016) for $600 (+ shipping cost). The materials are in great condition, with no highlighting etc. Email me at patent.xterminator (at) gmail (dot) com. Thanks.

Reply

569 Michael ENo Gravatar October 23, 2016 at 9:39 pm

Just updating now, as I need a refresher before starting my clerkship. Passed the exam on my first attempt in July, before starting law school. PLI really, really helped me. The published USPTO practice exams are simply too old at this point to be worth your time – PLI has updated models. I’m not sure if they’re available outside the full (expensive) program, but if they are, that would also be a good idea.

Like ‘dunno,’ my exam drew exceptionally heavily from AIA material, especially in the AM session, as I recall. I don’t think I had a single pre-AIA question in the front half.

Mypatentbar.com is a terrific resource, though only becoming more and more difficult to manage as you have to scroll further down the page. Good luck to all.

Reply

570 JNo Gravatar November 1, 2016 at 12:27 pm

I passed the exam yesterday on my first attempt and thought I would add my thoughts and tips to this page as I really appreciated reading all of yours.

I worked as an office aide and eventually a paralegal at an IP Law Firm starting in college. I didn’t plan on becoming an agent until it was recommended to me by colleagues in my office since I had a technical background. This gave me a HUGE advantage over your typical candidate going into this exam as I was very familiar with a lot of tested topics already. My reading comprehension on this exam was also at a higher level as I have been living and breathing most of the vocabulary over the last 5 years working at said law firm. I expected to get through it quickly and with ease. That being said, this exam was STILL a race against time and very stressful for me!

I used PLI to prep for this exam and I think their Patentware data bank was ESSENTIAL for me passing this exam. Their reading materials were also very thorough and complete. Their lectures were nice but not necessary. I found the lectures that pertained to reviewing their practice exams to be the most helpful. The lectures over corresponding MPEP chapters was time better spent just re-reading the material in their notes.

This was my study plan: I started studying about 4 months out about 1-2 hours a day and eventually ramped it up to 3-4 hours a day with many hours on the weekends (10-20 hours) in the last 2 months leading up to the exam. Anything I felt I didn’t have down, I made a flashcard for and reviewed these flashcards regurlarly. I took every practice exam PLI offered except the old exams (more on that later) and used their patentware to customize exams on sections I was weakest on (They give you these statistics!). I also re-read sections within their materials or re-watched their lectures on these same sections until I felt better about them.

Regarding the old exams: They are nice to get a feel for how actual exam questions will look and can be a good measure of your understanding but don’t expect to see repeats. I honestly remember only seeing 1or 2. Your time is better spent elsewhere. I didn’t go over every old question but did go over the repeat questions listed on this site. I recommend only going through the old exams once you feel you are “ready”. I went over them in my last prep week.

PDF in the exam: I had a lot of questions and worry about the search function going into the exam because of horror stories I had heard but I had no issues with it during the exam. It loads up relatively quickly (1-2 seconds) and navigating between chapters is via a drop down menu with the subsections being listed on the left side once you’ve opened a chapter. I realized that you can’t be impatient with it. If you try clicking within the screen as it’s loading it will freeze up but not for more than 10 seconds (but still, that’s 10 seconds!). So just be patient and you’ll be fine there.

Most important topics: Focus on AIA, you won’t get many pre-AIA questions (maybe 1-2). Focus on Appeals, PCT, Chapters 700 and 2100, PGR, IPR, Reissue, Reexam. I honestly opened up 2100 more than any other chapter during the exam. But honestly, questions with answers originating from 2100 are often verbatim language and can be found using the “find” feature very easily.

My Exam experience: Since I am a fast reader and test taker, I knew I would have some time to look up questions at the end. While going through the exam initially I looked up every 3rd or 4th question just to confirm an answer because I knew where to look. KNOW WHERE TO LOOK FOR THINGS! I marked questions I wanted to look up at the end if I had time. I had about 40 minutes at the end of the AM session and about 25 minutes in the PM session.

I can’t speak to any other prep courses but PLI was amazing and I recommend to anyone who is looking to take this exam. If you do the course as directed you will do just fine. Good luck to everyone!!

Reply

571 DaveNo Gravatar December 21, 2016 at 3:56 pm

Just took the “new” test and it was awful compared to the one I took this past summer. 1 or 2 repeats. Questions are written to trick you, so you need to strain your eyes and read very carefully. Several questions with blatant potential typos that would change the answer depending on the correction. Studying the available questions from the existing released exams or the posted questions was useless, as the modern questions and answers are very long (there were hardly any “gimme’s” like there are throughout the available study questions). The USPTO clearly saw a need for lot of practitioners in the early 2000’s (upwards of 40% questions released were repeated in the 1999-2003 exams) and in the last couple years must be trying to cull the number of practitioners entering the profession (with 5% of questions repeating and four times the material tested). Not going to stop me from trying to beat this thing though…good luck everyone!

Reply

572 IggyNo Gravatar January 3, 2017 at 6:01 pm

Dave- Can you tell me which areas of the MPEP were tested the most? Also, how much did you study and which course, if any, did you use? Thanks!

Reply

573 DanaNo Gravatar February 4, 2017 at 3:36 pm

Just got my preliminary pass.
Some thoughts:

1. The morning session was terrible. Terrible! The questions were long (one needed to scroll down to see the answers) and tricky (either because there was so much detail or the subject matter was just absurd). I ended up desperately searching the MPEP for many of the questions (mainly without success) and made a lot of best guesses. Was running out of time to confirm more answers.
1 or 2 repeat questions from prior exams.

After that terrible experience, I didn’t review any notes in the break (since it felt nothing in my notes could have prepared me for this level of detail) and took a walk instead to clear my head.
The afternoon was much better. Again, barely any repeat questions. Mainly AIA. All questions were way shorter and easier.

2. Conditions overall
I was provided with a whole booklet (6 or 8 pages of paper), more than I needed.
The room was freezing (63F, but felt colder). Used the 15 min in the beginning to write out 102 and pre-AIA 102 for reference instead of engaging with the tutorial.
Also, I came back earlier from my lunch break, and could presumably also have used a couple of minutes to write stuff down at that point.

3. MPEP
The resolution is terrible. One really has to zoom in to be able to read anything. There was no “whole word” or case specific search function. Just find and repeat find.

4. New functionality
It seems there is now a new function that lets you highlight question text (not answers) and cross out wrong answers. Very handy. No need to take notes on paper anymore.

5. Breaks
The whole check out, bathroom, security check, check in takes a couple of minutes. You might have to wait as others need to go through the same process. I avoided bathroom breaks.

6. Studying
I took the PES course and studied for 3 months. What I found most helpful was their exam engine. Even though there were very few repeat questions, the practice questions were similar to the exam questions in that they required one to pull knowledge from all kinds of different knowledge areas. That really helped me to “connect the dots” and understand broader concepts.

Good luck everybody!

Reply

574 Ashley LiuNo Gravatar February 20, 2017 at 5:10 pm

does your materials include the PATWARE access?

Reply

575 AlexNo Gravatar March 15, 2017 at 1:39 pm

I just passed my exam two days ago on my first try. I would like to share some thoughts with you guys.

I used Wysebridge.com and MPEP. I know that there is alert about not worth reading MPEP. But, still, I felt uncomfortable to just remember gist instead of knowing the big picture about the whole process of patent examination. Besides, as a non-native English speaker, I found it is truly necessary to dive myself into reading, especially those paragraph-long sentences. It was painful at the beginning, but paid me back in the test. In each session, I have around 35 minutes to re-check marked questions. Seven to eight months are needed to go over all 29 chapters. So, it really depends on your own choice.

I should say Wysebridge has done great job on arranging the massive materials in MPEP in a highly organized way. So, for a given chapter, I can read the most important points in Wysebridge’s review, and then read the corresponding chapters/sections in MPEP. Also, their AIA-Info is really helpful. Also, their practices are also informative. If you are on budget, it does not hurt to try Wysebridge anyways.

About the exam, like many people have said, pre-AIA stuff can be ignored. Except one question about pre-AIA 35 U.S.C. 102(b), all other pre-AIA-related questions are about the transmittal case, i.e. you have parent application filed before Mar. 16, 2013, but file a CIP after that date, which claims the benefit the previous one but contains some new claims. Later these new claims have been withdrawn. You should use pre-AIA or AIA. So, please read 2150-2153 very very carefully.

Don’t worry about the date of Dec. 8, 1993, Jun. 8, 1995, Jan. 1, 1996. Exam does not care about them anymore.

Good news for test-takers is that there would be at least 20 questions on AIA 35 U.S.C. 102(a)(b), but most of them are way easier than pre-AIA 102(a)-(g). PGR, IPR, Ex Parte Reexamination have about 10 questions. Please note that they, in most cases, entangle together. So, when you read 37 CFR 1.510-1.570, 42.100, 42.200, please compare them together.

Thank God that I just got about 3 questions about computer-based questions. I really don’t know what they are talking about. So, if you have resources, please practice this part a little bit.

I have some thoughts about the learning strategy. Although you should put most of your time into 2100, 700, 600, 1200, 1400, 1800, I would suggest that you should get familiar with some short chapters, such as 300, 1300, 1600, 2500. Since the questions about these chapters, if any, are usually shallow. If you are familiar with them, it is easy to grasp a point and save your time. I mean, the Exam is not only about how many questions that you know how to find related MPEP text, but also about how many questions you can answer correctly in no time. Again, this may be just for me. So take my suggestions carefully, please.

Good luck. Hope you guys pass your exams. Thank you.

Reply

576 fiscaliaNo Gravatar March 21, 2017 at 1:45 pm

I’m prepping for my test scheduled April 8. Today I found the USPTO Prometric tutorial module online, here: https://www.prometric.com/demos/uspto/starthere.htm

I don’t know about you folks, but there’s some useful info in there! It lets you at the very least see a peek at how the MPEP will be displayed and the controls, though you don’t actually get to interact with the MPEP viewer.

Reply

577 EKNo Gravatar October 23, 2019 at 6:14 am

This like to the Prometric USPTO exam tutorial does not work. Does anyone have any updated link to an official (Prometric, USPTO) registration exam tutorial? Thanks!

Reply

578 jgNo Gravatar November 25, 2019 at 9:54 am

The link has been fixed – took a long email discussion with prometric to get it back up and running.

579 TFNo Gravatar March 31, 2017 at 7:48 pm

OmniPrep relies on old test questions which seem to no longer be tested. They also don’t explain material. The new test seems to depend upon searching technique and proficiency as well as proficiency in new AIA material.

Reply

580 Felix3142No Gravatar May 21, 2019 at 11:13 am

Selling the PLI Patent Bar Study Guide 2018 for $350. It’s in excellent unmarked condition. Find the listing for “PLI Patent Bar Study Guide 2018” on eBay. Also selling the 2016 guide for $200, well marked but it has useful notes from the lectures and material wise is nearly identical to the 2018 guide (and has the same questions on all quizzes). I used my 2016 guide to pass the exam in 2019.

Reply

581 Claudia YeNo Gravatar June 11, 2019 at 7:39 pm

do you still have the 2018 PLI book? I will buy it.

Reply

582 BiancaNo Gravatar October 2, 2019 at 12:06 pm

Hello, when did you take the exam in 2019? I am about to take it and I have questions.

Reply

583 Becky DavisNo Gravatar September 26, 2019 at 11:36 am

Arnold & Porter, an international law firm based in Washington, DC, has an opening for a Patent Law Clerk in the Washington, DC office. The Patent Law Clerk will be directly supervised by an attorney and get individualized patent law training. Ideal candidates will be organized; detail-oriented; and able to follow up on instructions independently. Responsibilities include: preparing and filing new patent applications and other patent prosecution documents; conducting client technology and patent portfolio assessments; supporting business development; managing large domestic and foreign dockets and other duties as assigned.

Candidates should have a Ph.D., and significant graduate level experience. Candidates with broad training in the biological sciences are sought, with expertise in plant biology, biochemistry, genetics, chemistry, microbiology, entomology or chemical engineering preferred. Research experience or training in RNA silencing, genome editing, nucleic acids chemistry, microbiomes, quantitative genetics, process and reactor design, separation technologies, thermodynamics and kinetics is desirable. Successful candidates will have excellent academic credentials and strong oral and written communication skills. Law school or prior legal training or enrollment in law school is not required. Benefits include a competitive salary and a tuition reimbursement program when applicable.

Please include a cover letter, resume, school transcripts (please include each school), and writing sample with your online application.

Arnold & Porter is an equal opportunity and affirmative action employer that does not discriminate on the basis of race, color, creed, religion, national origin, sex (which includes pregnancy, childbirth, breastfeeding and related medical conditions), age, marital status, sexual orientation, gender, gender identity, gender expression, transgender, disability, medical condition, family leave status, citizenship status, ancestry, genetic information, military or veteran status, or any other characteristic protected by local, state, or federal laws, rules, or regulations. Our Firm’s equal opportunity policy applies to all employment practices and terms and conditions, including, without limitation, recruitment, employment, assignment, training, compensation, benefits, promotions, disciplinary action and terminations. To achieve our goal of equal opportunity, Arnold & Porter maintains an affirmative action plan through which it makes good faith efforts to recruit, hire and advance in employment qualified minorities, females, individuals with disabilities and protected veterans. If you would like more information about your EEO rights as an applicant under the law, please click EEO is the LAW and the Supplement poster.

Arnold & Porter is an EO Employer – M/F/Veteran/Disability/Sexual Orientation/Gender Identity.

Click here to apply: https://www.arnoldporter.com/en/careers/professional-staff/current-opportunities

Reply

584 ALNo Gravatar October 8, 2019 at 11:36 am

hello,
Selling my 2019 PLI Study guide for $350. You can find the listing on ebay.

Reply

585 JoNo Gravatar October 28, 2019 at 10:23 am

Has this book already sold?

Reply

586 KatNo Gravatar November 22, 2019 at 7:08 pm

looking to buy PLI exam Study Guide (any year after 2015). Please email me at yoonssoj@hotmail.com if you are selling one!

Reply

587 StevenNo Gravatar December 30, 2019 at 2:49 pm

Just passed the patent bar!
Some suggestions: Focus on AIA, RCE, 700&2100.
Sell my 2019 PLI patent bar materials!

Reply

588 CharleneNo Gravatar December 30, 2019 at 9:13 pm

Hello Steven, I’m interested in buying the materials. Can you please email me the details at jptemail@yahoo.com, if they’re still available? thanks!

Reply

589 JasonNo Gravatar January 6, 2020 at 3:43 pm

Steven, Congrats!

I am interested in buying 2019 PLI patent bar materials. Plz email to jasonliu3@gmail.com if it is available.

Reply

590 CarlaNo Gravatar January 11, 2020 at 6:23 pm

Hello,

I am looking for PLI study guide 2017 or later.

Please email me at cmecolikamp@gmail.com if you have one to sell.

Thank you!

Reply

591 VictoriaNo Gravatar February 22, 2020 at 8:27 pm

After four months of part time studying, I recently (Feb. 2020) took the exam and passed on my first try! FYI I had no pre-AIA 102/103 questions.

There’s already a lot of advice on this website on what to study (just do ctrl-F for “tips”) but for guidance on specifically HOW to study, I brain dumped some tips in an article here. It’s a lot of “what I had wish I had known” when starting to study. The article also has links to my flow charts, glossary of terms, and flash cards. Feel free to share!
http://www.pocketfuloflint.com/2020/02/21/practical-tips-for-passing-the-patent-bar-while-studying-part-time/

Reply

592 vtlimNo Gravatar March 16, 2020 at 3:07 pm

Selling my 2019 PLI materials. In great condition — no writing or highlighting. Email me at hydrargyrumate [at] gmail [dot] com. $800 or best offer.

Reply

593 AnnaNo Gravatar May 11, 2020 at 4:26 pm

Hi, I am looking for PLI study material 2018 or later. Please email at a.blossom123@gmail.com. Thank you.

Reply

594 Gangotri DeyNo Gravatar June 12, 2020 at 12:44 pm

Hello all,

I registered for the exam 19th May 2020 and got the letter from PTO yesterday 11th June 2020. I think there are a few changes. I have 6 months window to apply for the exam. i.e till December 12th. Also, I cannot find any date that is available before October 2020. It is a good time to prepare but I was hoping for an earlier date. If you plan on taking the test anytime soon I suggest you should register right away and waste no time. Also I would invite you for a group study in case you are interested. Please email me at holyriver6@gmail.com. Thanks, Dey

Reply

595 Shade LanzNo Gravatar August 23, 2020 at 4:00 pm

Hi all,

This is indeed a great forum! I am scheduled to take the bar in October and quite confused on which affordable prep course to got for. I can’t afford PLI, so I am currently leaning towards PatBar, but I will greatly appreciate any insights.

I need to lock down a course in a few days, so get on with my studying.
PS: I took patent law in my 3L and did very well, so I have some background knowledge.

Reply

596 ARNo Gravatar August 26, 2020 at 7:16 am

Shade,

I’m selling my 2020 PLI study guide and pre course materials for 1200 or best offer and can ship this out immediately. Please note that this will not include the course lectures or access to the patware questions.

I took a practice course before PLI and there is truly nothing comparable. I failed the first time and passed right away with ease after taking PLI!

Send me a message if interested!

Reply

597 Richard HoffbergNo Gravatar February 8, 2021 at 9:19 pm

Is anyone selling PLI study guides and course material? I am interested in buying. Email me at richard.hoffberg@gmail.com

Reply

598 AspenNo Gravatar February 9, 2021 at 10:15 pm

I’m studying to take the Patent bar, and I’ve started using PatBar, but am considering switching to PLI because there really isn’t much structure to the PatBar course. Can anyone comment on the structure that is in the PLI course? Such as, are there lectures that correlate to readings?

Reply

599 Austin WinterNo Gravatar February 17, 2021 at 7:14 pm

Yes for PLI ALL video lectures correlate to the readings. I will read the study guide then watch the videos. They also go very in depth regarding practice questions and mini-exams based on each chapter. Everything is clearly laid out and how long each lecture is. They also have a meter showing course progress and pace however the meter that reflects your pace isn’t all that accurate if you accidentally leave the course window open. Let me know if you have any other questions about the PLI course, I am currently taking it.

Reply

600 LTNo Gravatar February 22, 2021 at 6:17 pm

Hi, I just passed and is now trying to pass off my PLI materials. If you are interested please email me at leott58 [at] gmail [dot] com. Thanks.

Reply

601 Gangotri DeyNo Gravatar March 18, 2021 at 7:41 pm

Congrats. Do you recall any of the questions?

Reply

602 Austin WinterNo Gravatar March 18, 2021 at 9:17 pm

were there any Pre AIA questions or transition cases?

603 Constantine FirmeNo Gravatar March 22, 2021 at 1:51 am

How much would you like for the PLI course? Is your login for watching videos and going into the PLI site still valid? Please message me at ticktokewatch@gmail.com Thank you.

Reply

604 AndrewNo Gravatar March 26, 2021 at 10:01 pm

Hi everyone,

I just passed the exam on my first try using PLI materials. If you are serious about passing the exam, PLI is the gold standard and without it I couldn’t have passed. I’m looking to pass on the materials to someone else! Email me at fishboy50@gmail.com if you’re interested.

Good luck studying everyone!

Reply

605 CarrieNo Gravatar March 29, 2021 at 5:01 pm

I am looking for a study partner. I plan to take the patent bar in the next 3-4 months. If anyone interested, please reply: catear009 at gmail.com.

Thank you!

Carrie

Reply

606 JamesNo Gravatar June 5, 2021 at 8:06 pm

Selling my PLI 2020 binder plus all the practice exams with answers in PDF form. $500. Will pay for shipping.

Reply

607 Priyanka pariharNo Gravatar July 12, 2021 at 11:13 am

Hi,
I am looking for PLI course material. If anyone interested in selling please email me at priyankaggpt1@gmail.com.

Reply

608 bettyNo Gravatar July 17, 2021 at 4:43 pm

I’m looking for PLI study materials. Does anyone have any for sale?
Please and thank you.

Reply

609 Arnold & PorterNo Gravatar December 17, 2021 at 5:15 pm

Arnold & Porter, an international law firm, is seeking a Life Sciences Patent Law Clerk for the Intellectual Property practice group in the Washington DC office. The Patent Law Clerk will be directly supervised by an attorney and get individualized patent law training. Ideal candidates will be organized, detail-oriented and able to follow up on instructions independently. Benefits include a competitive salary and a tuition reimbursement program when applicable.

Please include a cover letter, resume, school transcripts (please include each school), and writing sample with your online application. Incomplete applications will not be considered.

Responsibilities include but are not limited to:

Preparing and filing new patent applications and other patent prosecution documents.
Conducting client technology and patent portfolio assessments.
Supporting business development.
Managing large domestic and foreign dockets.
Other duties as assigned.
QUALIFICATIONS:

Ph.D. and significant graduate level experience.
Broad training in the biological sciences are sought, with expertise in plant biology, biochemistry, genetics, chemistry, microbiology, entomology, immunology or chemical engineering highly preferred.
Research experience or training in RNA silencing, genome editing, nucleic acids chemistry, microbiomes, quantitative genetics, process and reactor design, separation technologies, thermodynamics and kinetics is desirable.
Excellent academic credentials and references.
Law school or prior legal training or enrollment in law school is not required.
Ability to maintain a high level of confidentiality.
Strong problem solving skills.
Ability to prioritize and manage time effectively working in a fast-paced environment while exhibiting a strong attention to detail.
Excellent organization and communications skills, both oral and written.
Ability to handle a variety of tasks simultaneously.
Exceptional client service both internal and external.
Arnold & Porter Kaye Scholer LLP is an equal opportunity and affirmative action employer that does not discriminate on the basis of race, color, creed, religion, national origin, sex (which includes pregnancy, childbirth, breastfeeding and related medical conditions), age, marital or partnership status, familial status, sexual orientation, gender, gender identity, gender expression, transgender, physical or mental disability, medical condition, family leave status, citizenship status, immigration status, ancestry, genetic information, military or veteran status, or any other characteristic protected by local, state, or federal laws, rules, or regulations. All qualified applicants will receive consideration for employment without regard to any characteristic protected by local, state, or federal laws, rules, or regulations.

Arnold & Porter Kaye Scholer LLP endeavors to make http://www.arnoldporter.com accessible to any and all users. If you would like to contact us regarding the accessibility of our website or need assistance completing the application process, please contact Director of Support Staff Stephanie Denmark at +1 202.942.6068. This contact information is for accommodation requests only and cannot be used to inquire about the status of applications.

For our EEO Policy Statement, please click here. If you would like more information about your EEO rights as an applicant under the law, please click EEO is the LAW and the Supplement poster.

Arnold & Porter is an EO Employer – M/F/Veteran/Disability/Sexual Orientation/Gender Identity.

Arnold & Porter Kaye Scholer LLP uses E-Verify, which is a web-based system, to confirm the eligibility of our employees to work in the United States. As an E-Verify employer, we verify the identity and employment eligibility of newly hired employees by electronically matching information provided by employees on the Form I-9, Employment Eligibility Verification, against records available to the Social Security Administration (SSA) and the Department of Homeland Security (DHS). We use E-Verify because we are a federal contractor containing the Federal Acquisition Regulation (FAR) E-Verify clause. Please see the posters for details regarding E-Verify or contact Arnold & Porter’s Human Resources Department for more information. E-Verify Participation Poster and Right-to-work Poster.

Click here to apply: https://www.arnoldporter.com/en/careers/professional-staff/current-opportunities

Reply

610 Kaela StephanoNo Gravatar January 31, 2022 at 9:57 am

Can a Korean patent be examined under the PPH?

What is a practitioner required to do upon termination by applicant?
do they need to return all file contents and any fees paid in advance?

Reply

611 GraceNo Gravatar March 29, 2022 at 12:56 am

I’m looking to pass off my 2021 PLI materials. Check it out at: https://www.ebay.com/itm/185362207239 if you’re interested. Thanks!

Reply

Leave a Comment

{ 1 trackback }